You are on page 1of 160

.

com

NEET
Previous Year Papers with Solutions
(2014 to 2018)

Ask, Learn & Lead it


With Better Score, Better Career and Ultimately By Building Better India
with POWER of Your KNOWLEDGE

Eduncle.com
Call us : 9982853222
-
Page # 2 PREVIOUS YEAR PAPER [NEET - 2018]

PREVIOUS YEAR 2018 PAPER


BIOLOGY
1. What is the role of NAD+ in cellular respiration? 8. What type of ecological pyramid would be
obtained with the following data?
(A) It is a nucleotide source for ATP synthesis
Secondary consumer : 120 g
(B) It functions as an electron carrier
Primary consumer : 60 g
(C) It functions as an enzyme
Primary Producer : 10 g
(D) It is the final electron acceptor for
(A) Upright pyramid of numbers
anaerobic respiration
(B) Pyramid of energy
(C) Inverted pyramid of biomass
2. Oxygen is not produced during photosynthesis
(D) Upright pyramid of biomass
by
(A) Cycas
9. Natality refers to
(B) Nostac
(A) Number of individuals leaving the habitat
(C) Green sulphur bacteria
(B) Birth rate
(D) Chara
(C) Death rate
(D) Number of individuals entering a habitat
3. Double fertilization is
(A) Fusion of two male gametes with one
10. World Ozone Day is celebrated on
egg
(A) 16th September (B) 21st April
(B) Fusion of one male gamete with two polar
(C) 5th June (D) 22nd April
nuclei.
(C) Fusion of two male gametes of a pollen 11. In stratosphere, which of the following
tube with two different eggs elements acts as a catalyst in degradation
(D) Syngamy and triple fusion of ozone and release of molecular oxygen?
(A) Fe (B) Cl
4. In which of the following froms is iron (C) Carbon (D) Oxygen
absorbed by plants?
(A) Free element 12. Niche is
(B) Ferrous (A) the range of temperature that the organism
(C) Ferric needs to live
(D) Both ferric and ferrous (B) the physical space where an organism live
(C) all the biological factors in the organism's
5. Which of the following elements is responsible environment
for maintaining turgor in cells? (D) the functional role played by the organism
(A) Potassium (B) Sodium where it lives
(C) Magnesium (D) Calcium
13. Which of the following is a secondary pollutant?
6. Which one of the following plants shows a (A)SO2 (B) CO2
very close relationship with a species of moth, (C) CO (D) O3
where none of the two can complete, its life
cycle without the other? 14. Which of the following statement is correct?
(A) Banana (B) Yucca (A) Horsetails are gymnosperms
(C) Hydrilla (D) Viola (B) Saleginella is heterosporous while Salvinia
is homosporous
7. Pollen grains can be strored for several years (C) Ovules are not enclosed by ovary wall in
in liquid nitrogen having temperature of gymnosperms
(A) –196ºC (B) –80ºC (D) Stems are usually unbranched in both
(C) –120ºC (D) –160ºC Cycas and Cedrus.
PREVIOUS YEAR PAPER [NEET - 2018] Page # 3
15. Pneumatophores occurs in 24. Select the correct match:
(A) Carnivorous plants (A) T.H. Morgan - Transduction
(B) Free-floating hydrophytes (B) F2 x Recessive parent - Dihybrid cross
(C) Halophytes (C) Ribozyme - Nucleic acid
(D) Submerged hydrophytes (D) G. Mendel - Transformation

16. Sweet potato is a modified 25. The correct order of steps in Polymerase
(A) Tap root (B) Adventitious root Chain reaction (PCR) is
(C) Stem (D) Rhizome
(A) Denaturation, Extension, Annealing
(B) Annealing, Extension, Denaturation
17. Secondary xylem and phloem in dicot stem
(C) Extension, Denaturation, Annealing
are produced by:
(D) Denaturation, Annealing, Extension
(A) Phellogen (B) Vascular cambium
(C) Apical meristems (D) Axillary meristems
26. In India, the organisation responsible for
18. Select the wrong statement assessing the safety of introducing genetically
(A) Pseudopodia are locomotory and feeding modified organisms for public use is
structures in Sporozoans (A) Research Committee on Genetic
(B) Mushrooms belong to Basidiomycetes Manipulation (RCGM)
(C) Cell wall is present in members of Fungi (B) Council for Scientific and Industrial
and Plantae Research (CSIR)
(D) Mitochondria are the powerhouse of the (C) Indian Council of Medical Research (ICMR)
cell in all kingdoms except Monera. (D) Genetic Engineering Appraisal Committee
(GEAC)
19. Casparian strips occur in
(A) Cortex (B) Pericycle 27. The stage during which separation of the
(C) Epidermis (D) Endodermis paired homologous chromosomes begins is
(A) Diakinesis (B) Diplotene
20. Plants having little or no secondary growth
(C) Pachytene (D) Zygotene
are
(A) Conifers
28. The Golgi complex participates in
(B) Deciduous angiosperms
(A) Respiration in bacteria
(C) Grasses
(B) Formation of secretory vesicles
(D) Cycads
(C) Fatty acid breakdown
21. A 'new' variety of rice was patended by a (D) Activation of amino acid
foreign company, though such varieties have
been present in india for a long time. This is 29. Stomatal movement is not affected by
related to (A) O2 concentration (B) Light
(A) Lerma Rojo (B) Sharbati Sonora (C) Temperature (D) CO2 concentration
(C) Co-667 (D) Basmati
30. Stomata in grass leaf are
22. Which of the following is commonly used as (A) Rectangular
a vector for introducing a DNA fragment in (B) Kidney shaped
human lymphocytes? (C) Dumb-bell-shaped
(A) -phage (B) Ti plasmid (D) Barrel shaped
(C) Retrovirus (D) pBR 322
31. The two functional groups characteristic of
23. Use of bioresources by multinational companies
sugars are
and organisation without authorisation fromv
(A) carbonyl and phosphate
the concerned country and its people is called.
(B) carbonyl and methyl
(A) Biodegradation (B) Biopiracy
(C) hydroxyl and methyl
(C) Bio-Infringement (D) Bioexploitation
(D) carbonyl and hydroxyl
Page # 4 PREVIOUS YEAR PAPER [NEET - 2018]
32. Which of the following is not a product of containing a list of
light reaction of photosynthesis? characters and their
(A) NADPH (B) NADH alternate which
(C) ATP (D) Oxygen are helpful in
identification of
33. Which of the following is true for nucleolus? various taxa.
(A) It takes part in spindle formation a b c d
(B) It is a membrane bound structure (A) ii iv iii i
(C) Larger nucleoli are present in dividing cells (B) iii ii i iv
(D) Is is a site for active ribosomal RNA synthesis. (C) i iv iii ii
(D) iii iv i ii
34. Which among the following is not a prokaryote?
(A) Nostoc (B) Mycobacterium
(C) Saccharomyces (D) Oscillatoria 39. Which of the following flowers only once in
its life-time?
35. Winged pollen grains are present in (A) mango (B) Jackfruit
(A) Mango (B) Cycas (C) Bamboo species (D) Papaya
(C) Mustard (D) Pinus
40. Which of the following pairs is wrongly
36. After karyogamy followed by meiosis, spores matched?
are produced exogenously in (A) XO type sex determination - Grasshopper
(A) Agaricus (B) Alternaria (B) ABO blood grouping - Co-dominance
(C) Neurospora (D) Saccharomyces (C) Starch synthesis in pea - Multiple alleles
(D) T.H. Morgan - Linkage
37. Which one is wrongly matched?
(A) Gemma cups - Marchantia 41. Offsets are produced by
(B) Biflagellate zoospores - Brown algae (A) Parthenocarpy (B) Mitotic divisions
(C) Uniflagellate gametes - Polysiphonia (C) Meiotic division (D) Parthenogenesis
(D) Unicellular organism - Chlorella
42. Which of the following has proved helpful in
38. Match the items given in Column I with those preserving pollen as fossils?
in Column II and select the correct option (A) Oil content
given below: (B) Cellulosic intine
Column I Column II (C) Pollenkitt
a. Herbarium i. It is a place having
(D) Sporopollenin
a collection of
preserved plants and
43. Select the correct statement:
animals
(A) Spliceosomes take part in translation.
b. key ii. A list that
(B) Punnett square was developed by a British
enumerates
scientist.
methodically all the
(C) Franklin Stahl coined the term "linkage".
species found in an
area with brief (D) Transduction was discovered by S. Altman.
decription aiding
indentification. 44. The experimental proof for semiconservative
c. Museum iii. Is a place where replication of DNA was first shown in a
dried and pressed (A) Plant (B) Bacterium
plant specimens (C) Fungus (D) Virus
mounted on sheets
are kept 45. Select the correct match:
d. Catalogue iv. A booklet (A) Matthew Meselson and F. Stahl – Pisum sativum
(B) Alfred Hershey and Martha Chase – TMV
(C) Alec Jeffreys – Streptococcus
pneumoniae
(D) Francois Jacob and Jacques Monod – Lac operon
PREVIOUS YEAR PAPER [NEET - 2018] Page # 5
46. Match the items given in Column I with those 50. AGGTATCGCAT is a sequence from the coding
in Column II and select the correct option strand of a gene. What will be the corresponding
given below: sequence of the transcribed mRNA?
Column I Column II (A) ACCUAUGCGAU
a. Tidal volume i. 2500 – 3000 mL (B) UGGTUTCGCAT
b. Inspiratory Reserve (C) AGGUAUCGCAU
volume (D) UCCAUAGCGUA
c. Expiratrory Reserve iii. 500 – 550 mL
volume 51. According to Hugo de Vries, the mechanism
d. Residual volume iv. 1000 – 1100 mL of evolution is
a b c d (A) Phenotypic variations
(B) Saltation
(A) i iv ii iii
(C) iii i iv ii (C) Multiple step mutations
(C) iii ii i iv (D) Minor mutations
(D) iv iii ii i
52. Match the items given in Column I with those
in Column II and select the correct option
47. Which of the following options correctly
given below:
represents the lung conditions in asthma and
Column I Column II
emphysema respectively? a. Proliferative Phase i. Breakdown of
(A) Increased respiratory surface; Inflammation endometrial lining
of bronchioles b. Secretory Phase ii. Follicular Phase
(B) Increased number of bronchioles; c. Menstruation iii. Luteal Phase
Increased respiratory surface a b c
(C) Inflammation of bronchioles; Decreased (A) ii iii i
respiratory surface (C) i iii ii
(D) Decreased respiratory surface; Inflammation (C) iii ii i
of bronchioles (D) iii i ii

48. Match the items given in Column I with those 53. A woman has an X-linked condition on one of
her X chromosomes. This chromosome can
in Column II and select the correct option
be inherited by
given below:
(A) Only grandchildren
Column I Column II
(B) Only sons
a. Tricuspid valve i. Between left atrium
(C) Only daughters
and left ventricle
(D) Both sons and daughters
b. Bicuspid valve ii. Between right
ventricle and
54. Ciliates differ from all other protozoans in
pulmonary artery
(A) using pseudopodia for capturing prey
c. Semilunar valve iii. Between right (B) having a contractile vacuole for removing
atrium and right excess water
ventricle (C) using flagella for locomotion
a b c (D) having two types of nuclei
(A) i ii iii
(C) i iii ii 55. Indentify the vertebrate group of animals
(C) iii i ii characterized by crop and gizzard in its
(D) ii i iii digestive system.
(A) Aves (B) Reptilia
49. All of the following are part of an operon (C) Amphibia (D) Osteichthyes
except
(A) an enhancer 56. Which of the following organisms are known
(B) structural genes as chief producers in the oceans?
(C) an operator (A) Cyanobacteria (B) Diatoms
(D) a promoter (C) Dinoflagellates (D) Euglenoids
Page # 6 PREVIOUS YEAR PAPER [NEET - 2018]
57. Which of the following features is used to 63. Which of the following is an amino acid derived
indentify a male cockroach from a female
hormone ?
cockroach?
(A) Forewings with darker tegmina (A) Estradiol (B) Ecdysone
(B) Presence of caudal styles (C) Epinephrine (D) Estriol
(C) Presence of a boat shaped sternum on
the 9th abdominal segment 64. Match the items given in Column I with those
(D) Presence of anal cerci
in Column II and select the correct option
given below :
58. Which of the following animals does not
undergo metamorphosis? Column I Column II
(A) Moth (B) Tunicate a. Glycosuria i. Accumulation of uric
(C) Earthworm (D) Starfish acid in joints
b. Gout ii. mass of crystallised
59. Which one of these animals is not a salts within the kidney
homeotherm? c. Renal calculi iii. Inflammation in
(A) Camelus (B) Chelone
glomeruli
(C) Macropus (D) Psittacula
d. Glomerular nephritis iv. Presence of glucose
60. The transparent lens in the human eye is in urine
held in its place by a b c d
(A) smooth muscles attached to the iris (A) ii iii i iv
(B) ligaments attached to the iris (B) i ii iii iv
(C) ligaments attached to the cilliary body
(C) iii ii iv i
(D) smooth muscles attached to the ciliary body
(D) iv i ii iii
61. Which of the following structure or reigions
is incorrectly paired with its function ? 65. Match the items given in Column I with those
(A) Hypothalamus : Production of in column II and select the correct option
releasing hormones given below
and regulation of
Column I Column II
temperature, hunger
(Function) (Part of Excretory system)
and thirst
(B) Limbic system : consists of fibre a. Ultrafiltration i. Henle's loop
tracts that b. Concentration of urine ii. Ureter
interconnect c. Transport of urine iii. Urinary
different regions of bladder
brain; controls
d. Storage of urine iv. Malpighian
movement.
corpuscle
(C) Medulla oblongata : controls respiration
and cardiovascular v. Proximal
reflexes convoluted
(D) Corpus callosum : band of fibres tubule
connecting left and
a b c d
right cerebral
hemispheres (A) v iv i ii
(B) iv i ii iii
62. Which of the following hormones can play a (C) iv v ii iii
significant role in osteoporosis ? (D) v iv i iii
(A) Estrogen and Parathyroid hormone
(B) Progesterone and Aldosterone 66. Which of the following gastric cells indirectly
(C) Aldosterone and Prolactin help in erythropoiesis ?
(D) Parathyroid hormone and Prolactin
(A) Goblet cell (B) Mucous cell
(C) Chief cells (D) Parietal cells
PREVIOUS YEAR PAPER [NEET - 2018] Page # 7
67. Match the items given in Column I with those 73. Many ribosomes may associate with a single
in Column II and select the correct option mRNA to form multiple copies of a polypeptide
tgiven below : simultaneously. Such strings of ribosomes an
Column I Column II termed as
a. Fibrinogen i. Osmotic (A) Plastidome (B) polyhedral bodies
balance (C) Polysome (D) Nucleosome
b. Globulin ii. Blood
clotting 74. Which of the following events does not occur
c. Albumin iii. Defence in rough endoplasmic reticulum ?
mechanism (A) Cleavage of signal peptide
a b c (B) Protein glycosylation
(A) i iii ii (C) Protein folding
(B) i ii iii
(D) Phospholipid synthesis
(C) iii ii i
(D) ii iii i 75. Which of the following terms describe human
dentition ?
68. Calcium is important in skeletal muscle (A) Pleurodont, Monophyodont, homodont
contraction because it (B) Thecodont, Diphyodont, Heterodont
(A) detaches the myosin head from the actin (C) Thecodont, Diphyodont, Homodont
filament. (D) pleurodont, diphyodont, heterodont
(B) activates the myosin ATPase by binding
to it.
76. In a growing population of a country
(C) binds to troponin to remove the masking
(A) reproductive and pre-reproductive
of active sites on actin for myosin individuals are equal in number
(D) prevents the formation of bonds between (B) reproductive individuals are less than the
the myosin cross bridges and the actin filament post-reproductive individuals
(C) pre-reproductive individuals are more than
69. Which of the following is an occupational the reproductive individuals
respiratory disorder ? (D) pre-reproductive, individuals are less than
(A) Botulism (B) Silicosis the reproductive individuals
(C) Anthracis (D) Emphysema
77. Match the items given in Column I with those
in Column II and select the correct option
70. Which of these statements is incorrect ?
given below :
(A) Glycolysis operates as long as it is supplie
Column I Column II
with NAD that can pick up hydrogen atoms
a. Eutrophication i. UV-B radiation
(B) Glycolysis occurs in cytosol
b. Sanitary landfill ii. Deforestation
(C) Enzymes of TCA cycle are present
mitochondrial matrix c. Snow blindness iii.nutrient enrichment
(D) Oxidative phosphorylation takes place d. Jhum cultivation iv. Waste disposal
a b c d
outer mitochondrial membrane
(A) iii iv i ii
71. Nissl bodies are mainly composed of (B) i iii iv ii
(A) Nucleic acids and SER (C) ii i iii iv
(B) DNA and RNA (D) i ii iv iii
(C) Proteins and lipids
(D) Free ribosomes and PER 78. Which part of poppy plant is used to obtain
72. Select the incorrct match : the drug "Smack" ?
(A) Submetacentric Chromosomes - L-shaped (A) Roots (B) Latex
chromosomes (C) Flowers (D) Leaves
(B) Allosomes - Sex
79. Which one of the following population
chromosomes
interactions is widely used in medical science
(C) Lampbrush chromosomes - Diplotene
for the production of antibiotics ?
bivalents
(A) Parasitism (B) Mutualism
(D) Polytene chromosomes - Oocytes of
(C) Commensalism (D) Amensalism
amphibians
Page # 8 PREVIOUS YEAR PAPER [NEET - 2018]
80. All of the following are included in Ex-situ 86. In which disease does mosquito transmitted
conservation' except pathogen cause chronic inflammation of
(A) Botanical gardens lymphatic vessels ?
(B) Sacred groves (A) Ringworm disease (B) Ascariasis
(C) Wildlife safari parks (C) Elephantiasis (D) Amoebiasis
(D) Seed banks
87. Which of the following is not an autoimmune
81. Hormones secreted by the placenta to disease ?
maintain pregancy are (A) Alzheimer's disease
(A) hCG, hPL, progestogens, estrogens (B) Rheumatoid arthritis
(B) hCG, hPL, estrogens, relaxin, oxytocin (C) Psoriasis
(D) Vitiligo
(C) hCG, hPL, progestogens, prolactin
(D) hCG, progestogens, estrogens, glucocorticoids 88. Which of the following characteristics represent
'Inheritance of blood groups' in humans ?
82. The contraceptive "SAHELI' a. Dominance
(A) is an IUD b. Co-dominance
(B) increases the concentration of estrogen c. Multiple allele
and prevents ovulation in females d. Incomplete dominance
(C) block estrogen receptors in the uterus, e. Polygenic inheritance
preventing eggs from getting implanted (A) b,d and e (B) a,b and c
(D) is a post-coital contraceptive (C) b,c and e (D) a,c and e

89. Among the following sets of examples for


83. The difference between spermiogenesis and
divergent evolution, select the incorrect option
spermiation is (A) Brain of bat, man and cheetah
(A) In spermiogenesis spermatozoa from (B) Heart of bat, man and cheetah
sertoli cells are released into the cavity of (C) Forelimbs of man, bat and cheetah
seminiferous tubules, while in spermiation (D) Eye of octopus, bat and man
spermatozoa are formed.
(B) In spermiogenesis spermatozoa are 90. Conversion of milk to curd improve its
formed, while in spermiation spermatids are nutritional value by increasing the amount of
formed (A) Vitamin B12 (B) Vitamin A
(C) In spermiogenesis spermatids are formed, (C) Vitamin D (D) Vitamin E
while in spermiation spermatozoa are formed
(D) In spermiogenesis spermatozoa are
formed, while in spermiation spermatozoa are
released from sertoli cells into the cavity of
seminiferous tubules.

84. The amnion of mammalian embryo is derived


from
(A) mesoderm and trophoblast
(B) endoderm and mesoderm
(C) ectoderm and mesoderm
(D) ectoderm and endoderm

85. The similarity of bone structure in the forelimbs


of many vertebrates is an example of
(A) Convergent evolution
(B) Analogy
(C) Homology
(D) Adaptive radiation
PREVIOUS YEAR PAPER [NEET - 2018] Page # 9

PHYSICS
1. The volume (V) of a monatomic gas varies 6. In Young's double slit experiment the separation
with its temperature (T), as shown in the d between the slits is 2mm, the wavelength 
graph. The ratio of work done by the gas, to of the light used is 5896 Å and distance D
the heat absorbed by it, when it undergoes between the screen and slits is 100 cm. It is
a change from state A to state B, is found that the angular width of the fringes is
0.20°. To increase the fringe angular width to
0.21° (with same  and D) the separation
between the slits needs to be changed to
(A) 2.1 mm (B) 1.9mm
(C) 1.8mm (D) 1.7 mm

7. An astronomical refracting telescope will have


large angular magnification and high angular
1 2 resolution, when it has an objective lens of
(A) (B) (A) large focal length and large diameter
3 3
(B) large focal length and small diameter
2 2 (C) small focal length and large diameter
(C) (D) (D) small focal length and small diameter
5 7
8. The kinetic energies of a planet in an elliptical
2. The fundamental frequency in an open organ orbit about the Sun, at positions A, B and C are
pipe is equal to the third harmonic of a closed KA, KB and KC, respectively. AC is the major axis
organ pipe If the length of the closed organ pipe and SB is perpendicular to AC at the position of
is 20 cm, the length of the open organ pipe is the Sun S as shown in the figure. Then
(A) 12.5 cm (B) 8 cm
(C) 13.2 cm (D) 16 cm

3. The efficiency of an ideal heat engine working


between the freezing point and boiling point (A) KB<KA<KC (B) KA>KB>KC
of water, is (C) KA<KB<KC (D) KB>KA>KC
(A) 6.25% (B) 20%
(C) 26.8% (D) 12.5% 9. A solid sphere is in rolling motion. In rolling
motion a body possesses translational kinetic
4. At what temperature will the rms speed of energy (Kt) as well as rotational kinetic energy
oxygen molecules become just sufficient for (Kr) simultaneously. The ratio Kt : (Kt + Kr)
escape from the Earth's atmosphere? (Given : for the sphere is
Mass of oxygen molecule (m) = 2.76×10-26kg (A) 10:7 (B) 5:7
(C) 7:10 (D) 2:5
(A) 5.016×104 K (B) 8.360×104 K
(C) 2.508×104 K (D) 1.254×104 K
10. If the mass of the Sun were ten times smaller
and the universal gravitational constant were
5. Unpolarised light is incident from air on a plane ten times larger in magnitude, which of the
surface of a material of refractive index ''. At following is not correct?
a particular angle of incidence 'i', it is found (A) Time period of a simple pendulum on the
that the reflected and refracted rays are Earth would decrease.
perpendicular to each other, Which of the (B) Walking on the ground would become more
following options is correct for this situation? difficult.
(C) Raindrops will fall faster.
1  1  (D) 'g' on the Earth will not change.
(A) i  sin  
 
11. A solid sphere is rotating freely about its
(B) Reflected light is polarised with its electric
symmetry axis in free space. The radius of
vector perpendicular to the plane of incidence
the sphere is increased keeping its mass same.
(C) Reflected light is polarised with its electric Which of the following physical quantities
vector parallel to the plane of incidence would remain constant for the sphere?
(A) Rotational kinetic energy
1  1 
(D) i  tan   (B) Moment of inertia
  (C) Angular velocity
(D) Angular momentum
Page # 10 PREVIOUS YEAR PAPER [NEET - 2018]
12. A metallic rod of mass per unit length 0.5 kg 17. In a p-n junction diode, change in
m-1 is lying horizontally on a smooth inclined temperature due to heating
plane which makes an angle of 30° with the (A) does not affect resistance of p-n junction
horizontal. The rod is not allowed to slide (B) affects only forward resistance
down by flowing a current through it when a (C) affects only reverse resistance
magnetic field of induction 0.25 T is acting (D) affects the overall V – I characteristics
on it in the vertical direction. the current of p-n junction.
flowing in the rod to keep it stationary is
(A) 14.76 A (B) 5.98 A 18. In the combination of the following gates the
(C) 7.14 A (D) 11.32 A output Y can be written in terms of inputs A
and B as
13. An inductor 20 mH, capacitor 100 F and a
resistor 50  are connected in series across
a source of emf, V = 10 sin 314 t. The power
loss in the circuit is
(A) 2.74 W (B) 0.43 W
(C) 0.79 W (D) 1.13 W

14. A thin diamagnetic rod is placed vertically (A) A.B+A.B (B) A.B  A.B
between the poles of an electromagnet. (C) A.B (D) A  B
When the current in the electromagnet is
switched on, the the diamagnetic rod is
pushed up, out of the horizontal magnetic 19. A carbon resistor of  47  4.7  k is to be marked
field. Hence the rod gains gravitational with rings of different colours for its indentification.
potential energy. The work required to do The colour code sequence will be
this comes from
(A) Yellow-Green - Violet-Gold
(A) the lattice structure of the material of
(B) Yellow-Violet-Orange-Silver
the rod
(C) Violet-Yellow-Orange-Silver
(B) the magnetic field
(D) Green-Orange -Violet -Gold
(C) the current source
(D) the induced electric field due to the
20. A set of 'n' equal resistors, of value 'R' each,
changing magnetic field
are connected in series to a battery of emf
15. Current sensitivity of a moving coil 'E' and internal resistance 'R'. The current
galvanometer is 5 div/mA and its voltage drawn is I. Now, the 'n' resistors are
sensitivity (angular deflection per unit voltage connected in parallel to the same battery.
applied) is 20 div/V. The resistance of the Then the current drawn from battery becomes
galvanometer is 10 I.The value of 'n' is
(A) 250 (B) 25 (A) 20 (B) 11
(C) 40 (D) 500 (C) 10 (D) 9

16. In the circuit shown in the figure, the input 21. A battery consists of a varibale number 'n' of
voltage Vi is 20 V, VBE = 0 and VCE = 0. The identical cells (having internal resistance 'r' each)
values of IB, IC and  are given by which are connected in series. The terminals of
the battery are short-circuited and the current
I is measured. Which of the graphs shows the
correct relationship between I and n ?
I I

(A) (B)
O n O n

I I

(A) IB = 20A, IC = 5mA,  = 250


(B) IB = 25A, IC = 5mA,  = 200 (C) (D)
(C) IB = 40A, IC = 10mA,  = 250 O O
n n
(D) IB = 40A, IC = 5mA,  = 125
PREVIOUS YEAR PAPER [NEET - 2018] Page # 11
22. A body initially at rest and sliding along a 27. The electrostatic force between the metal
frictionless track from a height h (as shown plates of an isolated parallel plate capacitor
in the figure) just completes a vertical circle
C having a charge Q and area A is
of diameter AB =D . The height h is equal to
(A) proportional to the square root of the
distance between the plates.
(B) linearly proportional to the distance
between the plates.
h
(C) independent ofthe distance between the
B
plates.
D
(D) inversely proportional to the distance
A
between the plates.
7
(A) D (B) D 28. A pendulum is hung from the roof of a
5
sufficiently high building and is moving freely
3 5 to and fro like a simple harmoni coscillator.
(C) D (D) D
2 4 The acceleration of the bob of the pendulum
is 20 m/s2 at a distance of 5 m from the mean
23. Three objects, A : (a solid sphere), B :
position. The time period of oscillation is
(a thin circular disk) and C: (a circular ring),
each have the same mass M and radius R. (A) 2 s (B)  s
They all spin with the same angular speed  (C) 2 s (D) 1 s
about their own symmetry axes. The amounts
of work (W) required to bring them to rest, 29. An electron falls from rest through a vertical
would satisfy the relation. distance h in a uniform and vertically upward
(A) WB > WA > WC (B) WA > WB > WC
directed electric field E. The direction of electric
(C) WC > WB > WA (D) WA > WC > WB
field is now reversed, keeping its magnitude
24. Which one of the following statements is the same . A proton is allowed to fall from rest
incorrect ? in it through the same vertical distance h. The
(A) Friction force opposes the relative motion time of fall of the electron, in comparision to
(B) Limiting value of static friction is directly the time of fall of the proton is.
proportional to normal reaction
(A) 10 times greater
(C) Rolling friction is smaller than sliding
(B) 5 times greater
friction .
(D) Coefficient of sliding friction has dimensions (C) smaller
of length. (D) equal

25. A moving block having mass m, collides with 30. An electron of mass m with aninitial velocity
another stationary block having mass 4m.
the lighter block comes to rest after collision. 
V  V0ˆi  V0  0  enters an electric fie
When the initial velocity of the lighter block
is v, then the value of coefficient of restitution

(e) will be E  E0ˆi E0  cons tan t  0  at t = 0. If 0
(A) 0.8 (B) 0.25
(C) 0.5 (D) 0.4 its de-Broglie wavelength intially, then de-
Broglie wavelength initially, then de-Broglie
26. A tuning fork is used to produce resonance wavelength at time is
in a glass tube. The length of the air column
in this tube can be adjusted by a variable  eE0 
piston. At room temperature of 27°C two (A) 0t (B) 0 1  mV t 
successive resonances are produced at 20  0 
cm and 73 cm of column length. If the
frequency of the tunning fork is 320 Hz, the 0
velocity of sound in air at 27°C is  eE0 
(A) 350 m/s (B) 339 m/s (C) 1  t (D) 0
 mV 0 
(C) 330 m/s (D) 300 m/s
Page # 12 PREVIOUS YEAR PAPER [NEET - 2018]
31. For a radioactive material, half -life 10 37. An object is placed at a distance of 40 cm
minutes. If initially there are 600 number from a concave mirror of focal length 15 cm.
nuclei, the time taken (in minutes) for the If the object is displaced through a distance
disintegration of 450 nuclei is of 20 cm towards the mirror, the displacement
(A) 30 (B) 10 of the image will be
(C) 20 (D) 15 (A) 30 cm towards the mirror
(B) 36 cm away from the mirror
32. When the light of frequency 20 (where 0 (C) 30 cm away from the mirror
threshold frequency), is incident on a metal (D) 36 cm towards the mirror
plate, the maximum velocity of electrons
emitted is v1. When the frequency of the 38. A toy car with charge q moves on a frictionless
horizontal plane surface under the influence
incident radiation is increased to 50, the 
of a uniform electric field E . Due to the force
maximum velocity of electrons emitted from

the same plate is v2. The ratio of v1 to v2 is qE , its velocity increases from 0 to 6 m/s in
(A) 4 : 1 (B) 1 : 4 one second duration. At that instant the
(C) 1 : 2 (D) 2 : 1 direction of the field is reversed. The car
continues to move for two more seconds
under the influence of this field. The average
33. The ratio of kinetic energy to the total energy
velocity and the average speed of the toy
an electron in a Bohr orbit of the hydrogen car between 0 to 3 seconds are respectively
atomic is (A) 1 m/s, 3.5 m/s (B) 1 m/s, 3 m/s
(A) 2 : –1 (B) 1 : –1 (C) 2 m/s, 4 m/s (D) 1.5 m/s, 3 m/s
(C) 1 : 1 (D) 1 : –2
39. A block of mass m is placed on a smooth
inclined wedge ABC of inclination  as shown
34. An em wave is propagating in a medium with
in the figure. The wedge is given an

a velocity V  V î . The instantaneous acceelration 'a' towards the right. The relation
oscillating electricfield of this em wave is between a and  for the block to remain
stationary on the wedge is -
along +y axis. Then the direction of oscillating
magnetic field of the em wave will be along.
A
(A) –y direction (B) +z direction
m
(C) –z direction (D) –x direction

35. The refractive index of the material of a prism


a
is 2 and the angle of the prism is 30°. One
of the two refracting surfaces of the prism is
made a mirror inwards, by silver coating. A C B
beam of monochromatic light entering the
prism from the other face will retrace its path g
(A) a = g cos  (B) a =
(after reflection from the silvered surface) if sin 
its angle of incidence on the prism is - g
(A) 30° (B) 45° (C) a = (D) a = g tan 
cos ec
(C) 60° (D) zero
40. A student measured the diameter of a small
36. The magnetic potential energy stored in a steel ball using a screw gauge of least count
0.001 cm. The main scale reading is 5 mm
certain inductor is 25 mJ, when the current
and 0 of circular scale division coincides with
in the inductor is 60 mA. This inductor is of 25 divisions above the reference level. If
inductance screw gauge has a zero error of –0.004 cm,
(A) 1.389 H (B) 138.88 H the correct diameter of the ball is -
(C) 0.138 H (D) 13.89 H (A) 0.053 cm (B) 0.525 cm
(C) 0.521 (D) 0.529 cm
PREVIOUS YEAR PAPER [NEET - 2018] Page # 13
 44. A sample of 0.1 g of water at 1000C and
41. The moment of the force, F  4 î  5 ĵ  6k̂ at
normal pressure (1.013 × 105 Nm–2) requires
(2,0,–3), about the point (2, –2, –2), is given by 54 cal of heat energy to convert to steam
at 100°C C. If the volume of the steam
(A)  7 î  8 ĵ  4k̂ (B)  4 î  ĵ  8k̂
produced is 167.1 CC, the change in internal
(C)  8 î  4 ĵ  7k̂ (D)  7 î  4 ĵ  8k̂ energy of the sample, is -
(A) 42.4 J (B) 208.7 J
(C) 104.3 J (D) 84.5 J
42. The power radiated by a black body is P and
it radiates maximum energy at wavelength 0.
45. A small sphere of radius 'r' falls from rest in a
If the temperature of teh black body is now
viscous liquid. As a result, heat is produced
changed so that it radiates maximum energy due to viscous force. The rate of production
3 of heat when the sphere attains its termianl
at wavelength  , the power radiated by it
4 0 velocity, is proportional to -
becomes nP. The value of n is - (A) r5 (B) r2
(C) r3 (D) r4
256 4
(A) (B)
81 3

3 81
(C) (D)
4 256

43. Two wires are made of the same material and


have the same volume. The first wire has
cross-sectional area A and the second wire
has cross-sectional area 3A. If the length of
the first wire is increased by l on applying a
force F, how much force is needed to stretch
the second wire by the same amount ?
(A) 4 F (B) 6 F
(C) 9 F (D) F
Page # 14 PREVIOUS YEAR PAPER [NEET - 2018]

CHEMISTRY
1. Following solutions were prepared by mixing 6. Carboxylic acids have higher boiling points than
different volumes of NaOH and HCl of aldehydes, ketones and even alcholos of
different concentrations:
comparable molecular mass. It is due to their
M M (A) more extensive association of carboxylic
(a) 60 mL HCl + 40 mL NaOH
10 10
acid via van der Waals force of attraction
M M (B) formation of carboxylate ion
(b) 55 mL HCl + 45 mL NaOh
10 10 (C) formation of intramolecular H-bonding
M M (D) formation of intermolecular H-bonding
(c) 75 mL HCl + 25 mL NaOH
5 5
M M 7. Compound A, C8H10O, is found to react with
(d) 100 mL HCl + 100 mL NaOH NaOI (produced by reacting Y with NaOH)
10 10
pH of which one of them will be equal to 1? and yields a yellow precipitate with
(A) d (B) a characteristic smell.
(C) b (D) c A and Y are respectively
2. On which of the following properties does
the coagulating power of an ion depend? (A) CH CH3 and I2
(A) Both magnitude and sign of the charge
OH
on the ion
(B) Size of the ion alone
(C) The magnitude of the charge on the ion alone CH2 CH2 OH and I2
(B)
(D) The sign of charge on the ion alone

3. The solubility of BaSO4 in water is 2.42 x 10–


3
gL–1 at 298 K. The value of its solubility (C) H3C CH 2 OH and I2
product (Ksp) will be
(Given molar mass of BaSO4 = 233 g mo–1)
(A) 1.08 x 10–14 mol2 L–2 CH 3
(B) 1.08 x 10–12 mol2 L–2 (D) CH 3 OH and I2
(C) 1.08 x 10–10 mol2 L–2
(D) 1.08 x 10–8 mol2 L–2

4. Given van der Waals constant for NH3, H2, O2 8. Magnesium reacts with an element (X) to
and CO2 are respectively 4.17, 0.244, 1.36 form an ionic comnpound. If the ground state
and 3.59, which one of the following gases electronic configuration of (X) is 1s2 2s2 2p3,
in most easily liquefied ?
the simplest formula for this compound is
(A) O2 (B) H2
(C) NH3 (D) CO2 (A) Mg2X (B) MgX2
(C) Mg2X3 (D) Mg3X2
5. In the reaction
9. Iron exhibits bcc structure at room
OH O Na
+
temperature. Above 900°C, it transforms to
CHO fcc structure. The ratio fo density of iron at
room temperature to that at 900°C (assuming
+CHCl3+NaOH
molar mass and atomic radii of iron remains
constant with temperature) is
the electrophile involved is
(A) dichloromethyl anion (CHCl2) 3 3 4 3
(A) (B)
 4 2 3 2
(B) formyl cation (CHO)

(C) dichloromethyl cation (C HCl2 ) 3 1
(C) (D)
(D) dichlorocarbene (:CCl2) 2 2
PREVIOUS YEAR PAPER [NEET - 2018] Page # 15
10. Which one is a wrong statement ? 17. Considering Ellingham diagram, which of the
(A) The electronic configuration of N atom is following metals can be used to reduce
alumina?
(A) Mg (B) Zn
(C) Fe (D) Cu
(B) An orbital is designated by three quantum
numbers while an electron in an atom is
18. For the redox reaction
designated by four quantum numbers.
(C) Total orbital angular momentum of 
MnO  C2O 2  2
 H  Mn  CO2  H2O the
4 4
electron in 's' orbital is equal to zero.
correct coefficients of the reactants for the
(D) The value of m for d 2 is zero
z
balanced equation are:

11. Consider the following species : MnO4 C2O24 H


CN+, CN–, NO and CN
(A) 2 16 5
Which one of these will have the highest
(B) 2 5 16
bond order ?
(A) CN+ (B) CN– (C) 16 5 2
(C) NO (D) CN (D) 5 16 2

12. In the structure of CIF3, the number of lone 19. Which one of the following conditions will
pairs of electrons on central atom 'Cl' is: favour maximum formation of the product in
(A) four (B) two the reaction:
(C) one (D) three
A 2 (g)  B2  X2 (g) 2H  –X kJ
13. The correct order of atomic radii in group 13 (A) High temperature and high pressure
elements is: (B) Low temperature and low pressure
(A) B < Ga < Al < Tl < In (C) Low temperature and high pressure
(B) B < Al < Ga < In < Tl (D) High temperature and low pressure
(C) B < Al < In < Ga < Tl
(D) B < Ga < Al < In < Tl
20. The correction factor 'a' to the ideal gas
equation corresponds to:
14. The correct order of N-compounds in its
(A) electric field present between the gas
decreasing order of oxidation states is:
(A) HNO3, NH4Cl, No, N2 molecules
(B) HNO3, No, NH4Cl, N2 (B) volume of the gas molecules
(C) HNO3,NO, N2, NH4Cl (C) density of the gas molecules
(D) NH4Cl, N2, NO, HNO3 (D) forces of attraction between the gas
molecules
15. Which one of the following elements is unable
to form MF63 ion? 21. When initial concentration of the reactant is
doubled, the half-life period of a zero order
(A) B (B) Al
(C) Ga (D) In reaction:
(A) is tripled (B) is doubled
16. Which of the following statements is not true (C) is halved (D) remains unchanged
for halogens?
(A) All but fluorine show positive oxidation
22. The bond dissociation energies of X2, Y2 and
states
(B) All are oxidizing agents XY are in the ratio of 1:05:1. H for the
(C) All form monobasic oxyacids formation of XY is –200kJ mol–1. The bond
(D) Chlorine has the highest electron-gain dissociation energy of X2 will be:
enthalpy (A) 800 kJ mol–1 (B) 100 kJ mol–1
–1
(C) 200 kJ mol (D) 400 kJ mol–1
Page # 16 PREVIOUS YEAR PAPER [NEET - 2018]
23. Match the metal ions given in Column I with 29. Among CaH2, BeH2, BaH2, the order of ionic
the spin magnetic moments of the ions given character is
in Column II and assign the correct code : (A) BeH2 < BaH2 < CaH2
Column I Column I
(B) CaH2 < BeH2 < BaH2
a. Co3+ i. 8 B.M. (C) BeH2 < CaH2 < BaH2
b . Cr 3+
ii. 35 B.M. (D) BaH2 < BeH2 < CaH2
3+
c. Fe iii. 3 B.M. 30. Consider the change in oxidation state of
2+
d. Ni iv. 24 B.M. Bromine corresponding to different emf values
v. as shown in the diagram below :
15 B.M.
a b c d
(A) iv i ii iii
(B) i ii iii iv
(C) iv v ii i
(D) iii v i ii

24. Which one of the following ions exhibits d-d Then the species undergoing disproportionation
transition and paramagnetism as well ? is
(A) MnO4 (B) Cr2O72 (A) Br2 (B) BrO4–

(C) BrO3 (D) HBrO
(C) CrO24 (D) MnO24
31. In which case is the number of molecules of
25. The type of isomerism shown by the complex water maximum ?
[CoCl2(en)2] is
(A) 0.00224 L of water vapours at 1 atm and
(A) Ionization isomerism
(B) Coordination isomerism 273 K
(C) Geometrical isomerism (B) 0.18 g of water
(D) Linkage isomerism (C) 18 mL of water
(D) 10–3 mol of water
26. The geometry and magnetic behaviour of the
complex [Ni(CO)4] are
32. Which of the following molecules represents
(A) square planar geometry and paramagnetic
(B) tetrahedral geometry and diamagnetic the order of hybridisation sp2, sp2, sp, sp
(C) square planar geometry and diamagnetic from left to right atoms ?
(D) tetrahedral geometry and paramagnetic (A) CH2=CH–CH=CH2
(B) CH2=CH–CCH
27. Iron carbonyl, Fe(CO)5 is (C) HCC–CCH
(A) trinuclear (B) mononuclear
(D) CH3–CH=CH–CH3
(C) tetranuclear (D) dinuclear
33. Which of the following carbocations is
28. The correct difference between first and
second - order reactions is that expected to be most stable ?
(A) a first-order reaction can be catalyzed;
a second-order reaction cannot be catalyzed
(B) The half-life of a first-order reaction does
not depend on [A]0; the half-life of a second-
order reaction does depend on [A]0 (A) (B)
(C) The rate of a first-order reaction does
not depend on reactant concentrations; the
rate of a second order reaction does depend
on reactant concentrations
(D) The rate of a first-order reaction does
depend on reactant concentration; the rate
of a second-order reaction does not depend
on reactant concentrations
(C) (D)
PREVIOUS YEAR PAPER [NEET - 2018] Page # 17
34. Which of the following is correct with respect 39. The compound C7H8 undergoes the following
to –I effect of the substitutents ? (R = alkyl) reactions:
(A) –NH2 > –OR > –F (B) –NR2 < –OR < –F 3Cl2 / 
C7H8  Br2 /Fe
 A  Zn /HCl
 B  C
(C) –NH2 < –OR < –F (D) –NR2 > –OR > – F The product 'C' is:
(A) 3-bromo-2, 4, 6-trichlorotoluene
35. Identify the major products P, Q and R in this (B) o-bromotoluene
following sequence of reactions : (C) m-bromotoluene
(D) p-bromotoluene

40. Which oxide of nitrogen is not a common


pollutant introduced into the atmosphere
P Q R both due to natural and human activity?
(A) N2O (B) NO2
(C) N2O5 (D) NO
(A) , , CH3CH(OH)CH3
41. Regarding cross-linked or network polymers,
which of the following statements is incorrect?
(A) Examples are bakelite and melamine.
(B) They are formed from bi- and tri-functional
monomers.
(B) , , (C) They contain covalent bonds between
various liner polymer chains.
(D) They contain strong covalent bonds in
their polymer chains.

42. Which of the following oxides is most acidic


(C) , , CH3CH2–OH in nature?
(A) BaO (B) BeO
(C) MgO (D) CaO

43. Nitration of aniline is strong acidic medium


also gives m-nitroaniline because:
(D) , , CH3–CO–CH3 (A) In absence of substituents nitro group
always goes to m-position.
(B) In electrophilic substitution reactions
amino group is meta directive.
(C) In spite of substituents nitro group always
36. Which of the following compounds can form
goes to only m-position.
a zwitterion ?
(D) In acidic (strong) medium aniline is
(A) Benzoic acid (B) Acetanilide present as anilinium ion.
(C) Aniline (D) Glycine
44. A mixture of 2.3 g formic acid and 4.5 g oxalic
37. The compound A on treatment with Na gives acid is treated with conc. H2SO4. The evolved
B, and with PCl5 gives C. B and C react gaseous mixture is passed through KOH
together to give diethyl ether. A, B and C pellets. Weight (in g) of the remaining product
are in the order: at STP will be:
(A) C2H5Cl, C2H6, C2H5OH (A) 2.8 (B) 3.0
(B) C2H5OH, C2H5Cl, C2H5ONa (C) 1.4 (D) 4.4
(C) C2H5OH, C2H6, C2H5Cl
45. The difference between amylose and
(D) C2H5OH, C2H5ONa, C2H5Cl
amylopectin is:
(A) Amylopectin have 1  4 -linkage and
38. Hydrocarbon (A) reacts with bromine by 1  6 -linkage
substitution to form an alkyl bromide which (B) Amylose have 1  4 -linkage and 1  6
by Wurtz reaction is converted to gaseous -linkage
hydrocarbon containing less than four carbon (C) Amylopectin have 1  4 -linkage and
atoms (A) is: 1  6 -linkage
(A) CH3 – CH3 (B) CH2 = CH2 (D) Amylose is made up of glucose and
(C) CH  CH (D) CH4 galactose
Page # 18 PREVIOUS YEAR PAPER [NEET - 2018]

ANSWERKEY
BIOLOGY

1. B 2. C 3. D 4. C 5. A 6. B 7. A
8. C 9. B 10. A 11. B 12. D 13. D 14. C
15. C 16. B 17. B 18. A 19. D 20. C 21. D

22. C 23. B 24. C 25. D 26. D 27. B 28. B


29. A 30. C 31. D 32. B 33. D 34. C 35. D

36. A 37. C 38. D 39. C 41. B 42. D 43. B


44. B 45. D 46. B 47. C 48. C 49. A 50. C
51. B 52. A 53. D 54. D 55. A 56. B 57. B
58. C 59. B 60. C 61. B 62. A 63. C 64. D
65. B 65. B 66. D 67. D 68. C 69. B 70. D
71. D 72. D 73. C 74. D 75. B 76. C 77. A
78. B 79. D 80. B 81. A 82. C 83. D 84. C
85. C 86. C 87. A 88. B 89. D 90. A

PHYSICS

1 C 2 C 3 C 4 B 5 B 6 B 7 A

8 B 9 B 10 D 11 D 12 D 13 C 14 C

15 A 16 D 17 D 18 B 19 B 20 C 21 C

22 D 23 C 24 D 25 B 26 B 27 C 28 B

29 C 30 C 31 C 32 C 33 B 34 B 35 B

36 D 37 B 38 B 39 D 40 D 41 D 42 A

43 C 44 B 45 A

CHEMISTRY

1 D 2 C 3 C 4 C 5 D 6 D 7 A

8 D 9 A 10 A 11 B 12 B 13 D 14 C

15 A 16 Bonus 17 A 18 B 19 C 20 D 21 B

22 A 23 C 24 D 25 C 26 B 27 B 28 B

29 C 30 D 31 C 32 B 33 A 34 B,C 35 D

36 D 37 D 38 D 39 C 40 C 41 D 42 B

43 D 44 A 45 C
PREVIOUS YEAR PAPER [NEET - 2018] Page # 19

SOLUTION
PHYSICS
1. C 4. B
p = constant
30  1.38  10 23  T
w = nRT ....(1) = (11.2 × 10–3)2
Q = nCPT (for constant pressure) 2.76  10 26
T = 8.360 × 104
5 
Q = n  2 R  T ....(1)
  5. B
Equation (1)/(2)

nRT 2 Reflected ray


  Completely
5  5
n R T Polarised
2 

2. C

 Refracted Partially Polarised


ray
2

 This is the condition of polarisation in which



2 light is incident on an interface at Brewster's
   2 angle.

Thus reflected light is polarised with its E
v v
fopen   perpendicular to the plane of incidence.
 2

6. B

=
d

4 
' =
d'
v
Fundamental f  0.20 d'
4  =
0.21 d
As given :
0.20 d'
3v v 3 1  =
0.21 2
4l1 = 2l2 = 4  20 = 2l2
 d' = 1.904
= 13.2
7. A
3. C R.P  d
Efficiency of heat engine
f0
T2 ln = f
h=1– T e
1
 Greater the focal length of the objective,
273 more is the magnification.
=1– Also for a telescope, the objective lens must
373
have a bigger aperture for acquiring more
= 1 – 0.731
no. of rays from a distant object.
26.8%
Page # 20 PREVIOUS YEAR PAPER [NEET - 2018]

8. B
1
We know that as r decreases V ses =
(v × r = constant) 100   100  10 6
The nearest point in perigee which is point A. 100
Next nearest is B and then C =

 KA > KB > KC
= R 2  (XL  XC )2
9. B
2
2 2  100 
Isolid sphere = MR2 = (50)   2   
5   
1 1
KET = m (r)2 = mr22 = (50)2  (25.56)2
2 2
1 2 1 = 3153.31
KE(R+T) = mR22 + m (r)2
2 5 2 p = Vrms lrms cos 
1 1 Vm2R
= mR22 + mR22 =
5 2 z2
7  10 2 
= mR22  
10 =   × 50
 2 
1
mr 22  10 2 
KT 2    50
K T  KR = 7 mr 22 
p=  2 

10 3153 .31
5 p = 0.79 watt
=
7
14. C
As the material is diamagnetic and is placed
10. D
vertically between the electro magnet, it is
10GMe pushed up due to the magnetic field.
g' = = 10 g This happens due to the current source which
R2e
makes current flow in the electro-magnet.
 g changes
15. A
11. D
5 div  1 mA
As we see that the torque acting on the
20 div  4 mA
body is zero, thus we can say that angular
V = iR
momentum is conserved.
l = 4 × 10–3 × R
12. D 1
ilb cos 30° = mg sin 30° R=
4  10 3
3 i × l 0.25 = 0.5l × 10 1000
=
500 4
i= = 250 
0.25
11.32
16. D
20V
13. C
 = 314
= 100  RC
XL = L C
= 100n × 20 × 10–3 RB
XL = 2 20V
VCE=0
1
XC = E
C
PREVIOUS YEAR PAPER [NEET - 2018] Page # 21

20. C
20 20
IB = = E
500  103 5  10 4 I=
E  nR
= 4 × 10–5 Where parallel combination current 10 I is
= 40 A given by
20 E R
IC = = 5 mA + = 10 I
4  103 R n
  = 125 E R E 
+ = 10  R  nR 
R n  
17. D How according to problem
On  sing the temperature of a PN junction. 1 + n/1 + 1/n = 10
The resistance falls. 10 = (1 + n/n+1) n
Current  ses in both cases. The overall V-I n = 10
characteristics of a p-n junction changes.
21. C
I is independent of the no. of batteries as the
18. B
terminals of the battery are short circuited.
0 0 I
A
B 0 AND
x
NOT
OR
1
AND 1 n
1
22. D
A B Y D
0 0 0
0 1 1 D/2
h
1 0 1
1 1 0
1
X - OR gate mv2 = mgh
2

19. B v= 2gh

47 × 103 ± 10% 1 1 D
m ( 2gh )2 = mg + MgD
2 2 2

Yellow Violet Oragne Silver D


h= +D
4
R = (47 ± 4.7) K
= 47 × 103 ± 4700 5D
h=
R = 47 × 103 ± 10% 4
BBROYGBVGWGS
0123456789 23. C
B - Black A - Solid sphere
B - Thin circular disc
B - Brown
C - Circular ring
R - Red
As all are rotating about their own symmetry
O - Orange axis, they have rotational KE
Y - Yellow
G - Green 2
Iss = MR2
B - Blue 5
V - Voilet MR 2
G - Gray Idis =
2
W - White
Iring = MR2
Gold  IR > IDis > Iss
Silver wC > wB > wA
Page # 22 PREVIOUS YEAR PAPER [NEET - 2018]

24. D 29. C
Coefficient of sliding friction has dimensions
of length. qE
a=
m
25. B
Before : 1
a
m
v Rest
ae >> ap
te << tp
m 4m
After : 30. C
 
Rest
v = v0 î , E  E0 ĵ

m m E0et
v = v0 +
m
Pi = Pt
mv + 4m × 0 = 4mv' + m × 0
h
 mv = 4mv' =
mv
v
 v' = h
4
 = m v0  E0et 

v 2  v1 v  m 
e = u  u  ev =
1 2 4
h
1  
e= = 0.25  = mv0 1  E0et 
4  mv 
 0 

26. B 0  h 
=  0  
 eE0   mv0 
1   t
 mv 0 

31. C
N0 = 600
N0 N0
N0
T1 2 T1 4
v
f= 2 2

600  300  150
v 450 nuclie disintegration in 2 T1/2 life
320 =  2 × 10
106  10 2
 20 minute
v = 339 m/s
32. C
E =  + KEmax
27. C
hf = hf0 + KEmax
2 KEmax = hf – hf0
F = 2A
0 1
mv12 = h (2f0 – f0)
 The force is independent of the distance 2
between the plates. 1
mv12 = hf0 ....(1)
2
28. B
1
mv22 = h (5f0 – f0)
5 2
T = 2
20
1
T =  sec mv22 = 4hf0 ....(2)
2
PREVIOUS YEAR PAPER [NEET - 2018] Page # 23

On dividing equation (1) by (2) 36. D


1
1 LI2 = 25 × 10–3
mv12 2
2 hf0
1 = 4hf
mv22 0 2  25  10 3
2 L=
(60  10 3 )2

v12 1 V1 1 50  103
=  V = =
v22 4 2 2 36  10 4
= 1.389 × 10
33. B = 13.89 H

13.6 37. B
KE =
n2
f=–15cm
z2
TE = –13.6
n2
U=-40cm
13.6
=-
n2

KE
  1 : 1
TE

34. B In first condition


  
P  EB 1 1 1 1 1 1
+ =  – =–
v u f v 40 15
Direction of propgation in î (x direction) and
1 1 1
= –
Ê is given in ĵ direction ( ĵ) . So from right v 40 15
hand thumb rule B is in + z direction.
1 15  40
=
v 600
35. B
600
v=–  – 24 cm
25
30°
Second Condition
i
1 1 1
+ =
v u f

1 1 1
 – =–
v 20 15
n 2
1 1 1
r2 = 0 = –
v 20 15
r1 = A
1 × sin i = n sin A 1 15  20
=
v 300
sin i = 2 sin 30°
i = 45° 300
v=  – 60 cm
Here e = 0 5
(No emergence) Displacement of image = 60 – 24
= 36 cm
Page # 24 PREVIOUS YEAR PAPER [NEET - 2018]

38. B 40. D
LC = 0.001 cm
E ms R = 5 mm
E
x x Zero error = 0.25 × 0.001
= 0.025 cm
qE
a x D = 0.5 + 0.025 + 0.004
m
= 0.529 cm
t = 0, v = 0 and t = 1, v = 6
6 = 0 + at' 41. D
a=6
(2,0,–3)
1 
x= × 6 (1)2 r
2
=3
displacement = 3
(2,–2,–2)
distance = 9

 3  = r  F
vavg = = 1 m/s
3
ˆ
ˆi ˆj k
9
vavg = = 3 m/s 0 2 1
3 =
4 5 6

39. D
î(12  5)  ĵ(4)  k̂(8)


m
 r = (2  2)î  (0  2) ĵ  (3  2)k̂

=  7 î  4 ĵ  8k̂
a
= 2ˆj  kˆ

42. A
PT4
macos 4
P1  2 
P4  P =   M × T = constant
ma
 2  1 

 4
mg sin   1 
P2 = P1 ×  
mg cos   2 
mg

4
 0 
For block at rest; component along the wedge = P1   4 
3
 0 
should be equal along the wedge should be
equal
 macos= mgsint 256
P2 = P1 ×
81
 a  g tan 
PREVIOUS YEAR PAPER [NEET - 2018] Page # 25

43. C 45. A

F L
=
A L
fv

FL
 = (L) = constant
A
mg
F1L1 F2L 2
A1 = A 2
 4 3
A 2F1L1 6rv =   3 r  g
F2 = A L  
1 2
v  r2
Rate of heat generated
F1  L  3A
L p  (mg) v
= A = 9 F1
3 4
p v3 g v
3
44. B p  r5
208.7 J
Q = U + W
U = Q – W
U = 54 × 4.2 – P (V)
 54 × 4.2 – (1.013 × 105 × 167.1 × 10–6)
 208.7 Joule
Page # 26 PREVIOUS YEAR PAPER [NEET - 2018]

CHEMISTRY
1. D
9. A
bcc  fcc
1 1
N1 V1  N2 V2 75 x  25 x room temp. (900°C)
Nf = = 5 5
V1  V2 Z=2 Z=4
100

Nf = 0.1
2
 pH = log [H+] 2
Z xMM  4r / 3 
= –log 0.1
d
bcc NA x a3 4
= =
=1
d
fcc Z xMM 3
3  4r 
NA x a  
 2
2. C

3. C dbcc 3 3
dfcc = 4 2
BaSO4  Ba2+ + So24

s s
10. A
Ksp = S2 The correct electronic configuration of N–
2
 2.42 x103  atom is
=  = 1.08 x 10–10 mol2/lt2
 233 

4. C
'ease of liquification depends on' a s All e– should be in same spin
 NH3
11. B
5. D CN– total e– = 14
1s2, *1s2, 2s2, *2s2, 2px2 = 2py2, 2pz2

BMO  ABMO 10  4
:CCl2 (Dichloro carbene) BO = = =3
2 2

12. B

6. D 2 lone pair on Cl– atom

7. A
Haloform reaction
13. D
OH O B < Ga < Al < In < Tl
I2+NaOH +
—CH—CH3 CHI3 + —C—ONa
(A) (y)
14. C
(C8H10O)

8. D 15. A
valency fector for Mg = 2 thus formula Due to absence of vacant d-orbitals
valency fector for x = 5 of comp = M3X2
16. Bonus
PREVIOUS YEAR PAPER [NEET - 2018] Page # 27
17. A 24. D
Because graph for Gºf of BeO is below then
graph of G°f of Al2O3 in ellingham diagram. 25. C
[MA2(a–a)2] geometrical & optical
18. B
MnO4– + C2O42 + H+  Mn2+ + Co2 + H2O 26. B
Ni(CO)4
2MnO4– + 5C2O42– + 16 H+  2Mn2+ +
10Co2 + 8H2O
2 : 5 : 16

19. C
for exothermic reaction lower temperature 27. B
favours product Fe(CO)5 Only one central metal atom

20. D 28. B
a corresponds to the force of attraction 0.693
between the gas molecules. t 12 =
k
(independent of Initial concentration)
21. B
1
Initial concentration t 12 =
t 1 = k xInitialconcentration
2
2k (depends on initial concentration)
t 1 2  Initial concentration
29. C
22. A
1 1
x + y2  x y
2 2 2
30. D
1 1 As, BrO4– is present in its highest oxidation
Hrxn = Ex  x + Ey  y – Ex–y = Hf(x) state(+7), so it will not undergo disproportination,
2 2
For, BrO3–
1 0.5 2H2O + 2 BrO3–  2BrO4– + 4H+ + 4e– – (1),
 a+ a – a = –200
2 2 E1O = –1.82
1 1 4e– + 5H+ + BrO3– HBrO + 2H2O – (2), EO2
 a + a – a = –200
2 4 = 1.5V
3  (1) + (2) = (3)
 a – a = –200
4 H+ + 3 BrO3  2 BrO4 + HBrO – (3)

1 1.82 x 4  1.5 x 4
  a = –200 Eo3 =
4 4
 a = 800 kj/ mol = –ve
Also for HBrO
4e– + 6H+ + 2HBrO  2Br2 + 4H2O
23. C
Co3+ Cr3+ – (1), E1O = 1.595
3+
Fe Ni2+
2H2O + HBrO  BrO3 + 5H+ + 4e–
3d6 3d3
3d5 3d8 – (2), EO2 = –1.5 V
unpaired e– 4 3 (1) + (2) = 3
5 2
H+ + 3HBrO  2Br2 + 2H2O + BrO3 – (3)
= 24 = 15
1.595 x 4  1.5 x 4
 EO3 =
35 8 4
= +ve
= n n  2 BM So, HBrO will disproportionate
Page # 28 PREVIOUS YEAR PAPER [NEET - 2018]

31. C 37. D
(1) At NTP, 0.00224 lt  0.0001
mole
0.18
(2) n = = 0.01 mole
18
(3) dH2O = 1 gm/ml
 w = 18 gm 38. D
Br2 2Na
18 CH4 CH3—Br CH 3—CH3 (Less than four carbon)
So, n= = 1 mole hv ether
18 (A)
(4) 10–3 mole
 maximum no. of moles is present in 39. C
18 ml of H2O.

32. B

40. C

41. D
33. A
42. B
BeO due to palarisation

43. D
Refer to theory

44. A
HCOOH  Co + H2O(l)
34. B,C (g)
2.3 gm
—NH2< OR < –F (–I order) 2.3
n=
46
35. D 1 1 1
n= moles moles
20 20 20
moles
H2C2O4  Co + Co2 + H2O(l)
(g) (g)
4.5 gm
4.5
n=
90
1 1 1
n= moles moles
20 20 20
1
moles moles
20
 on passing through KOH only
Co will remain as Co2 is absorbed.

36. D
1 1
 nCo = + moles
20 20
1
wCo = x 28
10
wCo = 2.8 gms.

45. 3
PREVIOUS YEAR PAPER [NEET - 2017] Page # 29

PREVIOUS YEAR 2017 PAPER


BIOLOGY
1. Which one of the following statements is 9. Coconut fruit is a :
correct, with reference to enzymes? (A) Berry (B) Nut
(A) Holoenzyme = Apoenzyme + Coenzyme (C) Capsule (D) Drupe
(B) Coenzyme = Apoenzyme + Holoenzyme
10. Adult human RBCs are enucleate. Which of
(C) Holoenzyme = Coenzyme + Co-factor
following statement(s) is/are most apropriate
(D) Apoenzyme = Holoenzyme + Coenzyme explanation for this feature?

2. A decrease in blood pressure/volume will not (a) They do not need to reproduce

cause the release of : (b) They are somatic cells

(A) Atrial Natriuretic Factor (c) They do not metabolize


(B) Aldosterone (d) All their internal space is available for
(C) ADH oxygen transport
(D) Renin Options :
(A) Only (a) (B) (a), (c) and (d)
3. Which cells of 'Crypts of Lieberkuhn' secrete (C) (b) and (c) (D) Only (d)
antibacterial lysozyme?
(A) Paneth cells (B) Zymogen cells 11. Capacitation occurs in :
(C) Kupffer cells (D) Argentaffin cells (A) Epididymis
(B) Vas deferens
4. Which of the following are not polymeric? (C) Female Reproductive tract
(A) Proteins (B) Polysaccharides (D) Rete testis
(C) Lipids (D) Nucleic acids
12. Which of the following are found in extreme
5. Functional megaspore in an angiosperm de- saline conditions?
velop into : (A) Eubacteria (B) Cyanobacteria
(A) Endosperm (B) Embryo sac (C) Mycobacteria (D) Archaebacteria
(C) Embryo (D) Ovule
13. Asymptote in a logistic growth curve is ob-
6. Myelin sheath is produced by : tained when :
(A) Astrocytes and Schwann Cells (A) K = N (B) K > N
(B) Oligodendrocytes and Osteoclasts (C) K < N
(C) Osteoclasts and Astrocytes (D) The value of r approaches zero
(D) Schwann cells and Oligodendrocytes
14. Artificial selection to obtain cows yielding
7. Attractants and rewards are required for : higher milk output reprepresents :
(A) Entomophily (B) Hydrophily (A) directional as it pushes the mean of the
(C) Cleistogamy (D) Anemophily character in one direction
(B) disruptive as it splits the population into
8. Receptor sites for neurotransmitters are pre- two, one yielding higher output and the other
sents on lower output.
(A) pre-synaptic membrane (C) stabilizing followed by disruptive as it sta-
(B) tips of axons bilizes the population to produce higher yield-
(C) post-synaptic membrane ing cows
(D) membranes of synnaptic vesicles (D) stabilizing selection as it stabilizes this
character in the population.
Page # 30 PREVIOUS YEAR PAPER [NEET - 2017]
15. Select the mismatch : 21. The genotypes of Husband and Wife are IAIB
(A) Rhodospirillum - Mycorrhiza and IAi.
(B) Anabaena - Nitrogen fixer Among the blood types of their children, how
(C) Rhizobium - Alfalfa many different genotypes and phenotypes
(D) Frankia - Alnus are possible ?
(A) 3 genotypes; 4 phenotypes
16. Good vision depends on adequate intake of
carotene rich food. . (B) 4 genotypes; 3 phenotypes.

Select the best option from the following (C) 4 genotypes; 4 phenotypes.
statements. (D) 3 genotypes; 3 phenotypes.
(a) Vitamin A derivatives are formed from
carotene. 22. Which of the following facilitates opening of
(b) The photopigments are embedded in the stomatal aperture ?
membrane discs of the inner segment. (A) Decrease in turgidity of guard cells
(c) Retinal is a derivative of Vitamin A (B) Radial orientation of cellulose microfibrils
(d) Retinal is a light absorbing part of all the in the cell wall of guard cells
visual photopigments. (C) Longitudinal orientation of cellulose mi-
Options : crofibrils in the cell wall of guard cells
(A) (a), (c) and (d) (B) (a) and (c) (D) Contraction of outer wall of guard cells
(C) (b), (c) and (d) (D) (a) and (b)
23. In Bougainvillea throns are the modification
17. The DNA fragments separated on an agar- of :
ose gel can be visualised after staining with (A) Adventitious root (B) Stem
: (C) Leaf (D) Stipules
(A) Acetocarmine (B) Aniline blue
(C) Ethidium bromide (D) Bromophenol blue 24. Which one of the following is related to Ex-
situ conservation of threatened animals and
18. The hepatic portal vein drains blood to liver plants?
from :
(A) Biodiversity hot spots
(A) Stomach (B) Kidneys (B) Amazon rainforest
(C) Intestine (D) Heart
(C) Himalayan region
19. The vascular cambium normally gives rise to (D) Wildlife Safari parks
:
25. Root hairs develop from the region of :
(A) Primary phloem (B) Secondary xylem
(A) Elongation
(C) Periderm (D) Phelloderm
(B) Root cap
20. Thalassemia and sickle cell anemia are caused (C) Meristematic activity
due to a problem in globin molecule synthe- (D) Maturation
sis. Select the correct statement :
(A) Both are due to quantitative defect in 26. A disease caused by an autosomal primary
globin chain synthesis. non-disjunction is :
(B) Thalassemia is due to less synthesis of (A) Klinefelter's Syndrome
globin molecules. (B) Turner's Syndrome
(C) Sickle cell anemia is due to a quantita- (C) Sickle Cell Anemia
tive problem of globin molecules. (D) Down's Syndrome
(D) Both are due to a qualitative defect in
globin chain synthesis.
PREVIOUS YEAR PAPER [NEET - 2017] Page # 31
27. The water potential of pure water is : Options :
(A) Less than zero (a) (b) (c) (d)
(B) More than zero but less than one (a) (b) (c) (d)
(C) More than one (A) (iii) (iv) (i) (ii)
(D) Zero (B) (iv) (ii) (iii) (i)
(C) (iv) (iii) (ii) (i)
28. Which of the following options gives the cor-
(D) (ii) (iii) (iv) (i)
rect sequence of events during mitosis?
(A) condensation  nuclear membrane dis- 34. The function of copper ions in copper re-
assembly  arrangement at equator  cen- leasing IUD's is :
tromere division  segregation  te- (A) They inhibit gametogenensis
lophase
(B) They make uterus unsuitable for implan-
(B) condensation  crossing over  nuclear
tation
membrane disassembly  segregation  te-
(C) They inhibit ovulation
lophase
(D) They suppress sperm motility and
(C) condensation  arrangement at equator
fertilising capacity of sperms
 centromere division  segregation  te-
lophase 35. Which of the following in sewage treatment
(D) condensation  nuclear membrane dis- removes suspended solids ?
assembly  crossing over  segregation  (A) Secondary treatment
telophase (B) Primary treatment
29. The process of separation and purification (C) Sludge treatment
of expressed protein before marketing is called (D) Tertiary treatment
(A) Downstream processing
36. An important characteristic that Hemichor-
(B) Bioprocessing
dates share with Chordates is :
(C) Postproduction processing
(D) Upstream processing (A) ventral tubular nerve cord
(B) pharynx with gill slits
30. A temporary endocrine gland in the humnan (C) pharynx without gill slits
body is : (D) absence of notochord
(A) Corpus cardiacum(B) Corpus luteum
(C) Corpus allatum (D) Pineal gland 37. The final proof of DNA as the genetic mate-
rial came from the experiments of :
31. Which of the following is made up of dead
(A) Hershey and Chase
cells ?
(B) Avery, Mcleod and McCarty
(A) Collenchyma (B) Phellem
(C) Hargobind Khorana
(C) Phloem (D) Xylem parenchyma
(D) Griffith
32. An example of colonial aloga is :
38. Among the following characters, which one
(A) Volvox (B) Ulothrix
was not considered by Mendel in his experi-
(C) Spirogyra (D) Chlorella
ments on pea ?
33. Match th following sexually transmitted dis- (A) Trichomes-Glandular or non-glandular
eases (Column-I) with their causative agent (B) Seed-Green or Yellow
(Column-II) and select the correct option. (C) Pod-Inflated or Constricted
Column-I Column-II (D) Stem-Tall or Dwarf
(a) Gonorrhea (i) HIV
(b) Syphilis (ii) Neisseria 39. Plants which produce characteristic pneumato-
(c) Genital Warts (iii) Treponema phores and show vivipary belong to :
(d) AIDS (iv) Human Papilloma- (A) Halophytes (B) Psammophytes
Virus (C) Hydrophytes (D) Mesophytes
Page # 32 PREVIOUS YEAR PAPER [NEET - 2017]
40. The pivot joint between atlas and axis is a 47. Select the correct route for the passage of
type of : sperms in male frogs :
(A) cartilaginous joint (B) synovial joint (A) Testes  Vasa efferentia  Kidney 
(C) saddle joint (D) fibrous joint Seminal Vesicle  Urinogenital duct  Cloaca
(B) Testes  Vasa efferentia  Bidder's ca-
41. With reference to factors affecting the rate
nal  Ureter  Cloaca
of photosynthiesis, which of the
(C) Testes  Vasa efferentia  Kidney 
following statemets is not correct ?
Bidder's canal  Urinogenital duct  Cloaca
(A) Increasing atmopheric CO2 concentration
up to 0.05% can enhance CO2 fixation rate (D) Testes  Bidder's canal  Kidney  Vasa
(B) C3 Plant respeond to higher temperature efferentia  Urinogenital duct  Cloaca
with enhanced photosynthesis while
48. In case of a couple where the male is having
C4 plants have much lower temperature optimum
a very low sperm count, which techniqure
(C) Tomato is greenhouse crop which can be
will be suitable for fertilisation ?
grown in CO 2 - enriched atmosphere
(A) Gamete intracytoplasmic fallopian transfer
for higher yield
(B) Artificial Insemination
(D) Light saturation for CO2 fixation occurs
at 10% of full sunlight (C) Intracytoplasmic sperm injection
(D) Intrauterine transfer
42. DNA fragments are :
(A) Negatively charged 49. Which ecosystem has the maximum biomass?
(B) Netural (A) Grassland ecosystem
(C) Either positively or negatively charged (B) Pond ecosystem
depending on their size (C) Lake ecosystem
(D) Positively charged (D) Forest ecosystem

43. Which of the following components provides 50. Lungs are made up of air-filled sacs, the al-
sticky character to the bacterial cell ? veoli. They do not collapse even after forceful
(A) Nuclear membrane (B) Plasma membrane expiration because of :
(C) Glycocalyx (D) Cell wall (A) Inspiratory Reserve Volume
(B) Tidal Volume
44. Which of the following options best represents (C) Expiratory Reserve Volume
the enzyme composition of pancreatic juice? (D) Residual Volume
(A) amylase, pepsin, trypsinogen, maltase
(B) peptidase, amylase, pepsin, rennin 51. Presence of plants arranged into well de-
(C) lipase, amylase, trypsinogen, fined vertical layers depending on their
procarboxypeptidase height can be seen best in :
(D) amylase, peptidase, trypsinogen, rennin (A) Tropical Rain Forest (B) Grassland
(C) Temperate forest (D) Tropical Savannah
45. Which among these is the correct combina-
tion of aquatic mammals ? 52. Which of the following statements is correct?
(A) Dolphins, Seals, Trygon (A) The decending limb of loop of Henle is
(B) Whales, Dolphins, Seals impermeable to water.
(C) Trygon, Whales, Seals (B) The ascending limb of loop of Henle is
(D) Seals, Dolphins, Sharks permeable to water.

46. Fruit and leaf drop at early stages can be (C) The descending limb of loop of Henle is
prevented by the application of : permeable to electrolytes.

(A) Ethylene (B) Auxins (D) The ascending limb of loop of Henle is
(C) Gibberellic acid (D) Cytokinins impermeable to water.
PREVIOUS YEAR PAPER [NEET - 2017] Page # 33
53. Alexander Von Humbolt described for the first 61. Which statement is wrong for Krebs' cycle ?
time : (A) There is one point in the cycle where
(A) Laws of limiting factor FAD+ is reduced to FADH2
(B) Species are realationships (B) During coversion of succinyl CoA to
(C) Population Growth equation succinic acid, a molecule of GTP is synthesised
(D) Ecological Biodiversity
(C) The cycle starts with condensation of
54. Zygotic meiosis is characteristic of : acetyl group (acetyl CoA) with pyruvic acid
(A) Fucus (B) Funaria to yield citric acid
(C) Chlamydomonas (D) Marchantia (D) There are three points in the cycle where
NAD+ is reduced to NADH + H+
55. If there are 999 bases in an RNA that codes
for a protein with 333 amino acids, 62. Phosphoenol pyruvate (PEP) is the primary
and the base at position 901 is deleted such CO2 acceptor in :
that the length of the RNA becomes
(A) C4 plants (B) C2 plants
998 bases, How many codons will be altered?
(C) C3 and C4 plants (D) C3 plants
(A) 11 (B) 33
(C) 333 (D) 1 63. During DNA replication, okazaki fragments are
56. Flowers which have single ovule in the ovary used to elongate :
and are packed into inflorescence are usually (A) The lagging strand towards replication fork.
pollinated by : (B) The leading strand away from replication fork
(A) Bee (B) Wind (C) The lagging strand away from the
(C) Bat (D) Water replication fork

57. Transplantation of tissues / organs fails of- (D) The leading strand towards replication fork.
ten due to non-accepatance by the patient's
64. Which of the following RNAs should be most
body. Which type of immune-response is
abundant in animal cell ?
responsible for such rejections ?
(A) t - RNA (B) m - RNA
(A) Cell-mediated immune response
(B) Hormonal immune response (C) mi - RNA (D) r - RNA
(C) Physiological immune response
65. GnRH, a hypothalamic hormone, needed in
(D) Autoimmune response
reproduction, acts on :
58. Life cycle of Ectocarpus and Fucus respec- (A) anterior pituitary gland and stimulates
tively are : secretion of LH and FSH.
(A) Diplontic Haplodiplontic (B) posterior pituitary gland and stimulates
(B) Haplodiplontic, Diplontic secretion of oxytocin and FSH.
(C) Haplodiplontic, Haplontic (C) posterior pituitary gland and stimulates
(D) Haplontic, Diplontic secretion of LH and relaxin.

59. A gene whose expression helps to indentify (D) anterior pituitary gland and stimulates
transformed cell is known as : secretion of LH and oxytocin.
(A) Vector (B) Plasmid
66. What is the criterion for DNA fragments
(C) Structural gene (D) Selectable marker
movement on agarose gel during gel
60. A dioecious flowering plant prevents both : electrophoresis ?
(A) Autogamy and geitonogamy (A) The smaller the fragment size, the farther
(B) Geitonogamy and xenogamy it moves
(C) Cleistogamy and xenogamy (B) Positively charged fragments move to
(D) Autogamy and xenogamy farther end
Page # 34 PREVIOUS YEAR PAPER [NEET - 2017]
(C) Negatively charged fragements do not (A) Pseudomonas (B) Mycoplasma
move (C) Nostoc (D) Bacillus
(D) The larger the fragment size, the farther
74. Which of the following represent order of
it moves
'Horse' ?
67. Hypersecretion of Growth Hormone in adults (A) Perissodactyla (B) Caballus
does not cause further increase in height, (C) Ferus (D) Equidae
because:
75. Frog's heart when taken out of the body
(A) Epiphyseal plates close after
continues to beat for sometimes.
adolescence.
Select the best option from the following
(B) Bones loose their sensitivity to Growth
statements.
Hormone in adults.
(a) Frog is a poikilotherm.
(C) Muscle fibres do not grow in size after birth
(b) Frog does not have any coronary
(D) Growth Hormone becomes inactive in
circulation.
adults.
(c) Heart is " myogenic" in nature.
68. DNA replication in bacteria occurs : (d) Heart is autoexcitable
(A) Within nucleolus Options :
(B) Prior to fission (A) only (d) (B) (a) and (b)
(C) Just before transcription (C) (c) and (d) (D) Only (c)
(D)During S phase
76. Homozygous purelines in cattle can be
69. Which one from those given below is the obtained by :
period for Mendel's hybridization experiments? (A) mating of unrelated individuals of same
(A) 1840 - 1850 (B) 1857 - 1869 breed.
(C) 1870 - 1877 (D) 1856 - 1863 (B) mating of individulas of different breed.

70. Viroids differ from viruses in having : (C) mating of individuals of different species.

(A) DNA molecules without protein coat (D) mating of related individuals of same
(B) RNA molecules with protein coat breed.

(C) RNA molecules without protein coat 77. Indentify the wrong statement in context of
(D) DNA molecules with protein coat heartwood. :

71. MALT contitutes about________ percent of (A) It is hightly durable


the lymphoid tissue in human body. (B) It conducts water and minerals efficiently
(A) 20% (B) 70% (C) It comprises dead elements with highly
(C) 10% (D) 50% lignified walls
(D) Organic componds are deposited in it.
72. Which of the following is correctly matched for
the poroduct produced by them ? 78. Anaphase Promoting Complex (APC) is a
(A) Mathanobacterium : Lactic acid protein degradation machinery necessary for
(B) Penicillium notatum : Acetic acid proper mitosis of animal cells. If APC is
(C) Sacchromyces cerevisiae : Ethanol defective in a human cell, which of the
(D) Acetobacter aceti : Antibiotics following is expected to occur ?
(A) Chromosomes will be fragmented
73. Which among the following are the smallest (B) Chromosomes will not segregate
living cells, known without a definite cell wall, (C) Recombination of chromosome arms will
pathogenic to plants as well as animals and can occur
survive without oxygen ?
(D) Chromosomes will not condense
PREVIOUS YEAR PAPER [NEET - 2017] Page # 35
79. Which of the following cell organelles is 84. A baby boy aged two year is admitted to
responsible for extracting energy from play school and passes through a dental
carbohydrates to form ATP ? check - up. The dentist observed that the
(A) Ribosome (B) Chloroplast boy had twenty teeth. Which teeth were
(C) Mitochondrion (D) Lysosome absent ?
(A) Canines (B) Pre - molars
80. Mycorrhizae are the example of :
(C) Molars (D) Incisors
(A) Amensalism (B) Antibiosis
(C) Mutualism (D) Fungistasis 85. Select the mismatch -
(A) Cycas - Dioecious
81. Our of 'X' pairs of ribs in humans only ' Y'
(B) Salvinia - Heterosporous
pairs are true ribs. Select the opotion that
(C) Equisetum - Homosporous
correctly represents values of X and Y and
(D) Pinus - Dioecious
provides their explanation :
(A) X = 12, Y = 5, True ribs are attached 86. The morphological nature of the edible part
dorsally to vetebral column and sternum on of coconut is :
the two ends (A) Cotyledon (B) Endosperm
(B) X = 24, Y = 7, True ribs are dorsally (C) Pericarp (D) Perisperm
attached to vertebral column but are free
on ventral side. 87. Double fertilization is exhibited by :

(C) X = 24, Y = 12, True ribs are dorsally (A) Algae (B) Fungi
attached to vertebral column but are free (C) Angiosperms (D) Gymnosperms
on ventral side.
88. Spliceosomes are not found in cells of :
(D) X = 12, Y = 7, True ribs are attached
(A) Fungi (B) Animals
dorsally to vertebral column and ventrally to
(C) Bacteria (D) Plants
the sternum.

89. The association of histone H1 with a


82. In case of poriferans, the spongocoel is lined
nucleosome indicates :
with flagellated cells called :
(A) DNA replication is occuring.
(A) Oscula
(B) The DNA is condensed into a Chromatin
(B) Choanocytes
fibre
(C) Mesenchymal cells
(C) The DNA double helix is exposed
(D) Ostia
(D) Transcription is occurring.
83. Which one of the following statements is not
90. The region of Biosphere Reserve which is
valid for aerosols ?
legally protected and where no human
(A) They alter rainfall and monsoon patterns
activity is allowed is known as :
(B) They cause increased agricultural
(A) Buffer zone (B) Transition zone
productivity
(C) Restoration zone (D) Core zone.
(C) They have negative impact on agricultural
(D) They are harmful to human health.
Page # 36 PREVIOUS YEAR PAPER [NEET - 2017]

PHYSICS
1. Spring of force constant k is cut into lengths 3(K1  K 2 )
of ratio 1:2:3. They are connected in series (A) (B) K1 + K2
2
and the new force constant is K’. Then they are
connected in parallel and force constant is k”. K1  K2
then k’ : k” is : (C) 2(K1+K2) (D)
2
(A) 1:9 (B) 1:11
(C) 1:14 (D) 1:6
7. A capacitor is charged by a battery. The
2. The ratio of resolving powers of an optical battery is removed and another identical
microscope for two wavelengths 1 = 4000 Å uncharged capacitor is connected in parallel.
and 2 = 6000 Å is : The total electrostatic energy of resulting
(A) 9:4 (B) 3:2 system :
(C) 16:81 (D) 8:27 (A) decreases by a factor of 2
(B) remains the same
3. The two nearest harmonics of a tube closed
(C) increases by a factor of 2
at one end and open at other end are 220
Hz and 260 Hz. What is the fundamental (D) increases by a factor of 4
frequency of the system?
(A) 20 Hz (B) 30 Hz 8. In a common emitter transistor amplifier the
(C) 40 Hz (D) 10 Hz audio signal voltage across the collector is 3
V. The resistance of collector is 3 K. If
4. Consider a drop of rain water having mass current gain is 100 and the base resistance
1g falling from a height of 1 km. It hits the is 2 k, the voltage and power gain of the
ground with a speed of 50 m/s. Take ‘g’
amplifier is :
constant with a value 10 m/s2. The work
(A) 15 and 200 (B) 150 and 15000
done by the (i) gravitational force and the
(ii) resistive force of air is : (C) 20 and 2000 (D) 200 and 1000
(A) (i) 1.25 J (ii)–8.25 J
(B) (i) 100 J (ii) 8.75 J 9. Thermodynamic processes are indicated in
(C) (i) 10 J (ii) –8.75 J the following diagram.
(D) (i) –10 J (ii) –8.25 J

5. A physical quantity of the dimensions of P


length that can be formed out of c, G and
e2 i IV
is [c is velocity of light, G is universal f
40 III
I
constant of gravitation and e is charge]: f
II
1
2
1
2
f 700K
2 2
2  e  1  e  500K
(A) c G  (B) 2   f
 40  C  G 40  300K
V
1
2
1 e2 1  e2 
(C) c G 4 (D) 2 G 
c  40 
0
Match the following :
Column-1 Column-2
6. Two rods A and B of different materials are
P. Process I a. Adiabatic
welded together as shown in figure. Their thermal
conductivities are K1 and K2. The thermal Q. Process II b. Isobaric
conductivity of the composite rod will be : R. Process III c. Isochoric
S. Process IV d. Isothermal
T1 T2 (A) Pc, Qa, Rd, Sb
(B) Pc, Qd, Rb, Sa
d (C) Pd, Qb, Ra, Sc
(D) Pa, Qc, Rd, Sb
PREVIOUS YEAR PAPER [NEET - 2017] Page # 37
10. Suppose the charge of a proton and an 15. A long solenoid of diameter 0.1 m has 2×104
electron differ slightly. One of them is –e, turns per meter. At the centre of the
the other is (e+e). If the net of electrostatic solenoid, a coil of 100 turns and radius 0.01
force and gravitational force between two m is placed with its axis coinciding with the
hydrogen atoms placed at a distance d (much solenoid axis. The current in the solenoid
greater than atomic size) apart is zero, then reduces at a constant rate to 0A from 4 A in
e is of the order of [Given mass of hydrogen 0.05 s. If the resistance of the coil is 102,
mh = 1.67×10-27 kg] the total charge flowing through the coil
(A) 10-23 C (B) 10–37 C during this time is :
(C) 10-47 C (D) 10-20 C (A) 16 C (B) 32 C
(C) 16 C (D) 32 C
11. The resistance of a wire is ‘R’ ohm. If it is
melted and stretched to ’n’ times its original 16. Preeti reached the metro station and found
length, its new resistance will be : that the escalator was not working. She walked
up the stationary escalator in time t1. On other
R days, if she remains stationary on the moving
(A) (B) n2R
n escalator, then the escalator takes her up in
time t2. The time taken by her to walk up on
R the moving escalator will be :
(C) (D) nR
n2
t1t 2 t1t2
(A) t  t (B) t  t
2 1 2 1
12. The given electrical network is equivalent to :
t1  t2
A Y (C) t1 – t2 (D)
2
B
17. Young's double slit experiment is first
(A) OR gate (B) NOR gate performed in air and then in a medium other
(C) NOT gate (D) AND gate than air. It is found that 8th bright fringe in
the medium lies where 5th dark fringe lies in
air. The refractive index of the medium is
13. The de-Broglie wavelength of a neutron in
nearly.
thermal equilibrium with heavy water at a (A) 1.59 (B) 1.69
temperature T (Kelvin) and mass m, is : (C) 1.78 (D) 1.25
h 2h 18. A beam of light from a source L is incident
(A) (B)
3mkT 3mkT normally on a plane mirror fixed at a certain
distance x from the source. The beam is
2h h reflected back as a spot on a scale placed
(C) (D) just above the source L. When the mirror is
mkT mkT
rotated through a small angle , the spot of
the light is found to move through a distance
14. Which one of the following represents y on the scale. The angle  is given by :
forward bias diode ? y x
(A) (B)
R x 2y
(A) -4V -3V
x y
(C) (D)
y 2x
R
(B) -2V +2V
19. If 1 and 2 be the apparent angles of dip
observed in two vertical planes at right angles
R to each other, then the true angle of dip  is
(C) 3V 5V
given by :
(A) tan2 = tan21 + tan22
R (B) cot2 = cot21 – cot22
(D) 0V -2V (C) tan2 = tan21 – tan22
(D) cot2 = cot21 + cot22
Page # 38 PREVIOUS YEAR PAPER [NEET - 2017]
20. Two cars moving in opposite directions approach 25. A spherical black body with a radius of 12 cm
each other with speed of 22 m/s and 16.5 m/s radiates 450 watt power at 500 K. If the radius
respectively. The driver of the first car blows were halved and the temperature doubled,
a horn having a frequency 400 Hz. The the power radiated in watt would be:
frequency heard by the driver of the second (A) 450 (B) 1000
(C) 1800 (D) 225
car is [velocity of sound 340 m/s]
(A) 361 Hz (B) 411 Hz 26. Figure shows a circuit that contains three
(C) 448 Hz (D) 350 Hz identical resistors with resistance R = 9.0 
each, two identical inductors with inductance
21. Two blocks A and B of masses 3m and m L = 2.0 mH each, and an ideal battery with
respectively are connected by a massless emf  = 18 V. The current "i" through the
and inextensible string. The whole system is battery just after the switch closed is, .......
suspended by a massless spring as shown in
figure. The magnitudes of acceleration of A
and B immediately after the string is cut, are
respectively.

(A) 0.2 A (B) 2 A


(C) 0 ampere (D) 2 mA

27. Radioactive material 'A' has decay constant


'8 '. and material 'B' has decay constant ''.
Initially they have same number of nuclei.
After what time, the ratio of number of muclei
1
g of material 'B' to that 'A' will be ?
(A) ,g (B) g, g e
3
1 1
g g g (A) (B)
7 8
(C) , (D) g,
3 3 3 1 1
(C) (D)
9 
22. A thin prism having refracting angle 10° is
made glass of refractive index 1.42. This prism 28. The diagrams below show regions of
is combination with another thin prism of glass equipotentials.
of refractive index 1.7. This combination
produces dispersion without deviation. The
refracting angle of second prism should be :
(A) 6° (B) 8°
(C) 10° (D) 4°

23. The acceleration due to gravity at a height


1 km above the earth is the same as at a
depth d below the surface of earth. Then :
3
(A) d = 1 km (B) d = km
2
1
(C) d = 2 km (D) d = km
2

24. A potentiometer is an accurate and versatile


device to make electrical measurements of
E.M.F. because the method involves :
(A) potential gradients
(B) a condition of no current flow through
the galvanometer
(C) a combination of cells, galvanometer and
resistances
(D) cells
PREVIOUS YEAR PAPER [NEET - 2017] Page # 39
A positive charge is moved from A to B in 33. Two Polaroids P1 and P2 are placed with their
each diagram. axis perpendicular to each other. Unpolarized
(A) In all the four cases the work done is the light I0 is incident on P1. A third polaroid P3 is
kept in between P1 and P2 such that its axis
same.
makes an angle 45° with that of P1. The
(B) Minimum work is required to move q in
intensity of transmitted light through P2 is :
figure (a).
(C) Maximum work is required to move q in I0 I0
(A) (B)
figure (b). 4 8
(D) Maximum work is required to move q in I0 I0
figure (c). (C) (D)
16 2

29. Two astronauts are floating in gravitational 34. The bulk modulus of a spherical object is ‘B’.
free space after having lost contact with If it is subjected to uniform pressure ‘p’, the
their spaceship. The two will : fractional decrease in radius is :
(A) move towards each other B 3p
(B) move away from each other (A) (B)
3p B
(C) will become stationary
(D) keep floating at the same distance p p
(C) (D)
between them 3B B

35. In an electromagnetic wave in free space


30. The x and y coordinates of the particle at
the root mean square value of the electric
any time are x = 5t – 2t 2 and y = 10t
field is Erms = 6V/m. The peak value of the
respectively, where x and y are in meters magnetic field is :
and t in seconds. The acceleration of the (A) 2.83×10-8T (B) 0.70×10-8T
particle at t = 2s is : (C) 4.23×10-8T (D) 1.41×10-8T
(A) 5 m/s2 (B) – 4 m/s2
(C) – 8 m/s 2
(D) 0 36. A rope is wound around a hollow cylinder of
mass 3 kg and radius 40 cm. What is the
31. One end of string of length l is connected to angular acceleration of the cylinder if the
rope is pulled with a force of 30 N?
a particle of mass ‘m’ and the other end is
(A) 0.25 rad/s2 (B) 25 rad/s2
connected to a small peg on a smooth (C) 5 m/s2 (D) 25 m/s2
horizontal table. If the particle moves in circle
with speed ‘v’, the net force on the particle 37. Two discs of same moment of inertia rotating
(directed towards center) will be (T about their regular axis passing through
represents the tension in the string) centre and perpendicular to the plane of disc
with angular velocities 1 and 2 . They are
mv2 mv2 brought into contact face to face coinciding
(A) T  (B) T 
l l the axis of rotation. The expression for loss
(C) Zero (D) T of energy during this process is :
1
(A) I(1  2 )2 (B) I(1  2 )2
32. A Particle executes linear simple harmonic 4
motion with an amplitude of 3 cm. When the
1 1
particle is at 2 cm from the mean position, (C) (  2 )2 (D) I(1  2 )2
8 1 2
the magnitude of its velocity is equal to that
of its acceleration. Then its time period in 38. The photoelectric threshold wavelength of
seconds is : silver is 3250×10-10m. The velocity of the
electron ejected from a silver surface by
5 4
(A) (B) ultraviolet light of wavelength 2536×10-10m is
2 5 (Given h = 4.14×10-15 eVs and c = 3×108 ms-1)
(A) 0.6×106 ms-1 (B) 61×103 ms-1
2 5 (C) 0.3×106 ms-1 (D) 6×105 ms-1
(C) (D)
3 
Page # 40 PREVIOUS YEAR PAPER [NEET - 2017]
39. A 250 – Turn rectangular coil of length 2.1
cm and width 1.25 cm carries a current of Pa Pa
85 A and subjected to a magnetic field of
F
strength 0.85T. Work done for rotating the A
coil by 180° against the torque is : 10 mm
(A) 4.55 J (B) 2.3 J E
Final water level
(C) 1.15 J (D) 9.1 J Oil 65mm
D
Initial water level
40. The ratio of wavelengths of the last line 65mm
of Balmer series and the last line of Lyman B C
series is :
(A) 1 (B) 4 Water
(C) 0.5 (D) 2

(A) 425 kg m-3 (B) 800 kg m-3


1 (C) 928 kg m-3 (D) 650 kg m-3
41. A carnot engine having an efficiency of
10
as heat engine, is used as a refrigerator. If the
work done on the system is 10 J, the amount 45. Which of the following statements are
of energy absorbed from the reservoir at lower correct ?
temperature is : (a) Centre of mass of a body always
(A) 90 J (B) 99 J coincides with the centre of gravity of the
(C) 100 J (D) 1 J body.
(b) Centre of mass of a body is the point at
42. A gas mixture consists of 2 moles of O2 and which the total gravitational torque on the
4 moles of Ar at temperature T. Neglecting body is zero.
all vibrational modes, the total internal energy (c) A couple on a body produce both
of the system is : translational and rotational motion in a body.
(A) 15 RT (B) 9 RT (d) Mechanical advantage greater than one
(C) 11 RT (D) 4 RT means that small effort can be used to lift a
large load.
43. An arrangement of three parallel straight (A) (a) and (b) (B) (b) and (c)
wires placed perpendicular to plane of paper (C) (c) and (d) (D) (b) and (d)
carrying same current ‘I’ along the same
direction is shown in figure. Magnitude of
force per unit length on the middle wire ‘B’ is
given by:
B d C
90º

A
2
2 0i 20i2
(A) (B)
d d
0i2 0i2
(C) (D)
2d 2d

44. A U tube with both ends open to the


atmosphere, is partially filled with water. Oil,
which is immiscible with water, is poured into
one side until it stands at a distance of 10
mm above the water level on the other side.
Meanwhile the water rises by 65 mm from
its original level (see diagram). The density
of the oil is :
PREVIOUS YEAR PAPER [NEET - 2017] Page # 41

CHEMISTRY
1. Name the gas that can readily decolour is 7. The equilibrium constants of the following are:
acidified KMnO4 solution: N2 + 3 H2  2 NH3 K1
(A) SO2 (B) NO2 N2 + O2  2 NO K2
(C) P2O5 (D) CO2
1
H2 + O  H2O K3
2. Mechanism of a hypothetical reaction X2 + 2 2
Y2  2 XY is given below: The equilibrium constant (K) of the reaction:
(i) X2  X + X (fast)
K
(ii) X + Y2 XY + Y (slow) 2NH3  5  2NO  3H2O, will be :
2
(iii) X2 + Y2  XY (fast)
The overall order of the reaction will be: (A) K2K33 / K1 (B) K2K3 / K1
(A) 2 (B) 0 (C) 1.5(D) 1
(C) K32K3 / K1 (D) K1K33 / K2
3. The element Z = 114 has been discovered
recently. It will belong to which of the 8. Which one is the most acidic compound ?
following family/group and electronic
OH
configuration?
OH
(A) Carbon family, [Rn] 5f14 6d107s27p2
(B) Oxygen family, [Rn] 5f14 6d107s27p4
(C) Nitrogen family, [Rn] 5f14 6d107s27p6 (A) (B)
(D) Halogen family, [Rn] 5f14 6d107s27p5 NO2

4. The heating of phenyl-methyl ethers with HI


produces. OH OH
(A) Iodobenzene (B) Phenol O2N NO2
(C) Benzene (D) Ethyl Chlorides
(C) (D)
5. Which one is the correct ordr of acidity?
NO2 CH3
(A) CH  CH > CH3 – C  CH > CH2 = CH2 >
CH3 – CH3
(B) CH  CH > CH2  CH2 > CH3 – C  CH > CH3 9. The correct increasing order of basic strength
– CH3 for the following compound is :
(C) CH3 – CH3 > CH2 = CH2 > CH3 – C  CH >
CH  CH NH2 NH2 NH2
(D) CH2 CH2 > CH3 – CH = CH2 > CH3 – C 
CH > CH  CH

6. Predict the correct intermediate and product


in the following reaction: NO 2 CH3
(I) (II) (III)
H2O,H2SO4
H3C – C  CH 
HgSO4
 int ermediate 
 Pr oduct (A) III < I < II (B) III < II < I
(A) (B)
(C) II < I < III (D) II < III < I
(A) A : H3C – C = CH2 B : H3C – C = CH2
OH SO4 10. Ionic mobility of which of the following alkali
metal ions is lowest when aqueous solution
(B) A : H3C – C – CH3 B : H3C – C CH of their salts are put under an electric fields?
(A) K (B) Rb
O
(C) Li (D) Na
(C) A : H3C – C – CH2 B : H3C – C – CH3
11. The most suitable method of separation of 1:
OH O
1 mixture of ortho and para - nitrophenols is:
(D) A : H3C – C – CH2 B : H3C – C – CH3 (A) Chromatography (B) Crystallisation
(C) Steam distillation (D) Sublimation
SO4 O
Page # 42 PREVIOUS YEAR PAPER [NEET - 2017]
12. HgCl2 and I2 both when dissolved in water 19. Which of the following pairs of compounds is
containing I– ions the pair of species formed is: isoelectronic and isostructural ?
(A) HgI2, I– (B) HgI24 , I3 (A) TeI2,XeF2 (B) IBr2 , XeF2
(C) Hg2I2, I –
(D) HgI2, I3 (C) IF3, XeF2 (D) BeCl2, XeF2

13. Mixture of chloroxylenol and terpineol acts 20. The IUPAC name of the compound
O
as : O
(A) antiseptic (B) antipyretic C
(C) antibiotic (D) analgesic H is

(A) 5-formylhex-2-en-3-one
14. An example of a sigma bonded organometallic
(B) 5-methyl-4-xoohex-2-en-5-al
compound is :
(C) 3-keto-2-methylhex-5-enal
(A) Grignard' reagent (B) Ferrocene
(C) Cobaltocene (D) Ruthenocene (D) 3-keto-2-methylhex-4-enal

15. A first order reaction has a specific reaction 21. Which one is the wrong statement ?
rate of 10–2sec–1. How much time will it take (A) The uncertainty principle is E × t  h/4
for 20 g of the reactant of reduce to 5 g ? (B) Half filled and fully filled orbitals have
(A) 138.6 sec (B) 346.5 sec greater stability due to greater exchange
(C) 693.0 sec (D) 238.6 sec energy, greater symmetry and more balanced
arrangement
16. Match the interhalogen compound of column (C) The energy of 2s orbital is less than the
I with the geometry in column II and assign energy of 2p orbital in case of hydfrogen like
the correct code. atoms.
Column I Column II (D) de-Brogle's wavelength is given by =
(a) XX' (i) T-shape h
, where m = mass of the particle,  =
(b) XX'3 (ii)Pentagonal bipyramial m
'
group velocity of the particle.
(c) XX 5
(iii) Linear
(D) XX'7 (iv) Square-pyramidal 22. Which is the incorrect statement ?
(v) Tetrahedral (A) Density decreases in case of crystals
Codes : with Schottky's defect.
(a) (b) (c) (D) (B) NaCl(s) insulator,silicon is semiconductor,
(A) (iii) (i) (iv) (ii) silver is conductor, quartz is piezo electric crystal.
(B) (v) (iv) (iii) (ii) (C) Frenkel defect is favoured in those ionic
(C) (iv) (iii) (ii) (i) compounds in which sizes of cation and anions
(D) (iii) (iv) (i) (ii) are almost equal.
(D) FeO0.98 has non stoichiometric metal
17. Concentration of the Ag+ ions in a saturated
solution of Ag2C2O4 is 2.2 × 10–4 mol L–1 . 23. The species, having bond angles of 120° is
Solubility product of Ag2C2O4 is : (A) ClF3 (B) NCl3
(A) 2.66 × 10–12 (B) 4.5 × 10–11
–12
(C) BCl3 (D) PH3
(C) 5.3 × 10 (D) 2.42 × 10–8
24. For a given reaction, H = 35.5 kJ mol–1 and
18. In the electrochemical cell :
S = 83.6 kJ –1 mol –1 . The reaction is
Zn|ZnSO4(0.01 M)||CuSO4 (1.0 M)|Cu, the
spontaneous at : (Assume that H and S
emf of this daniel cell is E 1 . When the
do not vary with temperature)
concentration of ZnSO4 is changed to 1.0 M
(A) T > 425 K (B) all temperatures
and that of CuSO4 changed to 0.01 M, the
(C) T > 298 K (D) T < 425 K
emf changes to E2.From the followings, which
one is the relationship between E1 and E2 ?
25. Which of the following is a sink for CO ?
RT
(Given, , 0.059) (A) Micro organisms present in the soil
F (B) Oceans
(A)E1 < E2 (B) E1 > E2
(C) Plants (D) Haemoglobin
(C) E2 = 0  E1 (D) E1 = E2
PREVIOUS YEAR PAPER [NEET - 2017] Page # 43
26. If molality of the dilute solutionis doubled, 31. The correct statement regarding electrophile is:
the value of molal depression constant (Kf) (A) Electrophile is a negatively charged
will be : species and can form a bond by accepting a
(A) halved (B) tripled pair of electrons from another electrohpile
(C) unchanged (D) doubled (B) Electrophiles are generally neutral species
and can form a bond by accepting a pair of
27. Which of the following is dependent on
electrons from a nucleophile
temperature ?
(A) Molarity (B) Mole fraction (C) Electrophile can be either neutral or
(C) Weight percentage (D) Molality positively charged species and can form a
bond by accepting a pair of electrons from a
28. Which one of the following statements is not nucleophile
correct ? (D) Electrophile is a negatively charged
(A) The value of equilibrium constant is species and can form a bond by accepting a
changed in the presence of a catalyst in the pair of electrons from a nucleophile
reaction at equilibrium.
(B) Enzymes catalyse mainly bio-chemical
32. A gas is allowed to expand in a well insulated
reactions
container against a constant external
(C) Coenzymes increase the catlytic acitivity
of enzyme. pressure of 2.5 atm from an initial volume of
(D) Catalyst does not initiate any reaction. 2.50 L to a final volume of 4.50 L. The change
in internal energy U of the gas in joules will be:
29. Identify A and predict the type of reaction (A) –500 J (B) –505 J
(C) +505 J (D) 1136.25 J
OCH3
NaNH 2
A 33. Which of the following reaction is appropriate
for converting acetamide to methanamine ?
Br (A) Hoffmann hypobromamide reaction
(B) Stephens reaction
OCH3
(C) Gabriels phthalimide snthesis
NH 2 (D) Carbylamine reaction
(A) and elimination addition

34. With respect to the conformers of ethane,


reaction which of the following statements is true ?
OCH3 (A) Bond angle changes but bond length
Br remains same
(B) and cine substitution reaction (B) Both bond angle and bond length change
(C) Both bond angles and bond length remains
OCH3 same
(D) Bond angle remains same but bond length
(C) and cine substitution reaction changes

35. In which pair of ions both the species contain


OCH3
S–S bond ?

(D) and substitution reaction (A) S2O62 , S2O32 (B) S2O72 , S2O82
NH2 (C) S4O62 , S2O72 (D) S2O72 , S2O32

30. The correct order of the stoichiometries of 36. It is because of inability of ns2 electrons of
AgCl formed when AgNO3 in excess is treated the valence shell to participate in bonding
with the complexes : COCl3.6 NH3, CoCl3 . that :
5NH3, CoCl3. 4 NH3 respectively is : (A) Sn2+ is oxidising while Pb4+ is reducing
(A) 3 AgCl, 1 AgCl, 2 AgCl (B) Sn2+ and Pb2+ are both oxidising and
(B) 3 AgCl,2 AgCl, 1 AgCl reducing
(C) 2 AgCl, 3AgCl, 1 AgCl (C) Sn4+ is reducing while Pb4+ is oxidising
(D) 1 AgCl, 3 AgCl, 2 AgCl (D) Sn2+ is reducing while Pb4+ is oxidising
Page # 44 PREVIOUS YEAR PAPER [NEET - 2017]

37. Correct increasing order for the wavelengths 41. Extraction of gold and silver involves leaching
of absorption in the visible region for the with CN– ion. Silver is later recovered by:
complexes of Co3+ is: (A) distillation (B) zone refining
(A) [Co(H2O)6]3+, [Co(en)3]3+, [Co(NH3)6]3+ (C) displacement with Zn (D) liquation
(B) [Co(H2O)6]3+, [Co(NH3)6]3+, [Co(en)3]3+
(C) [Co(NH3)6]3+, [Co(en)3]3+, [Co(H2O)6]3+
42. A 20 litre container at 400K contains CO2(g)
(D) [Co(en)3]3+, [Co(NH3)6]3+, [Co(H2O)6]3+
at pressure 0.4 atm and an excess of SrO
(neglect the volume of solid SrO). The volume
38. Consider the reactions:
of the container is now decreased by moving
[Ag(NH3)2]+
X Cu A – Silver mirror observed the movable piston fitted in the container.
573K OH
(C2H 6O) –
OH The maximum volume of the container, when
O Y pressure of CO2 attains its maximum value,
Z
NH2 – NH – C – NH2 will be:
Identify A, X, Y and Z (Given that: SrCO3(s) SrO(s) + CO2(g),
(A) A-Methoxymethane, X-Ethanol, Y- Kp = 1.6 atm)
Ethanoic acid, Z-Semicarbazide (A) 10 litre (B) 4 litre
(B) A-Ethanal, X-Ethanol, Y-But-2-enal, Z- (C) 2 litre (D) 5 litre
Semicarbazone
(C) A-Ethanol, X-Acetaldehyde, Y-Butanone,
Z-Hydrazone 43. Pick out the correct statement with respect
(D) A-Methoxymethane, X-Ethanoic acid, Y- to [Mn(CN)6]3– :
Acetate ion, Z-hydxrazine (A) It is sp3d2 hybridised and tetrahedral
(B) It is d2sp3 hybridised and octahedral
39. Of the following, which is the product formed
when cyclohexanone undergoes aldol (C) It is dsp2 hybridised and square planar
condensation followed by heating? (D) It is sp3d2 hybridised and octahedral

44. The reason for greater range of oxidation


(A)
states in actinoids is attributed to:
O
(A) actinoid contraction (B) 5f, 6d and
7s levels having comparable energies
(B) (C) 4f and 5d levels being close in energies
OH (D) the radioactive nature of actinoids

45. Which of the following statements is not


(C)
correct?
O O
(A) Ovalbumin is a simple food reserve in egg-
O
white
(B) Blood proteins thrombin and fibrinogen
(D) are involved in blood clotting
(C) Denaturation makes the proteins more
OH
active
40. Which one of the following pairs of species (D) Insulin maintains sugar level in the blood
have the same bond order? of a human body
(A) O2, NO+ (B) CN–, CO
(C) N2, O2 (D) CO, NO
PREVIOUS YEAR PAPER [NEET - 2017] Page # 45

ANSWERKEY
BIOLOGY
1. A 2. A 3. A 4. C 5. B 6. D 7. A

8. C 9. D 10. D 11. C 12. D 13. A 14. A

15. A 16. A 17. C 18. C 19. B 20. B 21. B

22. B 23. B 24. D 25. D 26. D 27. D 28. A

29. A 30. B 31. B 32. A 33. D 34. D 35. B


36. B 37. A 38. A 39. A 40. B 41. B 42. A

43. C 44. C 45. B 46. B 47. C 48. B 49. D

50. D 51. A 52. D 53. B 54. C 55. B 56. B


57. A 58. B 59. D 60. A 61. C 62. A 63. C

64. D 65. A 66. A 67. A 68. B 69. D 70. C

71. D 72. C 73. B 74. A 75. C 76. D 77. B

78. B 79. C 80. C 81. D 82. B 83. B 84. B

85. D 86. B 87. C 88. C 89. B 90. D

PHYSICS

1 B 2 B 3 A 4 C 5 D 6 D 7 A
8 B 9 A 10 B 11 B 12 B 13 A 14 D
15 B 16 B 17 C 18 D 19 D 20 C 21 A

22 A 23 C 24 B 25 C 26 Bonus 27 A 28 A
29 A 30 B 31 D 32 B 33 B 34 C 35 A
36 B 37 A 38 A & D 39 D 40 B 41 A 42 C
43 C 44 C 45 D

CHEMISTRY

1. A 2. C 3. A 4. B 5. A 6. C 7. A

8. C 9. C 10. C 11. C 12. B 13. A 14. A

15. A 16. A 17. C 18. B 19. B 20. D 21. C

22. C 23. C 24. A 25. A 26. C 27. A 28. A

29. D 30. B 31. C 32. B 33. A 34. C 35. A

36. D 37. D 38. B 39. A 40. B 41. C 42. D

43. B 44. B 45. C


Page # 46 PREVIOUS YEAR PAPER [NEET - 2017]

SOLUTION
PHYSICS
1. B y=z x + 6y = 1
1K1 = const. z = 1/2
x + 3y + 3z = 1 –x – 4y = 0
1 1 1 1
In series K  K  K  K  6 x = –2
s 1 2 3
–X – 2y – 2z = 0 2y = 1
1 1 y = 1/2
Kparallel = K1 + k2 + k3 = 1  
2 3 1/2
1  2 
1/2 e
2 G

6  3  2 11 C  4 
=   0
6 6

Ks 1 6 6. D
Kp = 6  11 = 1 : 11

K1 A
2. B T1 T2
K2 B
R.P  1 d

R1 : R2 = 2 : 1 = 6 : 4 = 3 : 2
1 1 1
= R + R
3. A R 1 2

V k(2) K  K2
= 40 = 1
2 d d
V
= 20 Hz K1  K 2
4 K=
2

4. C
7. A
1
wg + wR = mv2 q2
2 E=
wg = mgh 2C
1 q  same
= × 10 × 1000 C’ = 2C
1000
= 10 J E’ = E/2

1 1
10 + wR = × × 50 × 50 8. B
2 1000
V0 = 3 volt R0 = 3 × 103
10 + wR = 1.25
wR = –8.75  = 100 Ri = 2 × 103

R0 3
5. D AV =  R = 100 × = 150
G = M–1 L3 T–2 i 2
C = L1 T–1
R0 3
e2 Ap =  2 R = 100 × 100 ×
2
= 15000
= M1 L3 T–2 i
4πε 0

 e2 
2 9. A
  Cx Gy  
 I  Volume constant  Isochoric P  C
 4 0  II adiabatic
  Lx T–x (M–1 L3 T–2)y (M1 L3 T–2)z II  Temp Constant  Isothermal R  D
M0 L1 T0 = M–y+z Lx+3y+3z T–x–2y–2z IV  Pre constant  Adiabatic S  B
PREVIOUS YEAR PAPER [NEET - 2017] Page # 47
10. B 16. B
–e e + e
s
e t1 = v
p
K(e)2 a  (1.67  1027 )2
2

d d2 s
t2 = v
9 2
9 × 10 (e) = 6.627 × 10 –11
× 1.67 (10–27 2
) e

e = 10–37
s
t= v v
11. B p e

l s
R= A  A' n
A
t= s  s
t1 t2
.nl
R’ = A’ = A/n
A /n
t1t2
= n2R t= t t
1 2

12. B
17. C
A A+B A+B
B (A + B) = NOR
9D 8' D

2d 8d
13. A
9
h  '
= 16
p

18. D
h
=
2mE y
tan 2 
x
y
h
y
= 3  x
2m KT 2x
2

19. D
h
= At angle  from magnetic meridian
3mKT
tan 
tan 1  .....(1)
cos 
14. D
O is higther then –2V For other plane

tan  tan 
15. B tan 2   ....... (2)
cos(90  ) sin 
d NA(B2  B1 )
q 
R R tan  tan 
cos   sin  
tan 1 tan 2
NA 0n(i1  i2 )
=
R cos2   sin2   1

100    10 4  4  107  2  104(4)  1 1 


= tan2    1
102  tan2  tan 2 
2
 1
= 32 × 10–6
cot 2 1  cot 2 2  cot2 
Page # 48 PREVIOUS YEAR PAPER [NEET - 2017]
20. C 27. A
B N1 e 8t 1
A   t  e 7t 
N2 e e
22 0
S 16.5 7t = 1

 V  V0  1
t=
n’ = n  V  V  7
 s 

 340  16 .5  28. A
= 400  
w = qV
 340  22 
V  same
 356 .5 
= 400   = 448
 318  29. A

21. A 30. B
because after acceleration of B is g & A is x = 5t – 2t2 y = 10 t
less than g vx = 5 – 4t vy = 10
ax = –4 ay = 0
22. A
10 (1.42 – 1) = (1.7 – 1) A’
A’ = 6 31. D

mv2
23. C T=

gh = gd

 2h   d
g  1  R  = 1  R  g 32. B
   
a = 3 × 10–2
d = 2h y = 2 × 10–2
d = 2km
acceleration = –2x

24. B V =  a2  y2 = 2y
zero deflection mean no current

25. C a2  y2
4
=
Power  area power  Temp y
16
P2 = P1 = 4P1 = 1800 watt.
4 94 5
= =
2 2
26. Bonus
Just after switch is closed
2 2 4
T= = =
 5 5
L
R R
33. B
E
R L C
P1 P3 P2
I0 I
C = short circuit cos2 45  0
I0 I0/2 4 8
L = open circuit
current in R1 = 0 I0 I
cos2 45  0
18 2 4
 i = 4 amp
4 .5
PREVIOUS YEAR PAPER [NEET - 2017] Page # 49
34. C 38. 1&4
P V P 12400 12400
B=  = eV E= eV
V / V V B 3250 2536
 = 3.81 eV E = 4.89 eV
3dR P 4 1
 V R3 mv2  E  φ = 4.89 – 3.81 = 1eV
R B 3 2
dR P 4 1
 dV  3R 2dR mv2  1.6  10 19
R 3B 3 2

dV 3R 2dR 2  1.6  10 19


 v= = 0.59 × 106
V R3 9  10 31
dV 3dR = 0.6 × 106  6 × 105

V R
39. D
n = 250, l = 2.1 cm w = 1.25 cm
35. A i = 85 A b = 0.85 T
Esum = 6 w = MB (cos 1 – cos 2)
= 2MB
E0
C = 2 × 85 × 10–6 × 250 × 2.1 × 1.25 × 10–4
B0 × 0.85
= 9.1 × 10–6
E0 E 2 6
B0 =  2 sum = = 2 2  108
C C 3  108 40. B
= 2.82 × 10–8 For balmer - n1 = , n2 = 2 = 1
For lyman - n1 = , n2 = 1 = 2
36. B 1
 = F.R = I 2 = 4

FR 30  40  102
= = 41. A
I 3  40  40  10 4
1
10 1000 Efficiency of engine =
= × 102 = 10
40 40
Q1
= 25 =
10

37. A
I1 + I2 = 2I 0 100
w=10
  2 T1
0 = 1
2

1 Q2
KI = I (12 + 22)
2 T2
I
KF = (1 + 2)2 Q2 = 90 Jule
4
42. C
 12 22 12 22 212 
loss K = I  2  2  4  4  4  n1F1 nF
  Total internal energy = U = RT + 2 2
2 2
RT
 12 22 212 
=I  4  4  4  =
RT
[2 × 5 + 4 × 3]
  2

I 22
= (1 – 2)2 = RT = 11 RT
4 2
Page # 50 PREVIOUS YEAR PAPER [NEET - 2017]
43. C 44. C

B F2 P0
C 10 mm

F1 130 130

Fnet = F12  F22


F1 = F2 P0 + g (140) = P0 + w g (130)
= 2 F w (130) kg
P= = 928 3
2 140 m
0i
= 2
2d
45. D
0i 2 b,d are correct
2d
PREVIOUS YEAR PAPER [NEET - 2017] Page # 51

CHEMISTRY
1. A
8. C
KMnO4 + SO2  SO3 + Mn+2 (colourless)
Picric Acid
2. C
–M/–I  Acidic Streangth
2
x  9. (C)
r = K[x] [y2], Kc =
x2 
1/2 10. C
r = k1 x2  y2  [x] = k1c / 2 [x2]1/2

11. C
3. A
12. B
4. (B)

H I 13. A
O CH3 Dettol in a mixture of chloroxylenol and
terpinol
H– I 14. A

15. A
OH + CH3 I 2.3 a
t= log
K ax
Phenol
2.3 20 2.3  0.6
= log =
5. (A) 10 2 5 102
CH  CH > CH3 – C  CH > CH2 = CH2 > CH3 – = 138 sec
CH3
%S  EN 16. A
17. C
6. (C) Ag2C2O4  2Ag+ + C2O4–2
CH 3 –C  1 0 0
1- s 2s s
Oxymercuration CH3 – C = CH2 2s = 2.2 × 10–4
CH
s = 1.1 × 10–4
OH Ksp = (2s)2 + (s)
= (2.2 × 10–4)2 (1.1 × 10–4)
CH 3 – C – CH 3 = (2.2 × 2.2 × 1.1) × 10–12
= 5.324 × 10–12
O

7. (A) 18. B

Cu2 
0.06
1 Ecell = E°cell + log  2 
2NH3  N2 + 3H2 k 2 Zn 
1
Ecell = E°cell + 0.03 log 100 = E°cell + 0.06
3 Ecell2 = E°cell – 0.06
3H2 + O  3H2O K33
2 2 E1 > E2
N2 + O2  2NO K2
5 19. B
2NH3 + O  2NO + 3H2O
2 2
20. D
K 2K32
k= 21. C
K1
for H atom
Energy 2s = 2p
Page # 52 PREVIOUS YEAR PAPER [NEET - 2017]
22. C 35. A
Frenkel defeel
(fisher differance between size of (+) & (–)) 36. D

23. C 37. D

24. A 38. B
H
T> (y)
S CH3—CH=CH—CHO
35.5  1000
T>
83.6 Aldol
T > 424.6 K (A)
T > 425 K
Cu/300°C
25. A CH3—CH2—OH CH3—CHO
X
[Ag(NH3)2]
26. C
3K
MRTb2 MRTf2 / 57 CH3—COOH
Kb =
1000 Hv
, Kf =
1000 Hf Cu
O
(Kb = Unchanged) Kf = unchanged
NH 2 — NH—C—NH2
27. A O
Molarity
CH 3—CH=NH—N—C—NH 2
28. A
39. A
Value or Eqb. contant is not changed by
catalyst.
40. B
29. D

OCH3 41. C

42. D
subtitutation reaction
SrCO3  SrO + Co2
NH 3
(s) (s) (g)
Kp = PCO2
30. B
CoCl3. 6NH3 + AgNO3  3AgCl + [Co(NH2)6]Cl3 1.6 atm = PCO2
CoCl3. 5NH3 + AgNO3  2 AgCl + [Co(NH3)5Cl]
P1 V1 P2 V2 0.4  20 1.6  V2
Cl2 =
T1 T2 = 400 = 400
CoCl3.4NH3 + AgNO3
[Co(NH3)4Cl2]Cl  AgCl 0.4  20
V2 = =5
1.64
31. C
V2 = 5L
32. B
43. B
E = q + w
q =0 w = –pex + V 44. B
E = –505 J
= –5 × 101 45. C
= –505 J
33. A

34. C
PREVIOUS YEAR PAPER [NEET - 2016] Page # 53

PREVIOUS YEAR 2016 PAPER_PHASE-1


BIOLOGY
1. Gause's principle of competitive exclusion 7. Emerson's enhancement effect and Red drop
states that: have been instrumental in the discovery of:
(A) More abundant species will exclude the (A) Photophosphorylation and non-cyclic
less abundant species through competition.
electron transport
(B) Competition for the same resources ex-
(B) Two photosystems operating simulta-
cludes species having different food prefer-
neously
ences.
(C) Photophosphorylation and cyclic electron
(C) No two species can occupy the same niche
indefinitely for the same limiting resources. transport

(D) Larger organisms exclude smaller ones (D) Oxidative phosphorylation


through competition.
8. In which of the following, all three are
2. The two polypeptides of human insulin are macronutrients?
linked together by:
(A) Boron, zinc, manganese
(A) Hydrogen bonds (B) Phosphodiester bond (B) Iron, copper, molybdenum
(C) Covalent bond (D) Disulphide bridges
(C) Molybdenum, magnesium, manganese
(D) Nitrogen, nickel, phosphorus
3. The coconut water from tender coconut
represents:
9. Name the chronic respiratory disorder caused
(A) Endocarp (B) Fleshy mesocarp
mainly by cigarette smoking;
(C) Free nuclear proembryo
(D) Free nuclear endosperm (A) Emphysema (B) Asthma
(C) Respiratory acidosis
4. Which of the following statements is wrong (D) Respiratory alkalosis
for viroids?
(A) They lack a protein coat 10. A system of rotating crops with legume or
(B) They are smaller than viruses grass pasture to improve soil structure and
(C) They cause infections fertility is called:
(D) Their RNA is of high molecular weight
(A) Ley farming (B) Contour farming

5. Which of the following features is not present (C) Strip farming (D) Shifting agriculture
in the Phylum-Arthropoda?
11. Mitochondria and chloroplast are:
(A) Chitinous exoskeleton
(B) Metameric segmentation (a) semi-autonomous organelles

(C) Parapodia (b) formed by division of pre -existing organelle


(D) Jointed appendages and they contain DNA but lack protein
synthesizing machinery.
6. Which of the following most appropriately Which one of the following options is correct?
describes haemophilia?
(A) Both (a) and (b) are correct
(A) Recessive gene disorder
(B) (b) is true but (a) is false
(B) X-linked recessive gene disorder
(C) (a) is true but (b) is false
(C) Chromosomal disorder
(D) Both (a) and (b) are false
(D) Dominant gene disorder
Page # 54 PREVIOUS YEAR PAPER [NEET - 2016]
12. ln context of Amniocentesis, which of the 19. Microtubules are the constituents of:
following statement is in incorrect ? (A) Cilia, Flagella and Peroxisomes
(A) It is usually done when a woman is (B) Spindle fibres, Centrioles and Cilia
between 14-16 weeks pregnant. (C) Centrioles, Spindle fibres and Chromatin
(B) It is used for prenatal sex determination. (D) Centrosome, Nucleosome and Centrioles
(C) It can be used for detection of Down
syndrome 20. A complex of ribosomes attached to a single
(D) It can be used for detection of Cleft palate. strand of RNA is known as :
(A) Polysome (B) Polymer
13. In a chloroplast the highest number of (C) Polypeptide (D) Okazaki fragment
protons are found in:
21. Fertilization in humans is practically feasible
(A) Stroma
only if:
(B) Lumen of the thylakoids
(A) the sperms are transported into vagina
(C) Inter membrane space
just after the release of ovum in fallopian tube
(D) Antennae complex
(B) the ovum and sperms are transported
14. Photosensitive compound in human eye is simultaneously to ampullary - isthmic junction
made up of: of the fallopian tube

(A) Guanosine and Retinol (C) the ovum and sperms are transported
simultaneously to ampullary - isthmic junction
(B) Opsin and Retinal
of the cervix.
(C) Opsin and Retinol
(D) the sperms are transported into cervix
(D) Transducin and Retinene
within 48 hrs of release of ovum in uterus.

15. Spindle fibres attach on to: 22. Asthma may be attributed to:
(A) Telomere of the chromosome (A) bacterial infection of the lungs
(B) Kinetochore of the chromosome
(B) allergic reaction of the mast cells in the lungs
(C) Centromere of the chromosome
(C) inflammation of the trachea
(D) Kinetosome of the chromosome
(D) accumulation of fluid in the lungs

16. Which is the National Aquatic Animal of India? 23. The Avena curvature is used for bioassay of:
(A) Gangetic shark (B) River dolphin (A) ABA (B) GA3
(C) Blue whale (D) Sea-horse (C) IAA (D) Ethylene

17. Which of the following is required as 24. The standard petal of a papilionaceous
inducer(s) for the expression of Lac operon? corolla is also called :
(A) glucose (B) galactose (A) Carina (B) Pappus
(C) lactose (C) Vexillum (D) Corona
(D) lactose and galactose
25. Tricarpellary, syncarpous gynoecium is found
in flowers of :
18. Which of the following pairs of hormones are
(A) Liliaceae (B) Solanaceae
not antagonistic (having opposite effects) to
each other? (C) Fabaceae (D) Poaceae

(A) Parathormone - Calcitonin 26. One of the major components of cell wall of
(B) Insulin - Glucagon most fungi is:
(C) Aldosterone - Atrial Natriuretic Factor (A) Chitin (B) Peptidoglycan
(D) Relaxin - Inhibin (C) Cellulose (D) Hemicellulose
PREVIOUS YEAR PAPER [NEET - 2016] Page # 55
27. Select the incorrect statement: 33. Which of the following is not a feature of the
(A) FSH stimulates the sertoli cells which help plasmids ?
in spermiogenesis. (A) Independent replication
(B) LH triggers ovulation in ovary (B) Circular structure
(C) Transferable
(C) LH and FSH decrease gradually during
(D) Single-stranded
the follicular phase.
(D) LH triggers secretion of androgens from 34. Which of the following features is not present
the Leydig cells. in Periplaneta americana?
(A) Schizocoelom as body cavity
28. In meiosis crossing over is initiated at -
(B) Indeterminate and radial cleavage during
(A) Pachytene (B) Leptotene
embryonic development
(C) Zygotene (D) Diplotene
(C) Exoskeleton composed of N-acetylglucosamine
29. A tall true breeding garden pea plant is (D) Metamerically segmented body
crossed with a dwarf true breeding garden
pea plant. When the F1 plants were selfed 35. In higher vertebrates, the immune system
can distinguish self-cells and non-self. If this
the resulting genotypes were in the ratio of:
property is lost due to genetic abnormality
(A) 1 : 2 : 1 :: Tall homozygous : Tall
and it attacks self-cells, then it leads to:
heterozygous : Dwarf
(A) Allergic response (B) Graft rejection
(B) 1 : 2 : 1 :: Tall heterozygous : Tall
(C) Auto-immune disease
homozygous : Dwarf
(D) Active inmunity
(C) 3 : 1 :: Tall : Dwarf
(D) 3 : 1 ::Dwarf : Tall 36. Match the terms in Column-I with their de-
scription in Column-II and choose the correct
30. Which of the following is the most important option:
cause of animals and plants being driven to Column I ColumnII
extinction ? (a) Dominance (i) Many genes govern
(A) Over-exploitation a single character
(B) Alien species invasion (b) Codominance (ii) In a heterozygous
(C) Habitat loss and fragmentation organism only one
(D) Co - extinctions allele expresses
itself
31. Which one of the following is a characteris-
(c) Pleiotropy (iii) In a heterozygous
tic feature of cropland ecosystem ?
organism both
(A) Absence of soil organisms
alleles express
(B) Least genetic diversity
themselves fully
(C) Absence of weeds
(d) Polygenic (iv) A single gene
(D) Ecological succession
inheritance influences many
32. Changes in GnRH pulse frequency in females characters
is controlled by circulating levels of: Code: (a) (b) (c) (d)
(A) estrogen and progesterone (A) (ii) (i) (iv) (iii)
(B) estrogen and inhibin (B) (ii) (iii) (iv) (i)
(C) progesterone only (C) (iv) (i) (ii) (iii)
(D) progesterone and inhibin (D) (iv) (iii) (i) (ii)
Page # 56 PREVIOUS YEAR PAPER [NEET - 2016]
37. Joint Forest Management Concept was in- 43. Identify the correct statement on'inhibin' :
troduced in India during: (A) Inhibits the secretion of LH, FSH and
(A) 1960s (B) 1970s Prolactin.
(C) 1980s (D) 1990s
(B) Is produced by granulose cells in ovary

38. Pick out the correct statements: and inhibits the secretion of FSH.

(a) Haemophilia is a sex-linked recessive disease (C) Is produced by granulose cells in ovary
(b) Down's syndrome is due to aneuploidy. and inhibits the secretion of LH

(c) Phenylketonuria is an autosomal recessive (D) Is produced by nurse cells in testes and
gene disorder inhibits the secretion of LH.
(d) Sickle cell anaemia is an X- linked recessive
gene disorder. 44. Which part of the tobacco plant is infected

(A) (a) and (d) are correct by Meloidogyne incognita?


(B) (b) and (d) are correct (A) Flower (B) Leaf
(C) (a), (c) and (d) are correct (C) Stem (D) Root
(D) (a),(b) and (c) are correct
45. Antivenom injection contains preformed an-
39. Which one of the following statements is wrong tibodies while polio drops that are adminis-
(A) Cyanobacteria are also called blue-green tered into the body contain:
algae (A) Activated pathogens
(B) Golden algae are also called desmids. (B) Harvested antibodies
(C) Eubacteria are also called false bacteria. (C) Gamma globulin
(D) Phycomycetes are also called algal fungi. (D) Attenuated pathogens

40. Proximal end of the filament of stamen is at- 46. Which one of the following cell organelles is
tached to the: enclosed by a single membrane ?
(A) Anther (B) Connective
(A) Mitochondria (B) Chloroplasts
(C) Placenta (D) Thalamus or petal
(C) Lysosomes (D) Nuclei

41. Which of the following approaches does not


47. Lake of relaxation between successive stimuli
the defined action of contraceptive ?
in sustained muscle contraction is known as:
(A) Barrier methods prevent fertilization
(A) Spasm (B) Fatigue
(B) Intra uterine devices increase phagocytosis
or sperms, suppress sperm motility and (C) Tetanus (D) Tonus
fertilizing capacity of sperms
48. Which of the following is not a stem modification?
(C) Hormonal contraceptives Prevent/retard
(A) Pitcher of Nepenthes
entry of sperms, prevent ovulation and
fertilization (B) Thorns of citrus

(D) Vasectomy prevents spermatogenesis (C) Tendrils of cucumber


(D) Flattened structures of Opuntia
42. The taq polymerase enzyme is obtained from:
(A) Thermus aquaticus 49. Water souluble pigments found in plant cell
(B) Thiobacillus ferroxidans vacuoles are :
(C) Bacillus subtilis (A) Xanthophylls (B) Chlorophylls
(D) Pseudomonas putida (C) Carotenoids (D) Anthocyanins
PREVIOUS YEAR PAPER [NEET - 2016] Page # 57
50. Select the correct statement : (A) Cartilaginous Chondrichthyes
(A) Gymnosperm are both homosporous and endoskeleton
heterosporous (B) Viviparous Mammalia
(B) Salvinia, Ginkgo and Pinus all are gymno- (C) Possess a mouth Chordata
sperms with an upper and a
(C) Sequoia in one of the tallest trees lower jaw
(D) The leaves of gymnosperms are not well (D) 3 - chambered Reptilia
adapted to extremes of climate heart with one
incompletely divided
51. Which of the following is not required for any
ventricle
of the techniques of DNA fingerprinting avail-
able at present ? 56. Which of the following statements is not true
(A) Polymerase chain reaction for cancer cells in relation to mutations?
(B) Zinc finger analysis (A) Mutations inproto-oncogenes accelerate
(C) Restriction enzymes the cell cycle.
(D) DNA-DNA hybridization (B) Mutations destroy telomerase inhibitor.
(C) Mutations inactivate the cell control.
52. Which type of tissue correctly matches with (D) Mutations inhibit production of
its location? telomerase.
Tissue Location
57. The amino acid Tryptophan is the precursor
(A) Smooth muscle Wall of intestine
for the synthesis of:
(B) Areolar tissue Tendons
(A) Melatonin and Serotonin
(C) Transitional Tip of nose
(B) Thyroxine and Triiodothyronine
epithelium
(C) Estrogen and progesterone
(D) Cuboidal Lining of stomach (D) Cortisol and Cortisone
epithelium
58. Following are the two statements regarding
53. A plant in your garden avoids photorespiratory the origin of life:
losses, has improved water use efficiency, (a) The earliest organisms that appeared on
shows high rates of photosynthesis at high the earth were non-green and presumably
temperatures and has improved efficiency of anaerobes,
nitrogen utilisation. In which of the following (b) The first autotrophic organisms were the
physiological groups would you assign this chemoautotrophs that never released oxygen.
plant? Of the above statements which one of the
(A) C3 (B) C4 following options is correct?
(C) CAM (D) Nitrogen fixer (A) (a) is correct but (b) is false.
(C) (b) is correct but (a) is false
54. Which of the following structures is
homologus to the wing of a bird ? (C) Both (a) and (b) are correct
(D) Both (a) and (b) are false.
(A) Dorsal fin of a Shark
(B) Wing of a Moth
59. Reduction in pH of bood will:
(C) Hind limb of Rabbit
(A) reduce the rate of heart beat
(D) Flipper of Whale
(B) reduce the blood supply to the brain
55. Which of the following characteristic features (C) decrease the affinity of hemoglobin with
always holds true for the corresponding group oxygen
of animals? (D) release bicarbonate ions by the liver
Page # 58 PREVIOUS YEAR PAPER [NEET - 2016]
60. Analogous structures are a result of: 67. Which of the following would appear as the
(A) Divergent evolution pioneer organisms on bare rocks ?
(B) Convergent evolution (A) Lichens (B) Liverworts
(C) Shared ancestry (C) Mosses (D) Green algae
(D) Stabilizing selection
68. Which one of the following is the starter codon?

61. Which of the following is a restriction endo- (A) AUG (B) UGA
nuclease ? (C) UAA (D) UAG
(A) Hind II (B) Protease
69. Whieh one of the following characteristics is
(C) DNaseI (D) RNase
not shared by birds and mammals?

62. The term ecosystem was coined by: (A) Ossified endoskeleton
(A) E.P.Odum (B) A.G.Tansley (B) Breathing using lungs
(C) E.Haeckel (D) E.Warming (C) Viviparity
(D) Warm blooded nature
63. Which one of the following statements is wrong?
(A) Sucrose is a disaccharide. 70. Nomenclature is governed by certain universal
(B) Cellulose is a polysaccharide. rules. Which one of the following is contrary
(C) Uracil is a pyrimidine. to the rules of?
(D) Glycine is a sulphur containing amino acid (A) Biological names can be written in any
language.
64. In bryophytes and pteridophytes, transport (B) The first word in a biological name represents
of gametes requires: the genus name, and the second is a specific
(A) Wind (B) Insects epithet
(C) Birds (D) Water (C) The names are written in Latin and are
italicised
65. Whendoes the growth rate of a population
(D) When written by hand, the names are to
following the logistic model equal zero ? The
be underlined
logistic model given as dN/dt = rN(1-N/K):
(A) when N/K is exactly one.
71. Blood pressure in the pulmonary artery is:
(B) when N nears the carrying capacity of
(A) same an that in the aorta.
habitat.
(B) more than that in the carotid.
(C) when N/K equals zero.
(C) more than that in the pulmonary vein
(D) when death rate is greater than birth rate.
(D) less than that in the venae cavae.
66. Wtuch one of the following statements is
not true ? 72. Cotyledon of maize grain is called:
(A) Tapetum helps in the dehiscence of anther (A) plumule (B) coleorhiza
(B) Exine of pollen grains is made up of (C) coleoptile (D) scutellum
sporopollenin
73. In the stomach, gastric acid is secreted by the:
(C) Pollen grains of many species cause
severe allergies (A) gastrin saecreting cells

(D) Stored pollen in liquid nitrogen can be (B) parietal cells


used in the crop breeding programmes (C) peptic cells (D) acidic cells
PREVIOUS YEAR PAPER [NEET - 2016] Page # 59
74. Depletion of which gas in the atmosphere 80. In a testcross involving F1 dihybrid flies, more
can lead to an increased incidence of skin parental-type offspring were produced than
cancers: the recombinant-type offspring. This indicates:
(A) Nitrous oxide (B) Ozone (A) The two genes are located on two
(C) Ammonia (D) Methane different chromosomes.
(B) Chromosomes failed to separate during
75. Chrysophytes/ Euglenoids, Dinoflagellates
meiosis.
and Slime moulds are included in the kingdom:
(C) The two genes are linked and present on
(A) Monera (B) Protista
the same chromosome.
(C) Fungi (D) Animalia
(D) Both of the characters are controlled by
76. Watervapour comes out from the plant leaf more than one gene.
through the stomatal opening. Through the
same stomatal opening carbon dioxide 81. It is much easier for a small animal to run
diffuses into the plant during photosynthesis. uphill than for a large animal, because:
Reason out the above statements using one (A) It is easier to carry a small body weight.
of following options: (B) Smaller animals have a higher metabolic
(A) Both processes cannot happen simultaneously. rate.
(B) Both processes can happen together (C) Small animals have a lower O2 requirement.
because the diffusion coefficient of water (D) The efficiency of muscles in large animals
and CO2 is different is less than in the small animals.
(C) The above processes happen only during
night time. 82. Which of the following is not a characteristic
(D) One process occurs during day time, and feature during mitosis in somatic cells ?
the other at night (A) Spindle fibres
(B) Disappearance of nucleolus
77. In mammals, which blood vessel would
(C) Chromosome movement
normally carry largest amount of urea ?
(D) Synapsis
(A) Renal Vein (B) Dorsal Aorta
(C) Hepatic Vein (D) Hepatic Portal Vein 83. Which of the following statements is not correct?
(A) Pollen grains of many species can germinate
78. Seed formation without fertilization in on the stigma of a flower, but only one pollen
flowering plants involves the process of: tube of the same species grows into the style.
(A) Sporulation (B) Budding
(B) Insects thatconsume pollen or nectar
(C) Somatic hybridization without bringing about pollination are called
(D) Apomixis pollen/nectar robbers.
(C) Pollen germination and pollen tube growth
79. Which of the following is wrongly matched in
are regulated by chemical components of
the given table?
pollen interacting with those of the pistil.
Microbe Product Application
(D) Some reptiles have also been reported
Trichoderma immunosuppressive
(A) Cyclosporin A as pollinators in some plant species.
polysporum drug

Monascus lowering of blood


(B) Statins 84. Specialis edepidermal cells surrounding the
purpureus cholesterol
guard cells are called:
removal of clot
(C) Streptococcus Streptokinase
from blood vessel (A) Complementary cells
Clostridium removal of oil (B) Subsidiary cells
(D) Lipase
butylicum stains (C) Bulliform cells (D) Lenticels
Page # 60 PREVIOUS YEAR PAPER [NEET - 2016]
85. Which of the following guards the opening of 89. A cell at telophase stage is observed by a
hepatopancreatic duct into the duodenum ? student in a plant brought from the field. He
(A) Semilunar valve (B) Ileocaecal valve tells his teacher that this cell is not like other
(C) Pyloric sphincter (D) Sphincter of Oddi cells at telophase stage. There is no formation
of-cell plate and thus the cell is containing
86. &ems modified into flat green organs performing more number of chromosomes as compared
the functions of leaves are known as: to other dividing cells. This would result in:
(A) Cladodes (B) Phyllodes (A) Aneuploidy (B) Polyploidy
(C) Phylloclades (D) Scales (C) Somaclonal variation
(D) Polyteny
87. The primitive prokaryotes responsible for the
production of biogas from the dung of ruminant 90. A typical fat molecule is made up of:
animals, include the:
(A) Three glycerol molecules and one fatty
(A) Halophiles (B) Thermoacidophiles acid molecule
(C) Methanogens (D) Eubacteria (B) One glycerol and three fatty acid molecules

88. A river with an inflow of domestic sewage (C) One glycerol and one fatty acid molecule
rich in organic waste may result in: (D) Three glycerol and three fatty acid
(A) Drying of the river very soon due to algal molecules
bloom.
(B) Increased population of aquatic food web
organisms.
(C) An increased production of fish due to
biodegradable nutrients.
(D) Death of fish due to lack of oxygen.
PREVIOUS YEAR PAPER [NEET - 2016] Page # 61

PHYSICS
1. From a disc of a radius R and mass M, a 6. In a diffraction pattern due to a single slit of
circular hole of diameter R, whose rim passes width ‘a’, the first minimum is observed at an
through the centre is cut. What is the moment angle 30° when light of wavelength 5000Å is
of inertia of the remaining part of the disc incident on the slit. The first secondary
about a perpendicular axis, passing through maximum is observed at an angle of:
the centre?
(A) 15 MR2/32 (B) 13 MR2/32 1  1  1  2 
(C) 11 MR2/32 (D) 9 MR2/32 (A) sin  4  (B) sin  3 
   

2. A square loop ABCD carrying a current i, is 1  1  1  3 


placed near and coplanar with a long straight (C) sin   (D) sin  
conductor XY, carrying a current I, the net 2 4
force on the loop will be :
Y B C 7. At what height from the surface of earth
the gravitation potential and the value of g
are –5.4×107J kg-2 and 6.0 ms-2 respectively?
i
Take the radius of earth as 6400 km.
I L
(A) 2600 km (B) 1600 km
(C) 1400 km (D) 2000 km

x A D 8. Out of the following options which one can


L/2 L be used to produce a propagating
20Ii  0Ii electromagnetic wave?
(A) (B) (A) A charge moving at constant velocity.
3 2
(B) A stationary charge
20IiL 0IiL (C) A chargeless particle
(C) (D)
3 2 (D) An accelerating charge

3. The magnetic susceptibility is negative for : 9. Two identical charged spheres suspended
(A) diamagnetic material only from a common point by two massless strings
(B) paramagnetic material only of lengths l, are initially at a distance d (d<<l)
(C) ferromagnetic material only apart because of their mutual repulsion. The
(D) paramagnetic and ferromagnetic materials charges begin to leak from both the spheres
at a constant rate. As a result, the spheres
4. A siren emitting a sound of frequency 800
approach each other with a velocity . Then
Hz moves away from an observer towards a
 varies as a function of the distance x
cliff at a speed of 15 ms -1 . Then, the
frequency of sound that the observer hears between the spheres, as :
in the echo reflected from the cliff is : (A)   x 2
1
(B)   x
(Take velocity of sound in air = 330 ms-1)
(A) 765 Hz (B) 800 Hz 1

(C) 838 Hz (D) 885 Hz (C)   x 2 (D)   x 1

5. A capacitor of 2 F is charged as shown in 10. A uniform rope of length L and mass m1 hangs
the diagram. When the switch S is turned to vertically from a rigid support. A block of mass
position 2, the percentage of its stored m2 is attached to the free end of the rope. A
energy dissipated is : transverse pulse of wavelength 1 is produced
1 2
at the lower end of the rope. The wavelength
of the pulse when it reaches the top of the
S
ropes is 2. The ratio 2/1 is :

V m1 m1  m2
2F 8F (A) m (B)
2 m2

m2 m1  m2
(A) 0% (B) 20% (C) m1 (D) m1
(C) 75% (D) 80%
Page # 62 PREVIOUS YEAR PAPER [NEET - 2016]
11. A refrigerator works between 4°C and 30°C. 17. A npn transistor is connected in common
It is required to remove 600 calories of heat emitter configuration in a given amplifier. A
every second in order to keep the load resistance of 800  is connected in the
temperature of the refrigerated space collector circuit and the voltage drop across
constant. The power required is : (Take 1 it is 0.8V. If the current amplification factor
cal = 4.2 Joules) is 0.96 and the input resistance of the circuit
(A) 2.365 W (B) 23.65W is 192, the voltage gain and the power gain
(C) 236.5W (D)2365W of the amplifier will respectively be :
(A) 4, 3.84 (B) 3.69, 3.84
12. An air column, closed at one end and open (C) 4, 4 (D) 4, 3.69
at the other, resonates with a tuning fork
when the smallest length of the column is 50 18. The intensity at the maximum in a Young’s
cm, The next larger length of the column double slit experiment is I0. Distance between
resonating with the same tuning fork is : two slits is d = 5, where  is the wavelength
(A) 66.7 cm (B) 100 cm of light used in the experiment. What will be
(C) 150 cm (D) 200cm the intensity in front of one of the slits on
the screen placed at a distance D = 10 d?
13. Consider the junction diode as ideal. The I0
(A) I0 (B)
value of current flowing through AB is : 4
A 1k B
3 I0
+4V –6V (C) I (D)
4 0 2
(A) 0 A (B) 10-2 A
(C) 10-1 A (D) 10-3 A
19. A uniform circular disc of radius 50 cm at
rest is free to turn about an axis which is
14. The charge flowing through a resistance R
perpendicular to its plane and passes through
varies with time t as Q = at – bt2, where a
its centre. It is subjected to a torque which
and b are positive constants. The total heat
produces a constant angular acceleration of
produced in R is :
2.0 rad s-2. Its net acceleration in ms-2 at
a3R a3R the end of 2.0 s is approximately:
(A) (B) (A) 8.0 (B) 7.0
6b 3b
(C) 6.0 (D) 3.0
a3R a3R
(C) (D)
2b b 20. An electron of mass m and a photon have
same energy E. The ratio of de-Broglie
15. A black body is at a temperature of 5760 K. wavelengths associated with them is :
The energy of radiation emitted by the body 1 1

at wavelength 250 nm is U1, at wavelength 1  E 2  E 2


(A)  (B) 
500 nm is U2 and that at 1000 nm is U3. c  2m  
 2m 
Wien’s constant, b = 2.88×106 nmK. Which
1/2
1 1  2m 
of the following is correct? (C) c(2mE)2 (D)  
c  E 
(A) U1 = 0 (B) U3 = 0
(C) U1>U2 (D) U2>U1 (being velocity of light)

16. Coefficient of linear expansion of brass and 21. A disk and a sphere of same radius but
steel rods are 1 and 2. Length of brass and different masses roll off on two inclined
steel rods are l1 and l2 respectively. If (l2–l1) planes of the same altitude and length. Which
is maintained same at all temperatures, which one of the two objects gets to the bottom
one of the following relations holds good? of the plane first?
(A) Disk
(A) 1l2   2l1 (B) 1l22  2l12 (B) Sphere
(C) 12l2  22l1 (D) 1l1  2l2 (C) Both reach at the same time
(D) Depends on their masses
PREVIOUS YEAR PAPER [NEET - 2016] Page # 63
22. The angle of incidence for a ray of light at a 27. A particle moves so that its position vector
refracting surface of a prism is 45°. The angle 
is given by r  cos tx ˆ . Where  is a
ˆ  sin ty
of prism is 60°. If the ray suffers minimum
deviation through the prism, the angle of constant.
minimum deviation and refractive index of the Which of the following is true?
material of the prism respectively, are :
1 (A) Velocity and acceleration both are
(A) 45°; (B) 30°; 
2 2 perpendicular to r
(B) Velocity and acceleration both are parallel
1 
(C) 45°; 2 (D) 30°; to r
2 
(C) Velocity is perpendicular to r and
acceleration is directed towards the origin.
23. When an –particle of mass ‘m’ moving with 
velocity ‘’ bombards on a heavy nucleus of (D) Velocity is perpendicular to r and
charge ‘Ze’ its distance of closest approach acceleration is directed away from the origin.
from the nucleus depends on m as :
1 28. What is the minimum velocity with which a
1
(A) (B) body of mass m must enter a vertical loop of
m m
radius R so that it can complete the loop?
1
(C) (D) m
m2 (A) (B)
gR 2gR
24. A particle of mass 10 g moves along a circle (C) 3gR (D) 5gR
of radius 6.4 cm with a constant tangential
acceleration. What is the magnitude of this
acceleration if the kinetic energy of the 29. When a metallic surface is illuminated with
particle becomes equal to 8×10-4 J by the radiation of wavelength , the stopping
end of the second revolution after the potential is V. If the same surface is illuminated
beginning of the motion? with radiation of wavelength 2, the stopping
(A) 0.1 m/s2 (B) 0.15 m/s2
(C) 0.18 m/s2 (D) 0.2 m/s2 V
potential is . The threshold wavelength
4
25. The molecules of a given mass of a gas have for the metallic surface is :
r.m.s. velocity of 200 ms -1 at 27°C and
1.0×105 Nm-2 pressure. When the temperature (A) 4 (B) 5
and pressure of the gas are respectively,
127°C and 0.05×105Nm-2, the r.m.s. velocity 5
(C)  (D) 3
of its molecules in ms-1 is : 2
400
(A) 100 2 (B) 30. A gas is compressed isothermally to half its
3
initial volume. The same gas is compressed
100 2 100 separately through an adiabatic process until
(C) (D) its volume is again reduced to half. Then :
3 3
(A) Compressing the gas isothermally will
26. A long straight wire of radius a carries a require more work to be done.
steady current I. The current is uniformly (B) Compressing the gas through adiabatic
distributed over its cross–section. The ratio
process will require more wore to be done.
of the magnetic fields B and B’, at radial dis-
(C) Compressing the gas isothermally or
a adiabatically will require the same amount of
tances and 2a respectively, from the axis
2 work.
of the wire is (D) which of the case (whether compression
through isothermal or through adiabatic
1 1 process) requires more work will depend upon
(A) (B) the atomicity of the gas.
4 2
(C) 1 (D) 4
Page # 64 PREVIOUS YEAR PAPER [NEET - 2016]
31. A potentiometer wire is 100 cm long and a 37. If the magnitude of sum of two vectors is
constant potential difference is maintained equal to the magnitude of difference of the
across it. Two cells are connected in series two vectors, the angle between these vec-
first to support one another and then it op- tors is :
posite direction. The balance points are ob- (A) 0° (B) 90°
tained at 50 cm and 10 cm from the positive (C) 45° (D) 180°
end of the wire in the two cases. The ratio
of emf’s is : 38. Given the value of Rydberg constant is 107
(A) 5 : 1 (B) 5 : 4 m–1 the wave number of the last line of the
(C) 3 : 4 (D) 3 : 2 Balmer series in hydrogen spectrum will be :

32. A astronomical telescope has objective and (A) 0.025 × 104 m–1 (B) 0.5 × 107 m–1
eyepiece of focal lengths 40 cm and 4 cm (C) 0.25 × 107 m–1 (D) 2.5 × 107 m–1
respectively. To view an object 200 cm away
from the objective, the lenses must be 39. A body of mass 1 kg begins to move under
separated by a distance : the action of a time dependent force
(A) 37.3 cm (B) 46.0 cm 
(C) 50.0 cm (D) 54.0 cm F  2tˆi  3t2ˆj  N, where î and ĵ are unit
vectors along x and y axis. What power will
33. Two non–mixing liquids of densities  and n be developed by the force at the time t?
(n > 1) are put in a container. The height of
each liquid is h. A solid cylinder of length L (A) (2t2 + 3t3)W (B) (2t2 + 4t 4)W
and density d is put in this container. The (C) (2t3 + 3t4) (D) (2t3 + 3t5)W
cylinder floats with its axis vertical and length
pL(p < 1) in the denser liquid. The density d 40. An inductor 20 mH, a capacitor 50 F and a
is equal to :
resistor 40  are connected in series across
(A) {1 + (n + 1)p}
(B) {2 + (n + 1)p} a source of emf V = 10 sin 340 t. The power
(C) {2 + (n – 1)p} loss in A.C. circuit is :
(D) {1 + (n – 1)p}
(A) 0.51 W (B) 0.67 W
34. To get output 1 for the following circuit, the (C) 0.76 W (D) 0.89 W
correct choice for the input is :
A 41. If the velocity of a particle is v = At + Bt2,
B where A and B are constants, then the dis-
C Y
tance travelled by it between 1s and 2s is :
(A) A = 0, B = 1, C = 0
(B) A = 1, B = 0, C = 0
(C) A = 1, B = 1, C = 0
3
(D) A = 1, B = 0, C = 1 (A) A  4B (B) 3A + 7B
2
35. A piece of ice falls from a height h so that it
melts completely. Only one–quarter of the 3 7 A B
heat produced is observed by the ice and all (C) A B (D) 
2 3 2 3
energy of ice gets converted into heat dur-
ing its fall. The value of h is :
[Latent heat of ice is 3.4 × 105J/kg and g = 42. A long solenoid has 1000 turns. When a cur-
10 N/kg]
(A) 34 km (B) 544 km rent of 4A flows through it, the magnetic
(C) 136 km (D) 68 km flux linked with each turn of the solenoid is 4
36. The ratio of escape velocity at earth (ve) to × 10–3 Wb. The self–inductance of the sole-
the escape velocity at a planet (vp) whose
radius and mean density are twice as that of noid is :
earth is : (A) 4H (B) 3H
(A) 1 : 2 (B) 1 : 2 2 (C) 2H (D) 1H
(C) 1 : 4 (D) 1 : 2
PREVIOUS YEAR PAPER [NEET - 2016] Page # 65

43. A small signal voltage V(t) = V0 sin t is 45. A car is negotiating a curved road of radius
applied across an ideal capacitor C : R. The road is banked at an angle . The
(A) Current I(t), lags voltage V(t) by 90° coefficient of friction between the tyres of
(B) Over a full cycle the capacitor C does not the car and the road is s. The maximum
consume any energy from the voltage source. safe velocity on this road is :
(C) Current I(t) is in phase with voltage V(t).
(D) Current I(t) leads voltage V(t) by 180°.  s  tan   s  tan 
(A) gR2 gR
1   s tan  (B) 1   s tan 

44. Match the corresponding entries of column


1 with column 2. [Where m is the g s  tan  g  s  tan 
(C) (D)
magnification produced by the mirror] R 1   s tan  R 2 1  s tan 
Column 1 Column 2
(A) m = –2 (a) Convex mirror
1
(B) m =  (b) Concave mirror
2
(C) m = +2 (c) Real image
1
(D) m =  (d) Virtual image
2
(A) A  b and c ; B  b and c; C  b and d;
D  a and d
(B) A  a and c ; B  a and d; C  a and b;
D  c and d
(C) A  a and d ; B  b and c; C  b and d;
D  b and c
(D) A  c and d ; B  b and d; C  b and c;
D  a and d
Page # 66 PREVIOUS YEAR PAPER [NEET - 2016]

CHEMISTRY
46. Consider the molecules CH4 NH3 and H2O. 52. Which of the folloiwing statements is false ?
Which of the given statements is false ? (A) Mg2+ ions form a complex with ATP
(A) The H – C – H bond angle in CH4, the H – (B) Ca2+ ions are important in blood clotting
N – H bond angle in NH3, and the H– O – H (C) Ca2+ ions are not important in maintaining
bond angle in H2O are all greater than 90º. the regular beating of the heart
(B) The H – O – H bond angle in H2O is larger (D) Mg2+ ions are important in the green parts
than the H – C – H bond angle in CH4 of plants
(C) The H – O – H bond angle in H2O is
smaller than the H – N – H bond angle in NH3 53. Which of the following statements about
(D) The H – C – H bond angle in CH4 is larger hydrogen is incorrect ?
(A) Hydrogen has three isotopes of which
than the H – N – H bond angle in NH3
tritium is the most common
(B) Hydrogen never acts as cation in ionic
47. In the reaction
salts
H – C 
(1)NaNH2 / liq.NH4
CH   (C) Hydronium ion, H3O + exists freely in
(2)CH4CH2Br
solution
(1)NaNH2 /liq.NH3 (D) Dihydrogen does not act as a reducing
X  Y
(2)CH CH Br
3 2 agent
X and Y are :
(A) X = 1-Butyne ; Y = 3-Hexyne 54. The correct statement regarding a carbonyl
(B) X = 2-Butyne ; Y = 3-Hexyne compound with a hydrogen atom on its
(C) X = 2-Butyne ; Y = 2-Hexyne alphacarbon is :
(D) X = 1-Butyne ; Y = 2-Hexyne (A) a carbonyl compound with a hydrogen
atom on its alpha-carbon never equilibrates
48. Among the following the correct order of with its corresponding enol
acidity is : (B) a carbonyl compound with a hydrogen
atom on its alpha-carbon rapidly equilibrates
(A) HClO3 < HClO4 < HClO2 < HClO
with its corresponding enol and this process
(B) HClO < HClO2 < HClO3 < HClO4
is known as aldehyde-ketone equilibration.
(C) HClO2 < HClO2 < HClO < HClO3
(C) a carbonyl compound with a hydrogen
(D) HClO4 < HClO2 < HClO < HClO3
atom on its alpha-carbon rapidly equilibrates
with its corresponding enol and this process
49. The rate of first-order reaction is 0.04 mol l– is known as carbonylation
1 s–1 at 10 seconds and 0.03 mol l–1 s–1 at
(D) a carbonyl compound with a hydrogen
20 seconds after initiation of the reaction. atom on its alpha-carbon rapidly equilibrates
The half-life period of the reaction is : with its corresponding enol and this process
(A) 24.1 s (B) 34.1 s is known as keto-enol tautomerism
(C) 44.1 s (D) 54.1 s
55. MY and NY3 two nearly insoluble salts, have
50. Which one of the following characteristics is the same Ksp values of 6.2 × 10–13 at room
associated with adsorption ? temperature.
(A) G is negative but H and S are positive Which statement would be ture in regard to
(B) G, H and S all are negative MY and NY3 ?
(C) G and H are negative but S is positive (A) The molar solubilities of MY and NY3 in
(D) G and S are negative bu H is positive water are identical
(B) The molar solubility of MY in water is less
51. In which of the following options the order than that of NY3
of arrangement does not agree with the (C) The salts MY and NY3 are more soluble in
variation of property indicated against it ? 0.5 M KY than in pure water.
(A) Al3+ < Mg2+ < Na+ < F– (Increasing ionic (D) The addition of the salt of KY to solution
size) of MY and NY3 will have no effect on their
solubilities
(B) B < C < N < O (Increasing first ionisation
enthalpy) 56. In a protein molecule various amino acids are
(C) I < Br < Cl < F (Increasing electron gain linked together by :
enthalpy) (A) -glycosidic bond
(D) Li < Na < K < Rb (Increasing metallic (B) -glycosidic bond
radius) (C) peptide bond
(D) dative bond
PREVIOUS YEAR PAPER [NEET - 2016] Page # 67
57. Natural rubber has : 64. The correct thermodynamic conditions for the
(A) All cis-configuration spontaneous reaction at all temperature is :
(B) All trans-configuration (A) H < 0 and S = 0
(C) Alternate cis-and trans-configuration (B) H > 0 and S < 0
(D) Random cis-and trans-configuration (C) H < 0 and S > 0
(D) H < 0 and S < 0
58. Match items of Column I with the items of
Column II and assign the correct code :
65. Lithium has a bcc structure. Its density is 530
Column I Column II kg m–3 and its atomic mass is 6.94 g mol–1.
(i) Ultrapure Ge
(a) Cyanide process Calculate the edge length of a unit cell of
(ii) Dressing of ZnS
(b) Forth floatation process Lithium metal. (NA = 6.02 × 1023 mol–1)
(iii) Extraction of Al
(c) Electrolytic reduction (A) 154 pm (B) 352 pm
(iv) Extraction of Au
(d) Zone refining (C) 527 pm (D) 264 pm
(iv) Purification of Ni
Code :
66. Which one of the following orders is correct for
(a) (b) (c) (d)
the bond dissociation enthalpy of halogen
(A) (iv) (ii) (iii) (i)
(B) (ii) (iii) (i) (iv) molecules ?
(C) (i) (ii) (iii) (iv) (A) I2 > Br2 > Cl2 > F2
(D) (iii) (iv) (v) (i) (B) Cl2 > Br2 > F2 > I2
(C) Br2 > I2 > F2 > Cl2
59. Which one of the following statements is (D) F2 > Cl2 > Br2 > I2
correct when SO2 is passed through acidified
K2Cr2O7 solution ? 67. Which of the following is an analgesis ?
(A) The solution turns blue (A) Novalgin (B) Penicillin
(B) The solution is decolourized (C) Streptomycin (D) Chloromycetin
(C) SO2 is reduced
(D) Green Cr2 (SO4)3 is formed 68. Equal moles of hydrogen and oxygen gases
are placed in a container with a pin-hole
60. The electronic configurations of Eu (Atomic No. through which both can escape. What
63), Gd(Atomic No. 64) and Tb (Atomic No. 65)
fraction of the oxygen escapes in the time
are :
required for one-half of the hydrogen to
(A) [Xe]4 7 6s 2 , [Xe]4 8 6s 2 and [Xe]
485d16s2 escape ?
(B) [Xe]465d16s2, [Xe]47 5d1 6s2 and [Xe] (A) 1/8 (B) 1/4
496s2 (C) 3/8 (D) 1/2
(C) [Xe]465d16s2, [Xe]47 5d16s2 and [Xe]
485d16s2 69. Consider the nitration of benzene using mixed
(D) [Xe]476s2, [Xe]47 5d 16s2 and [Xe] conc. H2SO4 and HNO3. If a large amount of
496s2 KHSO4 is added to the mixture, the rate of
nitration will be :
61. Two electrons occupying the same orbital (A) faster (B) slower
are distinguished by: (C) unchanged (D) doubled
(A) Principal quantum number
(B) Magnetic quantum number 70. Predict the correct order among the following :
(C) Azimuthal quantum number
(A) lone pair - lone pair > lone pair - bond
(D) Spin quantum number
pair > bond pair - bond pair
(B) lone pair - lone pair > bond pair - bond
62. When copper is heated with conc. HNO3 it
produces : pair > lone pair - bond pair
(A) Cu(NO3)2 and NO2 (C) bond pair - bond pair > lone pair - bond
(B) Cu(NO3)2 and NO pair > lone pair - lone pair
(C) Cu(NO3)2, NO and NO2 (D) lone pair - bond pair > bond pair - bond
(D) Cu(NO3)2 and N2O pair > lone pair - lone pair

63. Which of the following reagents would 71. The product obtained as a result of a reaction
distinguish cis-cyclopenta-1, 2-diol from the of nitrogen with CaC2 is :
trans-isomer ? (A) Ca(CN)2 (B) CaCN
(A) Acetone (B) Ozone
(C) CaCN3 (D) Ca2CN
(C) MnO2 (D) Aluminium isopropoxide
Page # 68 PREVIOUS YEAR PAPER [NEET - 2016]
72. Consider the following liquid - vapour 79. Fog is a colloidal solution of :
equilibrium. (A) Liquid in gas (B) Gas in liquid


Liquid 
 Vapour (C) Solid is gas (D) Gas in gas
Which of the following relations is correct ?
80. Which of the following statements about the
dln G H dlnP H composition of the vapour over an ideal 1 : 1
(A) = (B) =
dT 2 RT 2 dT RT mol mixture of benzene and toluene is correct
H H dlnP H ? Assume that the temperature is constant
(C) = (D) = at 25°C. (Given Vapour Pressure Data is 25°C,
RT T2 dT RT 2
benzene = 12.8 kJ toluene = 3.85 kPa)
73. Match the compounds given in column I with (A) The vapour will contain a higher
the hybridisation and shape given in column percentage of benzene.
II and mark the correct option. (B) The vapour will contain a higher
Column I Column II
percentage of toluene.
(a) XeF6 (i) distorted octahedral
(b) XeO3 (ii) square planar (C) The vapour will contain equal amounts
(c) XeOF4 (iii) pyramidal benzene and toluene.
(d) XeF4 (iv) square pyramidal (D) Not enough information is given to make
Code : prediction.
(a) (b) (c) (d)
(A) (i) (iii) (iv) (ii)
81. The correct statement regarding the
(B) (i) (ii) (iv) (iii)
(C) (iv) (iii) (i) (ii) comparison staggered and eslipsed
(D) (iv) (i) (ii) (iii) conformations of ethane,
(A) The staggered conformation of ethane is
74. Which of the following has longest C — O low stable than eclipsed conformation,
bond length ? (Free C — O bond length in CO because staggered conformation has torsional
is 1.128 Å.) strain
(A) Ni(CO)4 (B) [Co(CO)4]2–
(B) The esclipsed conformation of ethane is
(C) [Fe(CO)4]2– (D) [Mn(CO)6]+
more stable than staggered conformation,
75. The pressure of H2 required to make the because eclipsed conformation has no
potential of H2 - electrode zero in pure water torsional strain
at 298 K is : (C) The eclipsed conformation of ethen is
(A) 10–14 atm (B) 10–12 atm more stable than staggered conformation
(C) 10–10 atm (D) 10–4 atm even though the eclipsed conformation has
torsional strain.
76. The addition of a catalyst during a chemical
reaction alters which of the following (D) The staggered conformation of ethane is
quantities ? more stable than eclipsed conformation,
(A) Entropy (B) Internal energy because staggered conformation has no
(C) Enthalpy (D) Activation energy torsion strain.

77. The ionic radii of A+ and B– ions are 0.98 × 82. The reaction
10 –10 m and 1.81 × 10 –10 m. The
coordination number of each ion in AB is :
(A) 6 (B) 4
(C) 8 (D) 2 can be classified as :
(A) Williamson ether synthesis reaction
78. Which is the correct statement for the given
acids ? (B) Alcohol formation reaction
(A) Phosphinic acid is a diprotic acid while (C) Dehydration reaction
phosphonic acid is a monoproti acid. (D) Williamson alcohol synthesis reaction
(B) Phosphinic acid is a monoprotic acid while
phosphonic acid is a diprotic acid. 83. The product formed by the reaction of an
(C) Both are triprotic acids aldehyde with a primary amine is :
(D) Both are diprotic acids.
(A) Schiff base (B) Ketone
(C) Carboxylic acid (D) Aromatic acid
PREVIOUS YEAR PAPER [NEET - 2016] Page # 69
84. Which of the following biphenyl is optically 87. The correct statement regarding RNA and
active? DNA respectively is
(A) The sugar component in RNA is arabinose
O2N
and the sugar component in DNA is 2-
deoxyribose.
(A) (B) The sugar component in RNA is ribose
and the sugar component in DNA is 2-
I deoxyribose.
(C) The sugar component in RNA is arabinose
Br Br
and the sugar component in DNA ribose.
(D) The sugar component in RNA is 2-
deoxyribose and the sugar component in DNA
(B)
is arabinose.
I I
88. The correct statement regarding the basicity
I of arylamines is :
(A) Arylamines are generally less basis than
(C) alkylamines because the nitrogen lone-pair
electrons are delocalized by interaction with
I the aromatic ring  electron system
(B) Arylamines are generally more basis than
CH3 alkylamines because the nitrogen lone-pair
electrons are not delocalized by interaction
(D) with the aromatic ring  electron system
(C) Arylamines are generally more basis than
CH3 alkylamines because of aryl group.
(D) Arylamines are generally more basis than
85. For the following reaction alkylamines because the nitrogen atom in
(A) CH3CH2CH2Br+KOH  arylamines is sp-hybridized.
CH3CH = CH2 + KBr + H2O
89. Which one given below is a non-reducing
H 3C CH3 H3C CH 3
sugar?
(B) + KOH + KBr (A) Maltose (B) Lactose
Br OH (C) Glucose (D) Sucrose
Br
+ Br2 90. The pair of electron in the given carbanion,
(C)
Br CH3  C , is present in which of the following
Which of the following statements is correct? orbitals?
(A) (a) and (b) are elimination reactions and (A) 2p (B) sp3
(c) is addition reaction. (C) sp2 (D) sp
(B) (a) is elimination, (b) is substitution and
(c) is addition reaction.
(C) (a) is elimination, (b) (c) are substitution
reaction
(D) (a) is substitution, (b) and (c) are
addition reaction

86. At 100ºC the vapour pressure of a solution


of 6.5 g of a solute in 100 g water is 732
mm. If Kb = 0.52, the boiling point of this
solution will be :
(A) 101°C (B) 100°C
(C) 102°C (D) 103°C
Page # 70 PREVIOUS YEAR PAPER [NEET - 2016]

ANSWERKEY
BIOLOGY
1. C 2. D 3. D 4. D 5. C 6. B 7. B
8. [Bonus] 9. A 10. A 11. C 12. D 13. B 14. B
15. B 16. B 17. C 18. D 19. B 20. A 21. B
22. B 23. C 24. C 25. A 26. A 27. C 28. A
29. A 30. C 31. B 32. A 33. D 34. B 35. C
36. B 37. C 38. D 39. C 40. D 41. D 42. A
43. B 44. D 45. D 46. C 47. C 48. A 49. D
50. C 51. B 52. A 53. B 54. D 55. A 56. D
57. A 58. C 59. C 60. B 61. A 62. B 63. D
64. D 65. A 66. A 67. A 68. A 69. C 70. A
71. C 72. D 73. B 74. B 75. B 76. B 77. C
78. D 79. D 80. C 81. B 82. D 83. A 84. B
85. D 86. C 87. C 88. D 89. B 90. B

PHYSICS

1. B 2. A 3. A 4. C 5. D 6. D 7. A

8. D 9. C 10. B 11. C 12. C 13. B 14. A

15. D 16. D 17. A 18. D 19. A 20. A 21. B

22. B 23. A 24. A 25. B 26. C 27. C 28. D

29. D 30. B 31. 4 32. 4 33. D 34. D 35. C

36. B 37. B 38. C 39. D 40. A 41. C 42. D

43. B 44. A 45. B

CHEMISTRY

1 B 2 A 3 B 4 A 5 B 6 B or C 7 C

8 B 9 D 10 B 11 C 12 A 13 A 14 D

15 D 16 D 17 A 18 A 19 C 20 B 21 B

22 A 23 A 24 B 25 A 26 Bonus 27 D 28 A

29 C 30 A 31 D 32 A 33 B 34 A 35 A

36 D 37 A 38 A 39 B 40 B 41 A 42 B

43 A 44 D 45 D
PREVIOUS YEAR PAPER [NEET - 2016] Page # 71

SOLUTION
PHYSICS
1. B
6. D
2
 M R 2  R  
 2   2  5000  10 10 1
MR 2 R 4  2   M  R   sin  = = =
= –  a a 2
2 2 4  2  
  a = 10 × 10–7
 
for first sec. maxima
M

MR 2 M M x x R 2 3
= –  32  16  a sin  =
2   2

1 3  2
= MR2    = 13MR sin  =
3
=
3
 2 32  32 2a 4

2. A 3
 = sin–1  
0I 4
0I
. i.L – . i.L
2(L / 2)  3L 
2 
 2  7. A
GM
0IiL 2  2  2 0Ii Vg = –
R h
;  ;
2L  3  3
GM
gh = =6
3. A (R  h)2
c = –ve for diamagnetic material only
Vg 5.4  107
4. C gh
=R+h=
6
nv
n’ = v  v R + h = 9 × 106
s h = 9000 – 6400
330 =2600 km
= 800 × = 838 Hz
315
8. D
By theory

9. C
n' s O
kq2
T sin  = ; T cos  = mg
d2
 Observer hears
n’ = 838 Hz
kq2 d
tan  = =
5. D mgd2 l
1 q2  d3 ; q  d3/2
U1 = cv2
2
dq 3
C1C2 dt

2 d .v
1
U = (C (V1 – V2)2
2 1  C2 )
dq
= constant
1  2  8  dt
U = × 10–6 (V2)
2  10 
1
U 8 v
d
U1 × 100 = 10 × 100 = 80%
 v  x–1/2
Page # 72 PREVIOUS YEAR PAPER [NEET - 2016]
10. B 14. A
T2 = (m1 + m0) g Q = at – bt2
Tension T1 = m2 g
d
i= = a – 2bt i=0
T dt
v = n =

a
t=
2b
v1 1 T1 m2
H = i2RT dH = l2Rdt
v2 =  2 = T2 = m1  m2 m2
a / 2b
2
1 m2 =  (a  2bt) Rdt
2 =
0
m1  m2
2
=R  (a  4b2 t2  4abt)dt
2 m2  m1
1 = m2 a
3
 2 2 t 4abt 2  2b
= R a t  4b  
11. C  3 2 a
T1 = 303
T2 = 277  a3 4b2  a3  4ab  a2 
= R  2b  3  8b3   2  4b2 
Q2 Q2 T2     
= = Q Q = T T
w 1 2 1 2
Ra3  1  1  1  a3R
= 2 6 2  =
b   6b
T1 work
15. D
600
b
=
T
T2
2.88  106
=
5760
Q2 277
= = 10.61 = 500 nm
w 26 u2 = max
Q2 600  4 .2 u2 > u1
w= =
10.61 10.61
16. D
= 236.5
l1 = l2
l11t = l22l
12. C
l11 = l22

l1 =
4 17. A
By theory
3
l2 =
4
18. D
l2 = 3l1
l2 = 150 cm 
I = I0 cos2  2 
 
13. B
i = VA – VB = + 4 – (–6) = 10 x = d sin 

10 2 2
i= = 10–2 A = x = d sin 
1000  

–4 –6 2
= × 5 sin 
A B 
PREVIOUS YEAR PAPER [NEET - 2016] Page # 73
21. B
d
2 2g(1  k2 / r 2 )
= 10  × t=
(10d)2  (d / 2)2 gsin 

k2
d  t 
= 10 × r2
2  10d sphere
= /2
22. B

I = I0 cos2  4 
 
60°
I 45°
= 0
2

19. A
 = 2i–A
 = I
= 2 × 45° – 60
=2
= 90 – 60
a = r
= 30°
1
a= ×2=1  A  m 
2 sin 
at = 1 =  2 
v = u + at sin A / 2
=0+2
v=2  60  30 
sin 
=  2 
v2 4 sin 30
ar = = =8
r 0.5
sin 45
=
a= a2r  a2t sin 30

= 64  1 12
= = 2
= 8 (approx) 2 1

20. A 23. A
Ui + Ki = Uf + Kf

e = ....(1) 1
2meE O+ mv2 = (K 2e) (2e)
2
h k4ze2
p = ....(2) r=
p mv2
1
E  mc2  r
 m
E  mc  c 

E  pc  24. A
P  E /c  r = 6.4 cm
m = 10 gm
hc 1 1
 photon = mv2 = 8 × 10–4 ; × 10–2 v2 = 8 × 10–4
E 2 2
v2 = 16 × 10–2
e h E v = 0.16
  = ×
p 2mE hc v2 = u2 + 2as

e 16 16 64
l E × = 2 × a × 2 × ×2
p = c 2m
100 100 100  10
a = 0.1 m/s2
Page # 74 PREVIOUS YEAR PAPER [NEET - 2016]
25. B 31. 4
T = 300
P = 105 E1  E2 50
 5
Vrms = 200 E1  E2 10
V T E1 + E2 = 5E1 – 5E2
V2 4E1 = 6E2
400
V1 = 300 E1 6 3
 
400 E2 4 2
V2 =
3
32. 4
26. C
1 1 1
+ =
0Ir2 v 200 40
B1 2R v = 50 cm
B2 = I
0 a/2
2d v0 ue

2 2a
 0I(R / 2)
B1 2R
B2 =  0I
=1:1
2(2R )
200cm
27. C
r–1 = cos ti + sin tj f0=40cm fc=4cm
v = –  sin t i +  cos tj
a = – 2 cos ti – 2 sin tj

r.v = 0 5 1
200
28. D
By theory 200
v=
4
29. D
50 + 4
hc hc ev = 54 cm
= w + ev ; =w+
 2 4
hc 33. D
h'  w 2hc hc
=w+  ; = 4w + –w
2 4  
(1-p)L
hc
3w = pL

hc
w=
3
L × A × d × g = PL × n × A × g × (1 – P)
0 = 3
L××A×g
30. B d = PN + (1 – P) 
AD = (pn + 1 – p)
=  [(1 + P (n–1)]

IT 34. D
y = (A + B) · C
y=A·C+B·C
Possibel when all ABC are 1 or when AC = 1
V/2 V or A · B = 1
WAD > WIT From options 4 is correct.
PREVIOUS YEAR PAPER [NEET - 2016] Page # 75
35. C 2
P  402  52 
1
mgh = mL 2
vrms R
4 P=
z2
4L
h= 2
g  10 
   40
2
4  3.4  105 P 
= 65.6  65
100
136 × 103 100  40
= 136 km P = 0.46  0.51W
2  4303.7
36. B
41. C
2GM 2G 4 V = At + Bt2
Ve = = . R 3
R R 3 2
x=  (At  Bt )dt
8
= GR 2 2
3  At 2 Bt 3 
  
R’ = 2R  2 3 1
’ = 2
V0 ’ = 2 2 V  4  1 8  1
=A  2  +B  3 
   
37. B
    3 7
A B  A B = A + B
2 3
 = 90°
42. D
38. C
N 1000  4  103
1 1 1 L  =1H
= 107    I 4
 4 
1 43. B
= 0.25 × 107 m–1 Option 2 is correct because it is case of

watt less current.
39. D
F = 2ti + 3t2j ....(1) 44. A
Concave
dv m = –2
m = 2t i + 3t2j
dt Real
2 3 Concave
2t 3t
v= i j ....(2) 1
2 3 m=
2
p = f.v. Real
= 2t3 + 3t5 w Concave
m =+2
40. A
XL = wL = 340 × 20 × 10–3 virtual
= 68 × 10–1 Convex
1
= 6.8  m=
2
1 1 virtual
XC  
C 340  50  106 45. B
10000    tan  
  58.8 rg 
170 v= 
 1   tan  
2
 Z 402  58.8  6.8
2
 402  52  = 65.6
Page # 76 PREVIOUS YEAR PAPER [NEET - 2016]

CHEMISTRY
1. B 10. B
My. S = K sp = 6.2  1013 = 6.2  1013
2. A
S1 = 8 × 10–7
H – C  C —H 
NaNH2 / liNH3
 H — C  C — Na NY3 = Ksp = S(35)3 = 2754 = 62 × 10–14
S 2  10–3.5
S2 > S 1
Solubility or Ny3 is greater than my
CH 3 — CH 2 —Br
11. C
Peptide b ends
CH — C  C — CH2 — CH3 O
1 – Butyne
—C—NH—
CH3 — CH2 – C  C — H 
NaNH2 / liqNH3
 CH3
— CH2— C  C — Na Peptide bond

12. A
Natural rubbar  cis configuration
CH 3 — CH 2 —Br 13. C
14. D
Green Cr2(SO4)3 is formed
CH3 — CH2 — C  C — CH2 — CH2 15. D
Eu: [xe] 4f7 6s2
3 – Hexyne Gd : [xe] 4f7 5d1 6s2
Tb : [xe] 4f9 6s2
3. B
16. D
4. A
same orbital (e–)
0.04
log
t10 0.03
t50% = log 100
50 are disting usished by
Spin Q.NO.
10 log 4  log3 0.60  0.48 0.12 1
t1 / 2 = log2
=
0.3
=
0.30
=
3 17. A
Cu + Con. HNO3 Cu(NO)3 + NO2
10  30 300
t1/2 = = = 25 (24.1) sec.
12 12 18. A
5. B Acetone as a pratective reagent for diols.
in adsorption G
 19. C
H  () G = H – TS
S  = [(–) – [] = (–) always = (–)]
H < 0 , S > 0
6. B or C
20. B
7. C gmw  Z
d = N  a3
A
8. B
6.94  2
0.530 =
9. D 6  1023  a3
6.94  2
a3 =
C—C=0 C = C— 6  1023  0.53
10
H OH a3 = 4.36 × 10–23 ×
10
Keto form enol form a3 = 43.6 × 10–24
keto – enol tautomerism a = (43.6)1/3 × 10–8 cm
PREVIOUS YEAR PAPER [NEET - 2016] Page # 77
21. B 29. C
Cl3 > Br2 > F2 > I2 [Fe(CO) 4] 2–

22. A 30. A
Novalgin analgeris as well as 2H+ + 2e–  H2
antipyretic (g)
PH2
23. A 0.06
1 ER = ER – log H 2
2  
r
mw
r PH2
0 = 0 –0.03 log 107 2 =0
vH2  
rH2 tH2
32 PH2
rO2 = vO2 = =1
2 1014
tO2
PH2 = 10–14 atm
1
= 2 =4 31. D
vO2

1
VO2 =
8
Ea decreases
v  moles

24. B
slower
KHSO4  K+ HSO4–
32. A
H—O—N=O + H+ HSO4–  r 0.98  10 10

= = 0.54
r 1.81  1010
O 0.414 – 0.732 = NaCl = 6 : 6

H2O +NO2 33. B


Phesphinic acid = H3PO2
If we the KHSO4 than rate decreases. Phasphanic acid H3PO3

25. A 34. A

26. Bonus 35. A


PA PA0x A
27. D yA = P = o

 S PA x A  PBo xB
Liquid 
 Vapour
12.8  0.5
r2 Hv  T2  T1  = 12.8  0.5   3.85  0.5 
log r =
1 2.3RT1T2

Hv 6.4 6.4


log2P = + log K = = = 0.76
RT 6.4  1.92 6.3
YA = 0.76
dlP Hc yB = 0.24
=
dT RT 2

28. A
XeF6 dirtorted octahedral
XeO3 Pyramidal
XeOF4 Sq. pyramidal
XeF4 Sq. Planar
Page # 78 PREVIOUS YEAR PAPER [NEET - 2016]
36. D 41. A
Tb = iKb m
H HH 6.5  1000
H H Tb (i) (0.52)
mw  100
0.52  65
H =
H H 30.58
H H H Tb = 1.1052
H 1
Tb = 101.1
Staggered Eclipred Tb1  101

42. B
37. A
williamon ether synthesis
43. A
Sodium alkoxide 
R—X
 Ether ..
NH2
Alkylamine
38. A
—CH = 0 + H2N — R  —CH=N—R R—NH2
Schiff's base Resonance
not present
Resonance
39. B
POS is not present 44. D
Sucrose
40. B (anomeric part in not free)
45. D
PREVIOUS YEAR PAPER [NEET - 2016] Page # 79

PREVIOUS YEAR 2016 PAPER_PHASE-2


BIOLOGY
1. A foreign DNA and plasmid cut by the same 9. Which of the following is correctly matched?
restriction endonuclease can be joined to form (A) Stratification-Population
a recombinant plasmid using (B) Aerenchyma-Opuntia
(A) ligase (B) Eco Rl (C) Age pyramid-Biome
(C) Taq polymerase (D) polymerase III (D) Parthenium hysterophorus–Threat to
biodiversity
2. Which of the following is not a component of
downstream processing? 10. Red List contains data or information on
(A) Expression (B) Separation (A) marine vertebrates only
(C) Purification (D) Preservation (B) all economically important plants
(C) plants whose products are in interna-
3. Which of the following restriction enzymes
tional trade
produces blunt ends?
(D) threatened species
(A) Hind III (B) Sal I
(C) Eco RV (D) Xho I 11. Which one of the following is wrong for fungi?
(A) They are both unicellular and multicellu-
4. Which kind of therapy was given in 1990 to
lar.
a four-year-old girl with adenosine deami-
nase (ADA) deficiency? (B) They are eukaryotic.

(A) Radiation therapy (B) Gene therapy (C) All fungi possess a purely cellulosic cell wall.

(C) Chemotherapy (D) Immunotherapy (D) They are heterotrophic.

12. Methanogens belong to


5. How many hot spots of biodiversity in the
world have been identified till date by Norman (A) Slime moulds (B) Eubacteria
Myers? (C) Archaebacteria (D) Dinoflagellates
(A) 43 (B) 17
(C) 25 (D) 34 13. Select the wrong statement.
(A) Diatoms are microscopic and float pas-
6. The primary producers of the deep-sea hy- sively in water.
drothermal vent ecosystem are (B) The walls of diatoms are easily destructible.
(A) coral reefs (B) green algae (C) 'Diatomaceous earth' is formed by the
(C) chemosynthetic bacteria cell walls of diatoms.
(D) blue-green algae (D) Diatoms are chief producers in the oceans.

7. Which of the following is correct for r-se- 14. The label of a herbarium sheet does not carry
lected species? information on
(A) Small number of progeny with large size (A) height of the plant
(B) Large number of progeny with small size (B) date of collection
(C) Large number of progeny with large size (C) name of collector
(D) Small number of progeny with small size (D) local names

15. Conifers are adapted to tolerate extreme


8. If '+' sign is assigned to beneficial interac-
tion, '–' sign to detrimental and '0' sign to environmental conditions because of
neutral interaction, then the population in- (A) presence of vessels
teraction represented by '+' '–' refers to (B) broad hardy leaves
(A) parasitism (B) mutualism (C) superficial stomata
(C) amensalism (D) commensalism (D) thick cuticle
Page # 80 PREVIOUS YEAR PAPER [NEET - 2016]
16. Which one of the following statements is wrong? 24. Select the mismatch .
(A) Laminaria and Sargassum are used as (A) Methanogens-Prokaryotes
food (B) Gas vacuoles-Green bacteria
(B) Algae increase the level of dissolved (C) Large central vacuoles-Animal cells
oxygen in the immediate environment. (D) Protists-Eukaryotes
(C) Algin is obtained from red algae, and
carrageenan from brown algae. 25. Select the wrong statement.
(D) Agar-agar is obtained from Gelidium and (A) Mycoplasma is a wall-less microorganism.
Gracilaria. (B) Bacterial cell wall is made up of
peptidoglycan.
17. The term 'polyadelphous' is related to (C) Pili and fimbriae are mainly involved in
(A) calyx (B) gynoecium motility of bacterial cells.
(C) androecium (D) corolla (D) Cyanobacteria lack flagellated cells.

18. How many plants among Indigofera, 26. A cell organelle containing hydrolytic enzymes is
Sesbania, Salvia, Allium, Aloe, mustard,
(A) mesosome (B) lysosome
groundnut, radish, gram and turnip have sta-
(C) microsome (D) ribosome
mens with different lengths in their flowers?
(A) Six (B) Three 27. During cell growth, DNA synthesis takes place in
(C) Four (D) Five
(A) M phase (B) S phase
(C) G1 phase (D) G2 phase
19. Radial symmetry is found in the flowers of
(A) Cassia (B) Brassica 28. Which of the following biomolecules is com-
(C) Trifolium (D) Pisum mon to respiration-mediated breakdown of
fats, carbohydrates and proteins?
20. Free-central placentation is found in
(A) Acetyl CoA
(A) Citrus (B) Dianthus (B) Glucose-6-phosphate
(C) Argemone (D) Brassica
(C) Fructose 1,6-bisphosphate
21. Cortex is the region found between (D) Pyruvic acid

(A) endodermis and vascular bundle


29. A few drops of sap were collected by cut-
(B) epidermis and stele
ting across a plant stem by a suitable method.
(C) pericycle and endodermis The sap was tested chemically. Which one
(D) endodermis and pith of the following test results indicates that it
is phloem sap?
22. The balloon-shaped structures called tyloses
(A) Absence of sugar (B) Acidic
(A) are linked to the ascent of sap through
xylem vessels (C) Alkaline (D) Low refractive index

(B) originate in the lumen of vessels


30. You are given a tissue with its potential for
(C) characterize the sapwood differentiation in an artificial culture. Which of
(D) are extensions of xylem parenchyma cells the following pairs of hormones would you add
into vessels to the medium to secure shoots as well as roots?
(A) Gibberellin and abscisic acid
23. A non-proteinaceous enzyme is
(B) IAA and gibberellin
(A) deoxyribonuclease
(C) Auxin and cytokinin
(B) lysozyme
(D) Auxin and abscisic acid
(C) ribozyme (D) ligase
PREVIOUS YEAR PAPER [NEET - 2016] Page # 81
31. Phytochrome is a Codes:
(A) chromoprotein (B) flavoprotein a b c d
(C) glycoprotein (D) lipoprotein (1 ) (iii) (i) (iv) (ii)
(B) (iv) (iii) (i) (ii)
32. Which is essential for the growth of root tip? (C) (ii) (i) (iv) (iii)
(A) Mn (B) Zn (D) (i) (ii) (iv) (iii)
(C) Fe (D) Ca
37. In majority of angiosperms

33. The process which makes major difference (A) a small central cell is present in the
embryo sac
between C3 and C4 plants is
(B) egg has a filiform apparatus
(A) respiration (B) glycolysis
(C) there are numerous antipodal cells
(C) Calvin cycle (D) photorespiration
(D) reduction division occurs in the
megaspore mother cells
34. Which one of the following statements is not
correct? 38. Pollination in water hyacinth and water lily is
(A) Water hyacinth, growing in the standing brought about by the agency of
water, drains oxygen from water that leads (A) bats (B) water
(C) insects or wind (D) birds
to the death of fishes.
(B) Offspring produced by the asexual re- 39. The ovule of an angiosperm is technically
production are called clone. equivalent to
(C) Microscopic, motile a sexual reproduc- (A) megaspore (B) megasporangium
tive structures are called zoospores. (C) megasporophyll
(D) In potato, banana and ginger, the plant- (D) megaspore mother cell
lets arise from the internodes present in the
40. Taylor conducted the experiments to prove
modified stem. semiconservative mode of chromosome rep-
lication on
35. Which one of the following generates new (A) E. coli (B) Vinca rosea
genetic combinations leading to variation?
(C) Vida faba
(A) Nucellar polyembryony (D) Drosophila melanogaster
(B) Vegetative reproduction
41. The mechanism that causes a gene to move
(C) Parthenogenesis
from one linkage group to another is called
(D) Sexual reproduction
(A) crossing-over (B) inversion
(C) duplication (D) translocation
36. Match Column-I with Column-II and select the
correct option using the codes given below : 42. The equivalent of a structural gene is
Column-I Column-II (A) recon (B) muton
a. Pistils fused (i) Gametogenesis (C) cistron (D) operon

together
43. A true breeding plant is
b. Formation of (ii) Pistillate (A) always homozygous recessive in its genetic
gametes constitution
c. Hyphae of higher (iii) Syncarpous (B) one that is able to breed on its own
Ascomycetes (C) produced due to cross-pollination among
unrelated plants
d. Unisexual female (iv) Dikaryotic
(D) near homozygous and produces offspring
flower of its own kind
Page # 82 PREVIOUS YEAR PAPER [NEET - 2016]
44. Which of the following rRNAs acts as struc- 50. Among the following edible fishes, which one
tural RNA as well as ribozyme in bacteria? is a marine fish having rich source of omega-
(A) 5·8 S rRNA (B) 5 S rRNA 3 fatty acids?
(C) 18 S rRNA (D) 23 S rRNA (A) Mackerel (B) Myst.us
(C) Mangur (D) Mrigala
45. Stirred-tank bioreactors have been designed for
(A) ensuring anaerobic conditions in the 51. Match Column-I with Column-ll and select the
culture vessel correct option using the codes given below :
(B) purification of product Column-l Column-II
(C) addition of preservatives to the product a. Citric acid (i) Trichoderma
(D) availability of oxygen throughout the process b. Cyclosporin A (ii) Clostridium
c. Statins (iii) Aspergillus
46. A molecule that can act as a genetic material d. Butyric acid (iv) Monascus
must fulfill the traits given below, except
Codes:
(A) it should provide the scope for slow
a b c d
changes that are required for evolution
(A) (iii) (iv) (i) (ii)
(B) it should be able to express itself in the
form of 'Mendelian characters' (B) (iii) (i) (ii) (iv)

(C) it should be able to generate its replica (C) (iii) (i) (iv) (ii)

(D) it should be unstable structurally and (D) (i) (iv) (ii) (iii)
chemically
52. Biochemical Oxygen Demand (BOD) may not
47. DNA-dependent RNA polymerase catalyzes be a good index for pollution for water bod-
transcription on one strand of the DNA which ies receiving effluents from
is called the (A) sugar industry (B) domestic sewage
(A) antistrand (B) template strand (C) dairy industry (D) petroleum industry
(C) coding strand (D) alpha strand
53. The principle of competitive exclusion was
48. lnterspecilic hybridization is the mating of stated by

(A) more closely related individuals within (A) Verhulst and Pearl (B) C. Danvin
same breed for 4-6 generations (C) G. F. Gause (D) MacArthur
(B) animals within same breed without hav-
54. Which of the following National Parks is home
ing common ancestors
to the famous musk deer or hangul?
(C) two different related species
(A) Dachigam National Park, Jammu & Kashmir
(D) superior males and females of different
breeds (B) Keibul Lamjao National Park, Manipur
(C) Bandhavgarh National Park, Madhya Pradesh
49. Which of t.he following is correct regarding (D) Eaglenest Wildlife Sanctuary, Arunachal
AIDS causative agent HIV? Pradesh
(A) HIV does not escape but attacks the
acquired immune response. 55. A lake which is rich in organic waste may
(B) HIV is enveloped virus containing one result in
molecule of single-stranded RNA and one (A) mortality of fish due to lack of oxygen
molecule of reverse transcriptase. (B) increased popu lation of aquatic organ-
(C) HIV is enveloped virus that contains two isms due to minerals
identical molecules of single-stranded RNA (C) drying of the lake due to algal bloom
and two molecules of revers transcriptase. (D) increased population of fish due to lots
(D) HIV is unenveloped retrovirus. of nutrients
PREVIOUS YEAR PAPER [NEET - 2016] Page # 83
56. The highest DDT concentration in aquatic 61. In male cockroaches, sperms are stored in
food chain shall occur in which part of the reproductive system?
(A) eel (B) phytoplankton (A) Vas deferens (B) Seminal vesicles
(C) seagull (D) crab (C) Mushroom glands (D) Testes

57. Which of the following sets of diseases is 62. Smooth muscles are
caused by bacteria? (A) voluntary, spindle-shaped, uninucleate
(A) Herpes and influenza
(B) involuntary, fusiform, non-striated
(B) Cholera and tetanus
(C) voluntary, multinucleate, cylindrical
(C) Typhoid and smallpox
(D) involuntary, cylindrical, striated
(D) Tetanus and mumps
63. Oxidative phosphorylation is
58. Match Column-I with Column-II for h ousefly
(A) formation of ATP by energy released from
classification and select the correct option
electrons removed during substrate oxidation
using the codes given below :
(B) formation of ATP by transfer of phos-
Column-I Column-II
phate group from a sUbstrate to ADP
a. Family (i) Diptera
(C) oxidation of phosphate group in ATP
b. Order (ii) Arthropoda
(D) addition of phosphate group to ATP
c Class (iii) Muscidae
d. Phylum (iv) Insecta 64. Which of the following is the least likely to
Codes : be involved in stabilizing the three-dimen-
a b c d sional folding of most proteins?
(A) (iv) (ii) (i) (iii) (A) Ester bonds (B) Hydrogen bonds
(B) (iii) (i) (iv) (ii) (C) Electrostatic interaction
(C) (iii) (ii) (iv) (i) (D) Hydrophobic interaction
(D) (iv) (iii) (ii) (i)
65. Which of the following describes graph
59. Choose the correct statement. correctly?

(A) All Pisces have gills covered by an operculum.


(B) All mammals are viviparous.
(C) All cyclostomes do not possess jaws and B
Potential energy

paired fins.
(D) All reptiles have a three-chambered heart. A

60. Study the four statements (A-D) given below Substrate


and select the two correct ones out of them :
A. Definition of biological species was given
by Ernst Mayr. Product
B. Photoperiod does not affect reproduction Reaction
in plants. (A) Exothermic reaction with energy A in ab-
C. Binomial nomenclature system was given sence of enzyme and B in presence of enzyme
by R. H. Whittaker. (B) Endothermic reaction with energy A in pres-
D. In unicellular organisms, reproduction is ence of enzyme and B in absence of enzyme
synonymous with growth. (C) Exothermic reaction with energy A in pres-
The two correct statements are ence of enzyme and B in absence of enzyme
(A) A and B (B) Band C (D) Endothermic reaction with energy A in ab-
(C) C and D (D) A and D sence of enzyme and B in presence of enzyme
Page # 84 PREVIOUS YEAR PAPER [NEET - 2016]
66. When cell has stalled DNA replication fork, 71. Graves' disease is caused due to
which checkpoint should be predominantly (A) hypersecretion of adrenal gland
activated? (B) hyposecretion of thyroid gland
(A) Both G2 / M and M (B) G1/S (C) hypersecretion of thyroid gland
(C) G2/M (D) M (D) hyposecretion of adrenal gland

67. Match the stages of meiosis in Column-I to 72. Name the ion responsible for unmasking of
their characteristic featu res in Column-ll and active sites for myosin for cross-bridge
select the correct option using the codes activity during muscle contraction.
given below : (A) Potassium (B) Calcium
Column-I Column-II (C) Magnesium (D) Sodium
a. Pachytene (i) Pairing of homologous
73. Name the blood cells, whose reduction in
chromosomes
number can cause clotting disorder, leading
b. Metaphase I (ii) Terminalization of
to excessive loss of blood from the body.
chiasmata
(A) Thrombocytes (B) Erythrocytes
c. Diakinesis (iii) Crossing-over (C) Leucocytes (D) Neutrophils
takes place
d. Zygotene (iv) Chromosomes align 74. Name a peptide hormone which acts mainly
at equatorial plate on hepatocytes, adipocytes and enhances
cellular glucose uptake and utilization.
Codes: a b c d
(A) Gastrin (B) Insulin
(A) (iv) (iii) (ii) (i)
(C) Glucagon (D) Secretin
(B) (iii) (iv) (ii) (i)
(C) (i) (iv) (ii) (iii) 75. Osteoporosis, an age-related disease of
skeletal system, may occur due to
(D) (ii) (iv) (iii) (i)
(A) accumulation of uric acid leading to
68. Which hormones do stimulate the production inflammation of joints
of pancreatic juice and bicarbonate? (B) immune disorder affecting neuromuscular
(A) Insulin and glucagon junction leading to fatigue
(B) Angiotensin and epinephrine (C) high concentration of Ca++ and Na+
(C) Gastrin and insulin (D) decreased level of estrogen
(D) Cholecystokinin and secretin
76. Serum differs from blood in
69. The partial pressure of oxygen in the alveoli (A) lacking antibodies
of the lungs is (B) lacking globulins
(A) less than that of carbon dioxide (C) lacking albumins
(B) equal to that in the blood (D) lacking clotting factors
(C) more than that in the blood
77. Lungs do not collapse between breaths and
(D) less than that in the blood some air always remains in the lungs which
can never be expelled because
70. Choose the correct statement.
(A) pressure in the lungs is higher than the
(A) Receptors do not produce graded potentials. atmospheric pressure
(B) Nociceptors respond to changes in pressure. (B) there is a negative pressure in the lungs
(C) Meissner's corpuscles are thermoreceptors. (C) there is a negative intrapleural pressure
(D) Photoreceptors in the human eye are pulling at the lung walls
depolarized during darkness and become (D) there is a positive intrapleural pressure
hyperpolarized in response to the light stimulus.
PREVIOUS YEAR PAPER [NEET - 2016] Page # 85
78. The posterior pituitary gland is not a 'true' Codes : a b c d
endocrine gland because (A) (i) (iv) (iii) (ii)
(A) it secretes enzymes (B) (iii) (iv) (ii) (i)
(B) it is provided with a duct
(C) (iii) (iv) (i) (ii)
(C) it only stores and releases hormones
(D) (iii) (i) (iv) (ii)
(D) it is under the regulation of hypothalamus
85. Several hormones like hCG, hPL, estrogen,
79. The part of nephron involved in active
progesterone are produced by
reabsorption of sodium is
(A) pituitary (B) ovary
(A) descending limb of Henle's loop
(C) placenta (D) fallopian tube
(B) distal convolu ted tubule
(C) proximal convoluted tubule 86. If a colour-blind man man ies a woman who
(D) Bowman's capsule is homozygous for normal colour vision , the
probability of their son being colour-blind is
80. Which of the following is hormone-releasing IUD?
(A) 1 (B) 0
(A) Cu7 (B) LNG-20
(C) 0.5 (D) 0.75
(C) Multiload 375 (D) Lippes loop
87. Genetic drift operates in
81. Which of the following is incorrect regarding
(A) slow reproductive population
vasectomy?
(A) Irreversible sterility (B) small isolated population
(B) No sperm occurs in seminal fluid (C) large isolated population
(C) No sperm occurs in epididymis (D) non-reproductive population
(D) Vasa deferentia is cut and tied
88. In Hardy-Weinberg equation, the frequency
82. Embryo with more than 16 blastomeres formed of heterozygous individual is represented by
due to in uitro fertilization is transferred into (A) q2 (B) p2
(A) cervix (B) uterus (C) 2pq (D) pq
(C) fallopian tube (D) fimbriae
89. The chronological order of human evolution
83. Which of the following depicts the correct from early to the recent is
pathway of transport of sperms? (A) Australopithecus  Homo habilis 
(A) Efferent ductules  Rete testis  Vas Ramapithecus  Homo erectus
deferens  Epididymis (B) Austraiopithecus  Ramapithecus 
(B) Rete testis  Efferent ductules  Homo habilis  Homo erectus
Epididymis  Vas deferens (C) Ramapithecus  Austraiopithecus 
(C) Rete testis  Epididymis  Efferent Homo habilis  Homo erectus
ductules  Vas deferens
(D) Ramapithecus  Homo habilis 
(D) Rete testis  Vas deferens  Efferent Australopithecus  Homo erectus
ductules  Epididymis
90. Which of the following is the correct
84. Match Column-I with Column-II and select the sequence of events in the origin of life?
correct option using the codes given below:
I. Formation of protobionts
Column-I Column-II
II. Synthesis of organic monomers
a. Mons pubis (i) Embryo formation
III. Synthesis of organic polymers
b. Antrum (ii) Sperm
IV. Fonnation of DNA-based genetic systems
c. Trophectoderm (iii) Female external
(A) II, III, IV, I (B) I, II, III, IV
genitalia
(C) I, III, II, IV (D) II, III, I, IV
d. Nebenkern (iv) Graafian follicle
Page # 86 PREVIOUS YEAR PAPER [NEET - 2016]

PHYSICS
1. A person can see clearly objects only when 6. The half - life of a radioactive substance is
they lie between 50 cm and 400 cm from his 30 minutes. The time ( in minutes) taken
eyes. In order to increase the maximum
between 40 % decay and 85% decay of the
distance of distinct vision to infinity, the type
and power of the correcting lens, the person same radioactive substance is
has to use , will be (A) 60 (B) 15
(A) convex, + 0.15 diopter (C) 30 (D) 45
(B) convex, + 2.25 diopter
(C) concave, – 0.25 diopter 7. For CE transistor amplifier. The audio signal
(C) concave, – 0.2 diopter voltage across the collector resistance of 2k
2. A linear aperture whose width is 0.02 cm is is 4v. if the current amplification factor of the
placed immediately in front of a lens of focal transistor is 100 and the base resistance is 1
length 60 cm. The apperature is illuminated k, then the input signal voltage is
normally by a parallel beam of wavelength (A) 15 mV (B) 10 mV
5 × 10–5 cm. The distance of the first dark (C) 20 mV (D) 30 mV
band of the diffraction pattern from the
centre of the screen is 8. The given circuit has two ideal diodes
(A) 0.15 (B) 0.10
connected as shown in the figure below.
(C) 0.25 (D) 0.20
The current flowing through the resistance
3. Electron of mass m with de - Broglie R1 will be
2
wavelength  fall on the target in an X -ray
tube. The cutoff wavelength ( 0 ) of the R1
emitted X - ray is D1 D2

(A) 0   0 10V
3 2
2
2mc
(B)  0 
h
2h (A) 3.13 A (B) 2.5 A
(C)  0 
mc (C) 10.0 A (D) 1.43 A
2m2 c2 2
(D) 0  9. What is the output Y in the following circuit
h2
when all the three inputs A, B,C are first 0 and
then 1 ?
4. Photons with energy 5 eV are incident on a
cathode C in a photoelectric cell. the maximum A
P
energy of emitted photoelectrons is 2 eV. B Q Y
When photons of energy 6 eV are incident on C
C, no photoelectrons will reach the anode A,
if the stopping potential of A relative to C is (A) 1 , 1 (B) 0, 1
(A) – 3 v (B) + 3 V (C) 0, 0 (D) 1, 0
(C) + 4 v (D) – 1 V
10. Planck’s constant (h), speed of light in vacuum
5. If an electron in a hydrogen atoms jumps (c) and Newton’s gravitational constant (g)
from the 3 rd orbit to the 2nd orbit, it emits are three fundamental constant, which of the
a photo of wavelength . when it jumps from following combinations of these has the
he 4th orbit to the 3rd orbit, the corresponding
dimension of length?
wavelength of the photon will be
20 16 Ge hG
(A)  (B)  (A) (B)
13 25 h3 / 2 c3 / 2
9 20
(C)  (D) 
16 7 hG hc
(C) (D)
c5 / 2 G
PREVIOUS YEAR PAPER [NEET - 2016] Page # 87
11. Two cars P and Q start from a point at the 14. A bullet of mass 10 g moving horizontally
same time a straight line and their positions with a velocity of 400 ms–1 strikes a wooden
are represented by xp(t) = at + bt2 and xQ(t) block of mass 2 kg which is suspended by a
= ft – t2. At what time do the cars have the light inextensible string of length 5 m. As a
same velocity ? result, the centre of gravity of the block is
fa a f found to rise a vertical distance of 10 cm.
(A) 2 1  b  (B) The speed of the bullet after it emerges out
1b
horizontally from the block will be
a f a f (A) 160 ms–1 (B) 100 ms–1
(C) 2 b  1 (D) 2 1  b  (C) 80 ms–1 (D) 120 ms–1

12. In the given figure, a = 15 m/s2 represents 15. Two identical balls A and B having velocities
the total acceleration of a particle moving in of 0.5 m/s and – 0.3 m/s respectively collide
the clockwise direction in a circle of radius R elastically in one dimensiion. The velocities of
= 2.5 m at a given instant of time. The speed B and A after the collision respectively will be
of the particle is (A) 0.3 m/s and 0.5 m/s
(B) – 0.5 m/s and 0.3 m/s
(C) 0.5 m/s and – 0.3 m/s
(D) – 0.3 m/s and 0.5 m/s
30°

R
a
O
16. A particle moves from a point 2iˆ  5j
ˆ to  
 4jˆ  3kˆ when a force of  4iˆ  3jˆ N is
(A) 6.2 m/s (B) 4.5 m/s applied. How much work has been done by
(C) 5.0 m/s (D) 5.7 m/s the force ?
(A) 2 J (B) 8 J
13. A rigid ball of mass m strikes a rigid all at 60° (C) 11 J (D) 5 J
and gets reflected without loss of speed as
shown in the figure below. The value of impulse
17. Two rotating bodies A and B of masses m
imparted by the wall on the ball will be
and 2 m with moments of inertial IA and IB(IB
> IA) have equal kinetic energy of rotation.
m
If LA and LB be be their angular momenta
V respectively, then

LB
(A) LA > LB (B) L A 
2
60°
60°
(C) LA = 2LB (D) LB > LA

18. A solid sphere of mass m and radius R is


V
rotating about its diameter. A solid cylinder
of the same mass and same redius is also
rotating about its geometrical axis with an
angullar speed twice that of the sphere .
mV the ratio of their kinetic energies of rotation
(A) (B) mV
3
Esphere
/ Ecylinder  will be
mV
(C) 2 mV (D) (A) 3 : 1 (B) 2 : 3
2
(C) 1 : 5 (D) 1 : 4
Page # 88 PREVIOUS YEAR PAPER [NEET - 2016]
19. A light rod of length l ha s two masses m1 21. A satellite of mass m is orbiting the earth (
and m2 attached to its two ends. The moment or radius R ) at a height h from its surface.
The total energy of the satellite in term of
of inertia of the system about an axis
g0. the value of acceleration due to gravity
perpendicular of the rod and passing through at the earth’s surface, is
the centre of mass is
2mg0R 2 mg0R 2
m1m2 2 (A)  (B) 2 R  h
(A) m1m2 l2 (B) m  m l R h  
1 2
mg0R2 2mg0R 2
(C)  2 R  h (D)
m1  m2 2
l
  R h
(C) m1m2 (D) m1  m2  l2
22. A rectangular film of liquid is extended from (
4cm × 2 cm) to ( 5cm × 4 cm). if the work
20. Starting from the centre of the each having done is 3×10–4 J, the value of the surface
radius R, the variation of g( acceleration due tension of the liquid is
to gravity) is shown by (A) 8.0 N m–1 (B) 0.250 N m–1
(C) 0.125 N m–1 (D) 0.2 N m–1

g 23. Three liquids of densities 1, 2 and 3 (with


1 > 2 >3), having the same value of surface
tension T, rise to the same height in three
identical capillaries. The angles of contance
(A)
1, 2 and 3 obey
O R r 
(A)   1  2  3 
2

(B)  1  2  3  0
2
g

(C) 0  1  2  3 
2
(B) 
(D)  1  2  3  
2
O R r
24. Two identical bodies are made of a material
for which the heat capacity increases with
temperature. One of these is at 100°C, while
the other one is at 0°C. if the two bodies
g
are brought into contact, then, assuming no
heat loss, the final common temperature is
(A) 0° C
(C) (B) 50° C
(C) more than 50° C
O R r (D) less than 50° C but greater than 0°C

25. A body cools from a temperature 3t to 2T in


10 minutes. The room temperature is T.
g
Assume that Newton’s law of cooling is
applicable. the temperature of the body at
the end of next 10 minutes will be
(D) 7
(A) T (B) T
4
O R r
3 4
(C) T (D) T
2 3
PREVIOUS YEAR PAPER [NEET - 2016] Page # 89
26. One mole of an ideal monatomic gas 32. An electric dipole is placed at an angle of 30°
undergoes a process described by the with an electric field intensity 2× 105 N/C. it
equation PV3 = constant. The heat capacity experiences a torque equal to 4 Nm. the charge
of the gas during this process is on the dipole, if the dipole length is 2 cm, is
3 (A) 4C (B) 5 mC
(A) R (B) R
2 (C) 2 mC (D) 5 mC
5
(C) R (D) 2R
2 33. A parallel - plate capacitor of area A, plate
separation d and capacitance C is file with
27. The temperature inside a refrigerator is t2°C
four dielectric materials having dielectric
and the room temperature is t1°C. the amount
of heat delivered to the room for each joule constants k1, k2, k3 and k4 as shown in the
of electrical energy consumed ideally will be figure below. if a single dielectric material is
t1  t2 t1 to be used to have the same capacitance C
(A) t  273 (B) t  t
1 1 2 in the is capacitor. then its dielectric constant
t1  273 t2  273 k is given by
(C) t  t (D) t  t
1 2 1 2
A/3 A/3 A/3

28. A given sample of an ideal gas occupies a


volume V at a pressure P and absolute
temperature T. the mass of each molecule of k1 k2 k3 d/2
the gas is m. Which of the following gives
the density of the gas ?
(A) mKT (B) P/ ( kT) d
(C) Pm / ( kT) (D) P / ( kTV)
k4
29. A body of mass m is attached to the lower
end of a spring whose upper end is fixed. the
spring has negligible mass. When the ma ss m
is slightly pulled down and released, it oscillates A
with a time period of 3s. when the mass m is
increased by 1kg. the time period of oscillations
1 1 1 1 3
becomes 5 s. The value of m in kg is (A) k  k  k  k  2k
1 2 3 4
9 3
(A) (B)
16 4 (B) k = k1 + k2 + k3 + 3k4
4 16
(C) (D) 2
3 9 (C) k  k  k2  k 3   2k 4
3 1
30. The second overtone of an open organ pipe
has the same frequency as the first overtone 2 3 1
of a closed pipe L metre long. the length of (D) k  k  k  k  k
1 2 3 4
the open pipe will be
(A) 4L (B) L
L
(C) 2L (D)
2 34. The potential difference (VA – VB) between

31. Three sound waves of equal amplitudes have the points A and B in the given figure is
frequencies (n – 1), n, ( n + 1). They VA 3V
+ – VB
superimpose to give beats. The number of
beats produced per second will be
(A) 2 (B) 1 (A) +9 V (B) – 3 V
(C) 4 (D) 3 (C) + 3 V (D) + 6 V
Page # 90 PREVIOUS YEAR PAPER [NEET - 2016]
35. A filament bulb ( 500W, 100V) is to be used
in a 230 V main supply. When a resistance R
is connected in series, it works perfectly and
the bulb consumes 500 W. the value of R is r
(A) 13  (B) 230  R R

(C) 46  (D) 26  1 2

36. A long wire carrying a steady current is bent 


into a circular loop of one turn. the magnetic dB 2
(A)  r in loop 1 and zero in loop 2
field at the centre of the loop is B. it is then dt
bent into a circular coil of n turns. the (B) zero in loop 1 and zero in loop 2
 
magnetic field at the centre of this coil of n dB 2 dB 2
(C)  r in loop 1 and  r in loop
turns will be dt dt
(A) 2n2 B (B) nB 
dB 2
(C) n2 B (D) 2nB 2(D)  r in loop 1 and zero in loop 2
dt
37. A bar magnet is hung by a thin cotton thread
41. The potential differences across the
in a uniform horizontal magnetic field and is resistance, capacitance and inductance are
in equilibrium state. The energy required to 80V, 40 V and 100V respectively in an L - C -
rotate it by 60° is W. Now the torque required r circuit . The power factore of this circuit is
to keep the magnet in this new position is (A) 1.0 (B) 0.4
(C) 0.5 (D) 0.8
2W W
(A) (B) 42. A 100  resistance and a capacitor of 100 
3 3
reactance are connected in series across a
220 V source. when the capacitor is 50%
3W charged, the peak value of the displacement
(C) 3W (D)
2 current is
(A) 11 2 A (B) 2.2 A
38. An electron is moving in a circular path under (C) 11 A (D) 4.4 A
the influence of a transverse magnetic field
of 3.57 ×10–2 T. if the value of e/m is 1.76 × 43. Two identical glass (g = 3/2) equiconvex
1011C/kg, the frequency of revolution of the lenses of focal length f each are kept in
contact. The space between the two lenses
electron is
is filled with water (w = 4/3). The focal length
(A) 6.28 MHz (B) 1 GHz of the combination is
(C) 100 MHz (D) 62.8 MHz (A) 3f/ 4 (B) f/3
(C) f (D) 4f /3
39. Which of the following combinations should
be selected for better tunning of an L – C – 44. An air bubble in a glass slab with refractive
index 1.5 ( near normal incidence) is 5 cm
R circuit used for comminution ? deep when viewed from one surface and 3
(A) R = 25 , L = 1.5 H, C = 45 F cm deep when viewed from the opposite face.
(B) R = 20 , L = 1.5 H, C = 35 F the thickness (in cm) of the slab is
(C) R = 25 , L = 2.5 H, C = 45 F (A) 16 (B) 8
(D) R = 15 , L = 3.5 H, C = 30 F (C) 10 (D) 12

45. The interference pattern is obtained with two


40. A uniform magnetic field is restricted within coherent light sources of intensity ratio n. In
a region of radius r. the magnetic field
Imax  Imin
 the interference pattern, the ratio I
dB max  Imin
changes with time at a rate . Loop 1 of
dt will be
radius R > r encloses the region r and loop 2 2 n n
of radius R is outside the region of magnetic (A) 2 (B)
n  1 n 1
field as shown in the figure below. then the
e.m.f generated is 2 n n
(C) (D) 2
n1 n  1
PREVIOUS YEAR PAPER [NEET - 2016] Page # 91

CHEMISTRY
1. A given nitrogen-containing aromatic 5. The correct order of strengths of the
compound A reacts with Sn/HCl, followed by carboxylic acids
HNO2 to give an unstable compound B. B, on
treatment with phenol, forms a beautiful
coloured compound C with the molecular is
formula C 12 H 10 N 2 O. The structure of
compound A is (A) II > I > III (B) I > II > III
(C) II > III > I (D) III > II > I
(A) (B) 6. The compound that will react most readily
with gaseous bromine has the formula
(A) C2H4 (B) C3H6
(C) C2H2 (D) C4H10
(C) (D)
7. Which one of the following compounds shows
the presence of intramolecular hydrogen bond ?
2. Consider the reaction (A) Concentrated acetic acid
CH3CH2CH2Br + NaCN  CH3CH2CH2CN + NaBr (B) H2O2
This reaction will be the fastest in (C) HCN
(A) water (D) Cellulose
(B) ethanol 8. The molar conductivity of a 0.5 mol/dm3
(C) methanol solution of AgNO3 with electrolytic conductivity
(D) N, N'-dimethylformamide (DMF) of 5.76 × 10–3 S cm–1 at 298 K is
(A) 28.8 S cm2/mol (B) 2.88 S cm2/mol
3. The correct structure of the product A (C) 11.52 S cm2/mol (D) 0.086 S cm2/mol
formed in the reaction
9. The decomposition of phosphine (PH3) on
tungsten at low pressure is a first order
H2  gas,1atmosphere  reaction. It is because the

Pd / carbon, ethanol
A is (A) rate of decomposition is very slow
(B) rate is proportional to the surface
coverage
(C) rate is inversely proportional to the
surface coverage
(A) (C) (D) rate is independent of the surface
coverage

10. The coagulation values in millimoles per litre


of the electrolytes used for the coagulation
of As2S3 are given below :
(C) (D) I. (NaCl) = 52, II. (BaCl2) = 0.69,
III. (MgSO4) = 0.22
The correct order of their coagulating power is
(A) III > I > II (B) I > II > III
4. Which among the given molecules can exhibit (C) II > I > III (D) III > II > I
tautomerism ?
11. During the electrolysis of molten sodium
chloride, the time required to produce 0.10
mole of chlorine gas using a current of 3
amperes is
(A) 330 minutes (B) 55 minutes
(C) 110 minutes (D) 220 minutes

12. How many electrons can fit in the orbital for


(A) Both II and III (B) III only which n = 3 and  = 1 ?
(A) 14 (B) 2
(C) Both I and III (D) Both I and II
(C) 6 (D) 10
Page # 92 PREVIOUS YEAR PAPER [NEET - 2016]
13. For a sample of perfect gas when its pressure 21. The number of electrons delivered at the
is changed isothermally from pi to pf, the cathode during electrolysis by a current of 1
entropy change is given by ampere in 60 seconds is (Charge on electron
= 1.60 × 10–19 C)
 pi   pf  (A) 7.48 × 1023 (B) 6 × 1023
(A) S = RT ln  p  (B) S = nR ln  p  (C) 6 × 1020 (D) 3.75 × 1020
 f  i
22. Boric acid is an acid because its molecule
 pi   pf 
(C) S = nR ln  p  (D) S = nRT in  p  (A) combines with proton from water
 f  i molecule
(B) contains replaceable H+ ion
14. The van’t Hoff factor (i) for a dilute aqueous (C) gives up a proton
solution of the strong electrolyte barium (D) accepts OH– from water releasing proton
hydroxide is 23. AlF3 is soluble in HF only in presence of KF. It
(A) 3 (B)0 is due to the formation of
(C) 1 (D) 2 (A) K[AlF3H] (B) K3[AlF3H3]
(C) K3[AlF6] (D) AlH3
15. The percentage of pyridine (C5H5N) that
forms pyridinium ion (C5H5N+H) in a 0.10 M 24. Zinc can be coated on iron to produce
aqueous pyridine solution (Kb for C5H5N= 1.7 galvanized iron but the reverse is not possible.
× 10–9) is It is because
(A) 1.6% (B) 0.0060% (A) zinc has higher negative electrode
potential than iron
(C) 0.013% (D) 0.77%
(B) zinc is lighter than iron
(C) zinc has lower melting point than iron
16. ln calcium fluoride, having the fluorite
(D) zinc has lower negative electrode
structure, the coordination numbers for
potential than iron
calcium ion (Ca2+) and fluoride ion (F–) are
(A) 4 and 8 (B) 4 and 2 25. The suspension of slaked lime in water is
(C) 6 and 6 (D) 8 and 4 known as
(A) aqueous solution of slaked lime
17. If the E°cell for a given reaction has a negative (B) limewater
value, which of the following gives the correct (C) quicklime D) milk of lime
relationships for the values of G° and Keq ?
(A) G° < 0 ; Keq < 1 (B) G°> 0 ; Keq < 1 26. The hybridizations of atomic orbitals of
(C) G° > 0 ; Keq > 1 (D) G° < 0 ; Keq > 1
nitrogen in NO2 , NO–3 and NH4 respectively
18. Which one of the following is incorrect for are
ideal solution ? (A) sp2, sp and sp3 (B) sp, sp3 and sp2
(C) sp2, sp3 and sp (D) sp, sp2 and sp3
(A) Gmix  0
(B) Hmix  0 27. Which of the following fluoro-compounds is
most likely to behave as a Lewis base?
(C) Umix  0
(A) SiF4 (B) BF4
(D)  P  Pobs  Pcalculated by Rault's law  0 (C) PF3 (D) CF4

19. The solubility of AgCl(s) with solubility 28. Which of the following pairs of ions is
product 1.6 × 10–10 in 0.1 M NaCl solution isoelectronic and isostructural?
would be (A) ClO–3 ,SO2– (B) CO2– –
3 3 ,NO3
(A) Zero (B) 1.26 × 10–5 M
(C) 1.6 × 10–9 M (D) 1.6 × 10–11 M (C) ClO–3 ,CO2–
3
(D) SO2– –
3 ,NO3

20. Suppose the elements X and Y combine to 29. In context with beryllium, which one of the
form two compounds XY2 and X3Y2. When following statements is incorrect?
0.1 mole of XY2 weights 10 g and 0.05 mole (A) Its hydride is electron-deficient and
of X3Y2 weight 9 g, the atomic weights of X polymeric.
and Y are (B) It is rendered passive by nitric acid.
(A) 30 , 20 (B) 40 , 30 (C) It forms Be2C.
(C) 60 , 40 (D) 20 , 30 (D) Its salts rarely hydrolyze.
PREVIOUS YEAR PAPER [NEET - 2016] Page # 93
30. Hot concentrated sulphuric acid is a 38. In which of the following molecules, all atoms
moderately strong oxidizing agent. Which of are coplanar?
the following reactions does not show
oxidizing behaviour?
(A) CaF2 + H2SO4  CaSO4 + 2HF
(B) Cu + 2H2SO4  CuSO4 + SO2 + 2H2O (A) (B)
(C) 3S + 2H28O4  3SO2 + 2H2O
(D) C + 2H2SO4  CO2 + 2SO2 + 2H20

31. Which of the following pairs of d-orbitals will


have electron density along the axes?
(A) dxy , dx2 –y2 (B) dx2 , dxy

(C) dxz, dyz (D) dz2 , dx2 –y2 (C) (D)

32. The correct geometry and hybridization for


XeF4 are 39. Which one of the following structures
(A) square planar, sp3 d2 represents nylon 6, 6 polymer?
(B) octahedral, sp3d2
(C) trigonal bipyramidal, sp3d
(D) planar triangle, sp3d3
(A)
33. Among the following, which one is a wrong
statement?
(A) I3 has bent geometry
H2 H2
(B) PH5 and BiCl5 do not exist C H H
C C
(C) p-d bonds are present in SO2 (B)
(D) SeF4 and CH4 have same shape NH2 CH3 66

34. The correct increasing order of trans-effect


of the following species is H2 H2
C H H
(A) CN– > Br– > C6H–5 > NH3 C C
(C)
(B) NH3 > CN– > Br– > C6H–5
NH 2 NH2 66
(C) CN– > C6H–5 > Br– > NH3
(D) Br– > CN– > NH3 > C6H–5
H2 H2
35. Which one of the following statements C H H
H2 H2 C C
related to lanthanons is incorrect? C H H 6
(A) Ce (+ 4) solutions are widely used as (D) C C CH3 COOH
oxidizing agent in volumetric analysis. NH2 Cl 6
(B) Europium shows +2 oxidation state.
(C) The basicity decreases as the ionic radius
decreases from Pr to Lu. 40. In pyrrole
(D) All the lanthanons are much more reactive
than aluminiulm. 4 3

36. Jahn-Teller effect is not observed in high spin 5 2


complexes of
(A) d9 (B) d7 ··
(C) d8 (D) d4 N1
H
37. Which of the following can be used as the The electron density is maximum on
halide component for Friedel-Crafts reaction? (A) 2 and 5 (B) 2 and 3
(A) Isopropyl chloride (B) Chlorobenzene
(C) 3 and 4 (D) 2 and 4
(C) Bromobenzene (D) Chloroethene
Page # 94 PREVIOUS YEAR PAPER [NEET - 2016]
41. Which of the following compounds shall not 44. The correct corresponding order of names of
produce propene by reaction with HBr four aldoses with configuration given below :
followed by elimination or direct only
elimination reaction?
H 2C CH 2
H2
(A) H C—C—CH (B) C
3 2Br
H2
respectively, is
H2 (A) D-erythrose, D-threose, L-erythrose, L-
(C) H C—C—CH (D) H2C C O
3 2OH threose
(B) L-erythrose, L-threose, L-erythrose, D-
42. Which one of the following nitro-compounds threose
does not react with nitrous acid? (C) D-threose, D-erythrose, L-threose, L-
CH3 erythrose
H2 (D) L-erythrose, L-threose, D-erythrose, D-
H3 C C H3 C C threose
(A) H NO (B) C NO2
2
H2 45. In the given reaction
O

H2
H3 C C
CH NO2
(C) the product p is
H3C

H 3C
H3 C C NO2
(D)
H3C (A) (B)

43. The central dogma of molecular genetics


states that the genetic information flows from
(A) DNA  RNA  Carbohydrates
(B) Amino acids  Proteins  DNA
(C) DNA  Carbohydrates  Proteins
(D) DNA  RNA  Proteins (C) (D)
PREVIOUS YEAR PAPER [NEET - 2016] Page # 95

ANSWERKEY
BIOLOGY
1. A 2. A 3. C 4. B 5. D 6. C 7. B
8. A 9. D 10. D 11. C 12. C 13. B 14. A
15. D 16. C 17. C 18. C 19. B 20. B 21. B
22. D 23. C 24. C 25. C 26. B 27. B 28. A
29. C 30. C 31. A 32. D 33. D 34. D 35. D
36. A 37. D 38. C 39. B 40. C 41. D 42. C
43. D 44. D 45. D 46. A 47. B 48. C 49. C
50. A 51. C 52. D 53. C 54. A 55. A 56. C
57. B 58. B 59. C 60. D 61. B 62. B 63. A
64. A 65. C 66. B 67. B 68. D 69. C 70. D
71. C 72. B 73. A 74. B 75. D 76. D 77. C
78. C 79. C 80. B 81. C 82. B 83. B 84. C
85. C 86. B 87. B 88. C 89. C 90. D

PHYSICS

1. C 2. A 3. B 4. A 5. D 6. A 7. C

8. B 9. D 10. B 11. A 12. D 13. B 14. D


15. C 16. D 17. D 18. C 19. B 20. C 21. C
22. C 23. C 24. C 25. C 26. A 27. C 28. C
29. A 30. C 31. A 32. C 33. D 34. A 35. D

36. C 37. C 38. B 39. D 40. A 41. D 42. B


43. A 44. D 45. C

CHEMISTRY

1. C 2. D 3. C 4. B 5. C 6. B 7. D

8. C 9. B 10. D 11. C 12. B 13. C 14. A

15. C 16. D 17. B 18. A 19. C 20. B 21. D

22. D 23. C 24. A 25. D 26. D 27. C 28. A

29. D 30. B 31. D 32. B 33. D 34. C 35. D

36. C 37. A 38. B 39. A 40. A 41. D 42. D

43. D 44. A 45. D


Page # 96 PREVIOUS YEAR PAPER [NEET - 2016]

SOLUTION
PHYSICS
1. C
For object at  if image formed at 400 cm, 1 1  
the reason can see =R   
'  9 16 
u=–
v = –400 1  7 
1 1 1 = R  16  9 
– = '  
v u f
1 1 ' 5 16  9
–0= = ×
 400 f  36 7
 P = –0.25 D concave
20
= 
2. A 7
d sin  = 
dy 6. A

D N = N0 e–t
D t = t1, N = 0.6 N0 = N0 e t1
y
d .6 = e t1
On solving distance = 0.15
t = t2 N = .15 N0 = N0 e t1
3. B .15 = e t2
Cut off wavelength 0 of x rays
(t2 – t1)
hc
0 = KE ....(1) .6
 e  2 1
 t t

.15
For de-broglle wavelength
h 4  e(t2 t1 )
 = 2m(KE) take n Both sides
h2 n(4)
 KE = put value in equation (1)  t2  t1
2m2 
hc h2
= 2 n(2)
0 2m2 t 2 – t1 =

2m2c
0 =
h n(2)
 TH  30

4. A
 t2 – t1 = 2 (30) = 60 min
(I) Ei = w +KE
5 = w +2 ev
w = 3ev 7. C
(II) 6 = 3 + KE IC 4
KE = 3ev
IB = 100 I0 =
2000
=–3v
5. D 2  103
= 100 = 2 × 10–3
1 IB
1 1
= R  2  2  IB = 2 × 10–5
 n
 1 n2
VB
–5
1 1 1 RB = 2 × 10
= R 4  9
   VB
= 2 × 10–5
1000
1  5  VB = 2 × 10–2
=R  
  36  = 20 mA
PREVIOUS YEAR PAPER [NEET - 2016] Page # 97
8. B 13. B
D1  R.B. mvcos60°
D2  F.B.
60°
10
I= = 2.5 A mvsin60° 60°
4
60°
9. D
mvcos60°
A AB
P
B mvsin60°
Q y dp = 2 mv cos 60°
C
C = mv
y = (ABC)
14. D
= ABC
at first y = 1 10  400 10
=2V+ × V’
then y = 0 1000 1000
4 = 2V + 0.01 V’
10. B 1
M0L0T0 = hxCyGz 2 V2 = 2 gh
2
x=z 2  9.8  0 .1
V2 =
M1L2 T 1M1L3 T 2 100
V = 1.4
L3 / 2 T 3 / 2
4 = 2 × 1.4 + 0.01 V’
L5 / 2 T 3 / 2 1.2 = 0.01 V’
=
L3 / 2 T 3 / 2
=L 1. 2
V’ =
0.01
11. A = 120 m/sec
VP = a + 2bt
Va = f – 2t 15. C
a + 2bt = f – 2t
As the collision is elastic in nature, the
f a velocities are interchanged.
=t
2b  2
0.5 m/s 0.3 m/s
12. D
 A  0.3 m/s
a2r  a2t  225
B  0.5 m/s
at
tan30  a
ar at 16. D
ar  at 3 ar 
r1 =  2î  5 ĵ
ar2 
a2r   225 r2 = 4 ĵ  3k̂
3

4 2
a  225 F = 4 î  3 ĵ
3 r   
 r = r2  r1
3
a2r  225  = 4 ĵ  3k̂ – (2 î  5 ĵ)
4
15 = 2 î  ĵ  3k̂
ar  3
2

w = F. r
v2 15 3

2.5 2 = (4î  3 ĵ) . (2 î  ĵ  3k̂ )
15  2.5  3 =8–3
v2  = 5.7 =5J
2
Page # 98 PREVIOUS YEAR PAPER [NEET - 2016]
17. D 21. C
A B TE = KE + PE
m 2m
1 GMm
IB > IA = mv02 –
KA = KB 2 R h

1 1 1 GM GMm
I w 2c = I w 2 = m –
2 A A 2 B B 2 R h R h

 LA 
2
 LB 
2 GMm gR2m
=– =–
IA   = I   2(R  h) 2(R  h)
 IA   IB 
B

IA 22. C
LA
= S = 2TA
LB IB
3 × 10–4 = 2 × T (12 × 10–4)
L I 3 1
T= = = 0.125
24 8
18. C
mr 23. C
2T cos  = hrdg
2 T  same
mr 2
5 r  same
h  same
cos   
mr 2
2 24. C
dQ = msdT
(wC = 2ws)
dQ  same
12
mr 2w2s 1
KS
 25 2 dT 
KC 1 mr ms
.4w2s
2 2 1
KS 1 dT 
s
KC = 5 s for 100°C is more so change in temp.  ,
so final will more than 50°
19. B
25. C

m1 m2 dT
= K (Tav.. – T)
dt
r1 r2
T
= K (2.5 T – T)
I = m1r12 + m2r22 10
m2r m1r T
r1 = m  m r2 = m  m = K 1.5 T
1 2 1 2 10
m1m2 1
2
I= m m r K=
1 2 15

20. C 2 T  T' 1  2 T  T'  T 


=
gr 10 15  2 
gin =
R 6T – 3T’ = T’
Gm g 6T
gs = T’ =
R2 4
gM 3
go = = T
r2 R r
2
PREVIOUS YEAR PAPER [NEET - 2016] Page # 99
26. A 30. C
PV3 = constant 3V 3V
R 3 2l0 = 4lC
C  Cv  CV  R and x = 3 l0 = 2lC, lc = l  l0 = 2L
1x 2

3 R 3 R 31. A
 R  R R
2 13 2 2 max. different n = 2

27. C3 32. C
Room   PE sin 
Q1
 4
Electric P   4  10 5
T 1 =t1 + 273 E sin  5 1
energy W 2  10 
2
P = qd
Q2 P 4  105
q   2  103 = 2 mC
d 2  102
T2 =t2 + 273

Refrigerator 33. D
w = Q1 – Q2
1 1 1
Q1 = Heat delivered to room
ceq. - C1  C2  C3 + C 4
w = Electric Energy
Q1 Q1 T1 t1  273
   1 1 1
w Q1  Q2 T1  T2 t1  t2 = +
C eq. A A A K 4 0 A
K1 0 K 2 0 K 3 0
3  3  3 d
28. C d d d 2
RT 2 2 2
P M = molecular mass
M
M = NAm
RT R C1
 k
NA m NA

kT Pm C2 C4
P  x
m kT y

29. A
m C3
T = 2 =3
R
m 1
2 =5
R d
d
d 2
m 1 5 2
= K eq. 0 A = 0 A K1  K 2  K 3  + K  A
m 3 3 4 0

1 25
1+ =
n 9 1 3 1
k eq. = 2K1  K2  K3  + 2K 4
1 16
m
=
m 9 2 3 1
9 K eq. = K1  K2  K3 + K 4
m=
16
Page # 100 PREVIOUS YEAR PAPER [NEET - 2016]
34. A 38. B
4 + 3 + 2 = V A – VB
V A – VB = 4 + 3 + 2 = 9 2r 2mv
T 
v qBv
35. D
qB
V2  f 
P = 500 w V = 100 v P 2m
R
1
V2 100  100 f=  3.57  102  1.75  1011
Rb    20 2
P 500
x x x x x

P 500 x x x x x
i  5 x x x x x
V 100 x x x x x
x x x x x
x x x x x
Rb x x x x x

3 R
x x x x x

6.28  109
= = 1 GHz
6.28

39. D
230V
Quality factor is greatest when resistance is
minimum.
230
i 5 40. A
Rb  R
230 = 5Rb + 5Rb AdB
Induced emf  e = –
5R = 230 – 5Rb dt
= 230 – 100 for loop 2 emf is zero
130 dB
R   26
5 e = – r
dt

36. C
41. D
 i VR = 80V
B 0 2r = 
2r VC = 40 V
Now same wire in n turns VL = 100 V
r
n 2r2     2r  r2 = Vnet = VR2  (VL  VC )2
n

0ni 0ni  i = (80)2  (60)2


B'    n2  0 
2r2 r   2r 
2
n
= n2B
VL-VC
37. C
1 = 0°
2 = 60° VR
w = PE [cos  – cos 60°]
= 100 volt
 1 PE
= PE 1   = VR
 2 2 R iR
cos  = = = V
Z iZ net
3
T = PE sin 60° = 2w  = 3 w
2 80
= = 0.8
100
PREVIOUS YEAR PAPER [NEET - 2016] Page # 101
42. B 44. D
R = 100
XC = 100
Z= R 2  X2C d1’ = 5 cm d2’ = 3cm
= 100 2 d1 n1
d2 = n2
220 22
Irms = =
100 2 10 2 1. 5 1. 5
d1 = ×5 d2 = ×3
1 1
22
Ipeak = d1 = 7.5 cm d2 = 4.5
2 × 10 2
dnet = d1 + d2 = 7.5 + 4.5 = 12 cm
= 2.2 amp. 45. C
2
43. A Imax .  I1  I2 
 
Let radius of curvature of equiconvex lens Imin . =  I  I 
be R  1 2 

2
 
 1  I2 
1 3  I1  2
1  n 
   
2 =  I2  =  
1  I  1 n 
 1 
1 1 1 1
= f + f + f
f 1 2 3 Imax  Imin (1  n )2  (1  n )2
=
1 1 3  1 1 Imax  Imin (1  n )2  (1  n )2
Given; = f =  2  1  R  R 
f 1    
4 n 2 n
For water part = =
2(1  n) 1n
1 4   1 1
  1   
f2 =  3  R R

1 1   2 
f2 = 3  R 

1 1 1
f3 = f = R

1 1 2 1
= +
f R 3R R
3R 3f
f= =
4 4
Page # 102 PREVIOUS YEAR PAPER [NEET - 2016]

CHEMISTRY
1. C 11. C
gm E Cl2 = F
i t
moles × V.F. =
96500
3 t
0.1 × 2 =
96500

0.1  2  96500
t= min = 107.22  110 min
3  60

12. B

13. C
p
2. D q E  w w nRT loge i
S = = = = p f
SN2 reaction T T T T
Polar aprotic solvent
DMF pi
= n R log p
3. C f

C = O Multiple bond is not affected by pd 14. A


Ba(OH)2  Ba+2 + 2OH–
4. B
i=3
Bridge carbon atom does not take participate
in tauetomerism
15. C
Acidic hydrogen must be present
C5H5N + HOH  C5H5NH+ + OH–
1 0 0
(1 – ) C c c

kb 1.7  109 –4
5. C % =  = = 1.7 × 10
c 0.1
= 1.3 × 10–4
= 0.013 %

16. D
6. B CaF2
C3H6 propene  CnH2n 8:4
more reactive (Nucleophilic strength )
17. B
7. D E°cell = (–)
G°cell = –nf E°cell = 
8. C
0.06
1000 E°cell = log k k<1
m = K × n
M
1000 5.76 576 57.6 18. A
= 5.76 × 10–3 × = = = Gmix = (–)
0.5 0.50 50 5
= 11.52 S cm2/mole 19. C

9. B K sp 1.6  1010
S= = = 1.6 × 10–9
c 0.1
10. D
As2S3 = (–)
20. B
Coagulating power  charge  
1 mole
1 x y2 = 100
coagulation value III > II > I x + 2y = 100 ....(i)
0.05 –––––9gm
PREVIOUS YEAR PAPER [NEET - 2016] Page # 103
39. A
9
1 ––-----  100 =180 n(Hexamethylene diamine) + n(adipic acid)
0.05
3x + 2y = 180 ....(ii)
x  2y = 100 ....(i)
– – –
2x = 80 NYLN–6,6
x = 40 (Condensation polymer)
y = 30
40. A
21. D
Q=Q
i×t=n×e
1 × 60 = n × 1.6 ×10–19
6
n= 41. D
1.6  10 19 H2C C O
60 60 Ketone
= × 10+19 = × 1020 = 3.75 × 1020
1.6 16
42. D
22. D Due to absence of –H
23. C H 3C
24. A H3 C C NO2
25. D H3C
26. D
43. D
27. C DNA  RNA  Proteins
28. A
44. A
29. D
30. B
31. D
32. B
33. D
45. D
34. C
35. D
36. C
37. A
followed by formation of carbocation

Isopropylchloride
(Friedel craft alkylation)
38. B
Due to resonance
Page # 104 PREVIOUS YEAR PAPER [NEET - 2015]

PREVIOUS YEAR 2015 PAPER


BIOLOGY
1. Read the different compunents from (11) to 8. the imperfect fungi which are decomposer of
(d) in the list given below and tell the cor- litter and help in mineral cycling belong to
rect order of the components with reference (A) Deuteromycetes (B) Basidiomycetes
to their arrangement from outer side to inner (C) Phycomycetes (D) Ascomvcetes
side in a woody dicot stem:
(1) Secondary corlex (2) Wood 9. Match the columns and identify the correct
option.
(3) Secondary phloem (4) Phellem
Column-I Column-II
The correct order is :
(1) thylakoids (i) Dis-shaped sacs in
(A) (c), (d), (b), (a) (B) (a), (b), (d), (c)
Golgi apparatus C
(C) (d), (a), (c), (b) (D) (d), (c), (a), (b)
(2) Cristae (ii) condensed structure
of DNA
2. Chrornatophores take part in :
(3) Cisternae (iii) flat membranous
(A) Photosynthesis (B) Growth
sacs in stroma
(C) Movement (D) Respiration
(4) Chromatin (iv) Infoldings

3. Which of the following joints would allow no mitochondriat


movement? (1) (2) (3) (4)

(A) Fibrous joint (B) Cartilaginous joint (A) (iv) (iii) (i) (ii)

(C) Synovial Joint (D) Ball and Socket joint (B) (iii) (iv) (i) ( ii)
(C) (iii) (i) (iv) (ii)
4. The wheat grain has an embryo with one (D) (iii) (iv) (ii) (i)
large, shield-shaped cotyledon known as:
10. Select the wrong statement
(A) Epiblast (B) Coleorrhizil
(A) the viroids were discovered by D.J.
(C) Scutellum (D) Coleoptile
Ivanowski

5. A gene showing codominance has : (B) W.M. Stanley showed that viruses could
be crystallized
(A) one allele dominant on the other
(C) The term 'contagium vivum fludum' was
(B) alleles tightly linked on the same chromosome
conined by M.W. Beijerinek
(C) alleles that are recessive to each other (D) Mosaic disease in tobacco and AIDS in
(S) both alleles independently expressed in the human being are caused by viruses
heterozygote
11. During biological nitrogen fixation inactiva-
6. Which of the .following structures is not tion of nitrogenase by oxygen poisoning is
found in a prokaryotic cell? prevented by

(A) Nuclear envelope (B) Ribosome (A) Leghaemogolobin (B) XanthophyII

(C) Mesosome (D) Plasma membrant? (C) Carotene (D) Cytochrome

12. The species confined lo a particular and not


7. The term "linkage" was coined by:
found elsewhere is termed as:
(A) T.H. Morgan (B) T. Boveri
(A) Keystone (B) Alien
(C) G. Mendel (D) W. Slitton
(C) Fndemi (D) Rare
PREVIOUS YEAR PAPER [NEET - 2015] Page # 105
13. Which Which one of the following hormones 19. Golden rice is a genetically modified crop plant
is not involved in sugar metabolism? where the incorporated gene i's meant for
(A) Cortisone (B) Aldosterone biosynthesis of:
(C) Insulin (D) Glucagon (A) Vitamin B (B) Vitamin C
(C) Omega 3 (D) Vitamin A
14. Which of the following is not a function of
the skeletal system? 20. Outbreeding is an important strategy of ani-
(A) Production of erythrocvtes mal husbandry because it:
(B) Storage of minerals (A) helps in accumulation of superior genes.
(C) Production of body heat (B) is useful in producing purelines of animals.
(D) Locomotion (C) is useful in overcoming inbreeding
depression.
15. Which one of the follow ing is not applicilble
(D) exposes harmful recessive genes that are
to RNA?
eliminated by selection.
(A) Complementary base pairing
(B) 5, phosphoryl and 3' hydroxyl ends 21.  Which one of the following hormones though
(C) Heterocyclic nitrogenous bases synthesised elsewhere, is stored and released
(D) Chargaff's rule by the master gland?
(A) Antidiuretic hormone
16. Which one is a wrong statement? (B) Luteinizing hormone
(A) Archegonia are found in Bryophyta,
(C) Prolactin
Pteridophyta and Gymnosperms
(D) Melanocyte stimulating hormone
(B) Mucor has biflagellate zoospores
(C) Haploid elldosperrn is typical feature of 22.  An association of individuals of different spe-
gymnospemls cies living in the same habital and having
(D) Brown algae have chlorophyll a and c, functional interactions is:
and fucoxanthin (A) Ecological niche (B) Biotic community
(C) Ecosystem (D) Population
17. A childless couple can be assisted to have a
child through a technique called GIFT. The
23.  In which of the following both pairs have
full form of this technique is:
correct combination?
(A) Gamete inseminated fallopian transfer
Gaseous nutrien t cycl e Carbon and Nitrogen
(A)
(B) Gamete intra fallopian transfer Sedimentary n utrient cycle Su lphur and Phosphoru s
(C) Gamete internal fertilization and transfer Gaseous nutrien t cycl e Carbon and sulph ur
(B)
(D) Germ cen internal fallopian transfer Sedimentary n utrient cycle Nitrogen and Phosphours

Gaseous nutrien t cycl e Nitrogen and sulp hus


(C)
18. The wings of a bird and the wings of an in- Sedimentary n utrient cycle Carbon and Phosphoru s
sect are: Gaseous nutrien t cycl e Su lphur and Phosphoru s
(D)
(A) homologous structures and represent di- Sedimentary n utrient cycle Carbon and Nitrogen

vergent evolution
(B) analogous structures and represent con- 24.  Identify the correct order of organisation of
vergent evolution . genetic material from largest to smallest:

(C) phylogenetic structures and represent (A) Chromosome, gene, genome, nucleotide
divergent evolu tion (B) Genome, chromosome, nucleotide, gene
(D) homologous structures and represent (C) Genome, chromosome, gene, nucleotide
convergent evolution (D) Chromosome, genome, nucleotide, gene
Page # 106 PREVIOUS YEAR PAPER [NEET - 2015]
25.  A jawless fish, which lays eggs in fresh 33. Destruction of the anterior horn cells of the
water and whose ammocoetes larvae after spinal cord would result in loss of :
metamorphosis retum to the ocean is: (A) Sensory impulses
(A) Eptatretus (B) Myxine (B) Voluntary motor impulses
(C) Neomyxine (D) Petromyzon
(C) Commissural impulses

26. Industrial melanism is an example of : (D) integrating impulses

(A) Neo Darwinism (B) Natural selection


34. The cutting of DNA at specific locations be-
(C) Mutation (D) Neo Lamarckism come possible with the discovery of :

27. Cell wall is absent in : (A) Restricition enzymes

(A) Aspergillus (B) Funaria (B) Probes

(C) Mycoplasma (D) Nostoc (C) Selectable markers


(D) Ligases
28. The chitinous exoskeleton of arthropods is
formed by the polymerisation of : 35. In the following human pedigree, the filled
(A) Keratin sulphate and chondraitin sulphate symbols represent the affected individuals.
(B) D-glucosamine Identify the type of given pedigree.

(C) N-acetyl glucosamine (i)


(D) lipoglycans
(ii)
29. Filiform apparatus is characteristic feature
of : (iii)
(A) Generative cell (B) Nucller embryo
(iv)
(C) Aleurone cell (D) Synergids
(A) Autosomal dominant
30. In angiosperms, microsprogenesis and me- (B) X-linked resessive
gasporogenesis :
(C) Autosomal resessive
(A) occur in anther
(D) X-linked dominant
(B) form gametes without further divisions
(C) Involve meiosis (D) occur in ovule 36. A colour blind man marries a woman with nor-
mal sight who has no history of colour blind-
31. Metagenesis refers to : ness in her famil. What is the probability of
(A) Presence of different morphic forms their grandson being colour blind ?
(B) Alternation of generation between asexual (A) 0.5 (B) 1
and sexual phases of an organism (C) Nil (D) 0.25
(C) Occurrence of a drastic change in form
during post-embyonic development 37. Flowers are unisexual in :
(D) Presence of a segmented body and par- (A) Pea (B) Cucumber
thenogenetic mode of reproduction
(C) China rose (D) Onion

32. Which of the following immunoglobulins does


38. Roots play insignificant role in absorption of
constitule the largest percentage in human
water in :
milk ?
(A) Sunflower (B) Pistia
(A) lgD (B) lgM
(C) Pea (D) Wheat
(C) lgA (D) lgG
PREVIOUS YEAR PAPER [NEET - 2015] Page # 107
39. Balbiani rings rings are sites of : 45. Which of the following are not membrane-bound?
(A) Lipid synthesis (A) Vacuoles (B) Ribosomes
(B) Nucleotide synthesis (C) Lysosomes (D) Mesosomes
(C) Polysoccharide synthesis
46. In his classic experiments on pea plants,
(D) RNA and protein synthesis
Mendel did not use :

40. Which of the followil'lg pairs .is not correctly (A) Seed colour (B) Pod length
matched? (C) Seed shape (D) Flower position
Mode of reproduction Example
47. During ecological succession :
(A) Offset Water hyacinth
(A) the gradual and predictable change in
(B) Rhizome Banana
species composition occursin a given area.
(C) Binary fission Sargassum (B) the establishment of a new biotic
(D) Conidia Penicillium community is very fast in its primary phase.
(C) the number and types of animals remain
41. Ectopic pregnancies are referred to as :
constant
(A) Pregnancies with genetic abnormality.
(D) the changes lead to a community that is
(B) Implantation of embryo at site other than in near equilibrium with the envirounment and
uterus is called pioneer community
(C) Implantation of defective embryo in the
uterus 48. The body cells in cockroach discharge their
(D) Pregnancies terrninated due to hormonal nitrogenous waste in the haemolymph mainly
imbalance in the form of
(A) Ammonis (B) Potassium urate
42. Choose the wrong statement : (C) Urea (D) Calcium carbonate
(A) Penicillium is multicelluar and produces
antibioties 49. Which of the following biomolecules does have
a phosphodiester bond?
(B) Neurospom is used in the study of
biochemical genetics (A) Fatty acids in a diglyceride

(C) Morels and truffles are poisonous mushrooms (B) Monosaccharides in a polysaccharide

(D) Yeast is unicellular and useful in fermentation (C) Amino acids in a polypeptide
(D) Nucleic acids in a nucleotide
43. The function of the gap junction is to :
50. The UN conference of Parties on climate
(A) performing cementing to keep
change in the year 2012 was held at
neighbouring cells together.
(A) Durban (B) Doha
(B) facilitate communication between adjoin-
ing cells by connecting the cytoplasm for rapid (C) Lima (D) Warsaw
transfer of ions, small molecules and some
51. Arrange the following events of meiosis in
large molecules.
correct sequence:
(C) separate two cells from each other.
(a) Crossing over (b) Synapsis
(D) stop substance from leaking across a tissue.
(c) Terminalisation of chiasmata
44. Axile placentation is present in : (d) Disappearance of nucleous

(A) Dianthus (B) Lemon (A) (b), (a,) (d), (c) (B) (b), (a) (c), (d)

(C) Pea (D) Argemone (C) (a), (b) (c), (d) (D) (b), (c) (d), (a)
Page # 108 PREVIOUS YEAR PAPER [NEET - 2015]
52. Root pressure develops due to: 59. Body having meshwork of cells, internal,
(A) Active absorption cavities lined with food filtering flagellated
cells and indirect development are the
(B) Low osmotic potential in soil
characteristics of phylum:
(C) Passive absorption (A) Coelenterate (B) Porifera
(D) Increase in tranpiration (C) Mollusca (D) Protozoa

53. Which one of the following animals has two 60. The oxygen evolved during photosynthesis
separate circulatory pathways? comes from water molecules. Which one of
(A) Frog (B) Lizard the following pairs of elements is involved in
this reaction?
(C) Whale (D) Shark
(A) manganese and Chlorine
54. Which of the following events is not associated (B) Manganese and Potassium
with ovulation in human female? (C) Magnesium and Molybdenum
(A) Decrease in estradiol (D) Magnesium and Chlorine
(B) Full development of Graafian follicle
61. The primary denition in human differs from
(C) Release of secondary oocyte permanent dentition in not having one of the
(D) LH surge following type of teeth:
(A) Canine (B) Premolars
55. Most animals that live in deep oceanic waters
(C) Molars (D) Incisors
are:
(A) primary consumers 62. Coconut water from a tender coconut is:

(B) secondary consumers (A) Immature embryo


(B) Free nuclear endosperm
(C) tertiary consumers
(C) Innermost layers of the seed coat
(D) detritivores
(D) Degenerated nucellus
56. If you suspect major deficiency of antibodies
63. Which of the following layers in an antral
in a person, to which of the following would
follicle is acellular?
you look for confirmatory evidence?
(A) Granulosa (B) Theca interna
(A) Fibrinogin in plasma
(C) Stroma (D) Zona pellucida
(B) Serum albumins
(C) Haemocytes (D) Serum globulins 64. The introduction of t-DNA into plates involves :
(A) Infection of the plant by agrobacterium
57. The structures that help some becteria to tumefaciens
attach to rock and/or host tissues are: (B) Altering the pH of the soil, then heat-
(A) Rhizoids (B) Fimbriae shocking the plants

(C) Mesosomes (D) Holdfast (C) Exposing the plates to cold for a brief
period
58. Increase in concentration of the toxicant at (D) Allowing the plant roots to stand in water
successive tropic levels is known as:
65. In which group of organisms the cell walls form
(A) Biomagnification (B) Biodeterioration
two thin overlapping shells which fit together?
(C) Biotransformation
(A) Chryosphytes (B) Euglenoids
(D) Biogechemical cycling (C) Dinoflagellates (D) Slime moulds
PREVIOUS YEAR PAPER [NEET - 2015] Page # 109
66. Human urine is usually acidic because : 73. A column of water within xylem vessels of
(A) The sodium transporter exchanges one tall trees does not break under its weight
hydrogen ion for each sodium ion, in because of :
peritubular capillaries. (A) Dissolved sugars in water
(B) Excreted plasma proteins are acidic
(B) Tensile strength of water
(C) potassium and sodium exchange
(C) Lignification of xylem vessels
generates acidity
(D) Positive root pressure
(D) hydrogen ions are actively secreted into
the filtrate.
74. Acid rain is caused by increase in the
67. In photosynthesis, the light-independent atmospheric concentration of :
reactions take place at : (A) SO2 and NO2 (B) SO3 and CO
(A) Thylakoid lumen (B) Photosystem I (C) CO2 and CO (D) O3 and dust
(C) Photosystem II (D) Stromal matrix
75. The enzyme that is not present in succus
68. In mammalian eye, the 'fovea' is the center entericus is :
of the visual field, where :
(A) maltase (B) nucleases
(A) high density of cones occur, but has no
(C) nucleosidase (D) lipase
rods.
(B) the optic nerve leaves the eye. 76. In which of the following interatctions both
(C) only rods are present partners are adversely affected ?
(D) more rods than cones are found. (A) Competition (B) Predation

69. The DNA molecule to which the gene of (C) Parasitism (D) Mutualism
interest is integrated for cloning is called :
77. Match the following list of microbes and their
(A) Transformer (B) Vector
importance :
(C) Template (D) Carrier
Sacharomyces Production of
(a) (i)
cerevisiae immunosuppressive agents
70. Pick up the wrong statement :
Monascus
(b) purpureus (ii)
(A) Cell wall is absent in Animalia Riperuing of Swiss cheese
Trichoderma Commercial production of
(c) polysporum (iii) ethanol
(B) Protista have photosynthetic and
heterotrophic modes of nutrition (d) Propionibacterium (iv) Production of bl ood
sharmanii cholesterol lowering angents
(C) Some fungi are edible
(D) Nuclear membrance is present in Monera (a) (b) (c) (d)
(A) (iii) (iv) (i) (ii)
71. Among china rose, mustard, brinjal, potato,
guava, cucumber, onion and tulip, how many (B) (iv) (iii) (ii) (i)
plates have superior ovary ? (C) (iv) (ii) (i) (iii)
(A) Five (B) Six (D) (iii) (i) (iv) (ii)
(C) Three (D) Four
78. A pleiotropic gene :
72. Name the pulmonary disease in which alveolar
(A) is expressed only in primitive plants.
surface area involved in gas exchange is
drastically reduced due to damage in the alveolar (B) is a gene evolved during Plicene
walls . (C) controls a trait only in combination with
(A) Pleurisy (B) Emphysema another gene
(C) Pneumonia (D) Asthma (D) controls multiple traits in an individual
Page # 110 PREVIOUS YEAR PAPER [NEET - 2015]
79. A protoplast is a cell : 86. Doctors use stethoscope to hear the sounds
(A) Without plasma membrane produced during each cardiac cycle. The
(B) without nucleus second sound is heard when:

(C) undegoing division (A) AB valves open up

(D) without cell wall (B) Ventricular walls vibrate due to gushing
in of blood from atria
80. Which of the following are most suitable (C) Semilunar valves close down after the
indicators of SO2 pollution in the environment? blood flows into vessels from ventricles
(A) Lichens (B) Conifers (D) AV node receives signal from SA node
(C) Algae (D) Fungi
87. Auxin can be bioassayed by:
81. Grafted kindney may be rejected in a patient (A) Avena coeoptile curvature
due to :
(B) Hydroponics
(A) Humoral immune response
(C) Potometer
(B) Cell-mediated immune respone
(D) Lettuce hypocotyl elongation
(C) Passive immune respone
(D) Innate immune respone 88. Satellite DNA is important because it:
(A) codes for proteins needed in cell cycle.
82. Which one of the following fruits is
(B) shows high degree of polymorphism in
parthenocarpic?
population and also the same degree of
(A) Brinjal (B) Apple polymorphism in an individula, which is
(C) Jackfruit (D) Banana heritable from parents to children.
(C) does not code for proteins and is same
83. Which of the following diseases is caused by
in all members of the population.
a protozan ?
(D) codes for enzymes needed for DNA
(A) Syphilis (B) Unfluenza
replaction.
(C) Babesiosis (D) Blastomycosis
89. Cellular organelles with membranes are:
84. In human females, meiosis-II is not completed
(A) nuclei, ribosomes and mitochondria
until ?
(B) chromosomes, ribosomes and endoplasmic
(A) puberty (B) fertilization
(C) endoplasmic reticulum, ribosomes and nuclei
(C) uterine implantation (D) birth
(D) Lysosomes, Golgi apparatus and mitochondria
85. Mele gametophyte in angiosperms produces:
90. Eutrophication of water bodies leading to
(A) Two sperms and a vegetative cell
killing of fishes is mainly due to non-availability
(B) Single sperm and a vegetative cell of:
(C) Single sperm and a two vegetative cells (A) food (B) light
(D) Three sperms (C) essential minerals (D) oxygen
PREVIOUS YEAR PAPER [NEET - 2015] Page # 111

PHYSICS
1. The cylindrical tube of a spray pump has
radius R, one end of which has n fine holes,
each of radius r. If the speed of the liquid in
the tube is V, the speed of the ejection of
the liquid through the holes is -
mg
VR 2 VR 2
(A) (B)
n2r 2 nr 2
(A) 0.6 and 0.6 (B) 0.6 and 0.5
VR 2 V 2R
(C) (D) (C) 0.5 and 0.6 (D) 0.4 and 0.3
n3r 2 nr
5. An ideal gas is compressed to half its initial
2. Point masses m1 and m2 are placed at the volume by means of several processes. Which
opposite ends of a rigid rod of length L, and of the process results in the maimum work
negligible mass. The rod is to be set rotating done on the gas ?
about an axis perpendicular to it. The position (A) Adiabatic (B) Isobaric
of point P on this rod though which the axis (C) Isochoric (D) Isothermal
should pass so that the work required to set
the rod rotating with angular velocity 0 is 6. A ball is thrown vertically downwards from a
height of 20 m with an initial velocity 0. It collides
minimum, is given by :
with the ground, loses 50 percent of its energy
in collision and rebounds to the same height.
The initial velocity 0 is (take g = 10 ms–2)
(A) 14 ms–1 (B) 20 ms–1
m1 P m2
(C) 28 ms–1 (D) 10 ms–1
x (L–x)
7. In the spectrum of hydrogen, the ratio of the
longest wavelength in the Lyman series to the
longest wavelength in the balmer series is -
m1L m1 4 9
(A) x  m  m (B) x  m L (A) (B)
1 2 2
9 4
m2 m2L 27 5
(C) x  m L (D) x  m  m (C) (D)
1 1 2
5 27

3. A proton and an alpha particle both enter a 8. A source of sound S emitting waves of
region of uniform magnetic field B, moving at frequency 100 Hz and an observer O are
right angles to the field B. If the radius of located at some distance from each other.
circular orbits for both the patricles is equal The source is moving with a speed of 19.4
and the kinetic energy acquired by proton is ms–1 at an angle of 60° with the source
1 MeV, the energy acquired by the alpha observer line as s hown in the figure. The
particle will be - observer is at rest. The apparent frequency
(A) 4 MeV (B) 0.5 MeV observed by the observer (velocity of sound
(C) 1.5 MeV (D) 1 MeV in air 330 ms–1) is -

VS
4. A plank with a box on it at one end is gradually
raised about the other end. As the angle of
inclination with the horizontal reaches 30°,
the box starts to slip and slides 4.0 m down 60°
the plank in 4.0 s. The coefficients of static O
and kinetic friction between the box and the (A) 100 Hz (B) 103 Hz
plank with be, respectively - (C) 106 Hz (D) 97 Hz
Page # 112 PREVIOUS YEAR PAPER [NEET - 2015]
9. If dimensions of critical velocity vc of a liquid 14. A remote-sensing satellite of earth revolves
flowing through a tube are expressed as in a circular orbit at a height of 0.25 × 106 m
[xyrz], where , and r are the coefficient above the surface of earth. If earth's radius
of viscosity of liquid, density of liquid and is 6.38 × 106 m and g = 9.8 ms–1, then the
radius of the tube respectively, then the orbital speed of the satellite is -
values of x,y and z are given by - (A) 7.76 kms–1 (B) 8.56 kms–1
(A) 1, –1, –1 (B) –1, –1, 1 (C) 9.13 kms–1 (D) 6.67 kms–1
(C) –1, –1, –1 (D) 1,1,1
15. Two metal wires of identical dimensions are
10. 4.0 g of a gas occupies 22.4 litres at NTP, connected in series. If 1 and 2 are the
The specific heat capacity of the gas at conductivities of the metal wires respectively,
constnat volume is 5.0 JK–1 mol–1. If the speed the effective conductivity of the combination is
of sound in this gas at NTP is 952 ms–1, then
the heat capacity at constant pressure is - 212 1  2
(A) (B)
(Take gas constnat r= 8.3 JK–1 mol–1) 1  2 212
(A) 8.0 JK–1 mol–1 (B) 7.5 JK–1 mol–1
–1
(C) 7.0 JK mol –1 (D) 8.5 JK–1 mol–1 1  2 12
(C) (D)
12 1  2
 
11. If vectors A = cos t î + sin t ĵ and B = 16. A satellite S is moving in an elliptical orbit
t t around the earth. The mass of the satellite
cos î  sin ĵ are functions of time, is very small compared to the mass of the
2 2
then the value of t at which they are earth. Then,
orthogonal to each other is - (A) The angular momentum of S about the
centre of the earth changes in direction, but
 
(A) t = (B) t = its magnitude remains constant
4 2 (B) The total mechanical energy of S varies
 periodically with time.
(C) t = (D) t = 0 (C) The linear momentum of S remains
 constant in magnitude
(D) The acceleration of S is always directed
12. In the given figure, a diode D is connected towards the centre of the earth
to an external resistance R = 100  and an
e.m.f. of 3.5 V. If the barrier potential 17. Two particles A and B, move with constant
developed across the diode is 0.5 V, the  
velocities 1 and 2 . At the initial momentum
current in the circuit will be -
100
 
D their position vectors are r1 and r2 respectively..
R The condition for particles and B for their
collision is -
   
r1  r2 2  1
(A)     
r1  r2 2  1
   
(B) r1.1  r2.2
   
(C) r1  1  r2  2
3.5 v    
(A) 30 mA (B) 40 mA (D) r1 – r2  1  2
(C) 20 mA (D) 35 mA
18. Two stones of masses m and 2m are whirled
13. If potential (in volts) in a region is expressed in horizontal circles, the heavier one in a radius
as V(x,y,z) = 6xy – y + 2yz, the electric field r/2 and the ligher one is radius r. The tangential
(in N/C) at point (1,1,0) is - speed of lighter stone is an times that of the
value of heavier stone when they experience

(A)  3 î  5 ĵ  3k̂  
(B)  6 î  5 ĵ  2k̂  same centripetal forces. The value of n is -
(A) 2 (B) 3
(C)  2 î  3 ĵ  k̂  (D)  6 î  9 ĵ  k̂  (C) 4 (D) 1
PREVIOUS YEAR PAPER [NEET - 2015] Page # 113
19. A parallel plate air capacitor has capacity 'C', 24. A photoelectric surface is illuminated successively
distance of separation between plates is 'd' by monochromatic light of wavelength  and
and potential difference 'V' is applied betwen /2. If the maximum kinetic energy of the
the plates. Force of attraction between the emitted photoelectrons in the second case
plates of the parallel plate air capacitor is - is 3 times that in the firrst case, the work
function of the surface of the material is -
C2 V 2 CV2
(A) (B) hc hc
2d 2d (A) (B)
2 
CV2 C2 V 2
(C) (D)
d 2d 2hc hc
(C) (D)
  3
20. The position vector of a particle R as a
function of time is given by -
 25. In an astronomical telescope in normal
R  4 sin(2t)î  4 cos(2t)î adjustment a straight block line of length L
is drawn on inside part of objective lens. The
Where R is in meters, t is in second and î eye-piece from a real image of this line. The
length of this image is I. The magnification
and ĵ denote unit vectors along x and y of the telescope is :
direction, respectively. Which one of the L L
following statements is wrong for the motion (A) 1 (B) 1
I I
of particle ?
 L I L
(A) Acceleration vector is along  R (C) (D)
L I I
v2
(B) Magnitude of acceleration vector is .
R 26. Two slits in Youngs experiment have width in
(C) Magnitude of the velocity of particle is 8 the ratio 1 : 25. The ratio of the intensity at
meter/second the maxima and minima in the interference
(D) Path of the particle is circle of radius 3
meters. Imax
pattern I is :
min
21. A series R-C circuit is connected to an alternating
voltage source. Consider two situations : 9 121
(A) (B)
(i) When capacitor is air filled 4 49
(ii) When capacitor is mica filled
49 4
Current through resistor is i and voltage across (C) (D)
capacitor is V then : 121 9
(A) Va < Vb (B) Va > Vb
(C) ia > ib (D) Va = Vb 27. Two vessels separately contain two ideal
gases A and B at the same temperature, the
22. A string is stretched between fixed points pressure of A being twice that of B. Under
separated by 75.0 cm. It is observed to have such conditions, the density of A is found to
resonant frequencies of 420 hz and 315 Hz. be 1.5 time sthe denstiyh of B. The ratio of
There are no other resonant frequencies molecular weight of A and B is -
between these two. The lowest resonant (A) 2/3 (B) 3/4
frequency for this string is - (C) 2 (D) 1/2
(A) 155 Hz (B) 205 Hz
(C) 10.5 Hz (D) 105 Hz 28. A circuit contains an ammeter, a battery of
30 V and a resistance 40.8 ohm all connected
23. The coefficient of performance of a
in series. If the ammeter has a coil of
refrigerator is 5. If the temperature inside
resistance 480 ohm and a shunt of 20 ohm,
freezer is –20°C, the temperatures of the
the reading in the ammeter will be -
surroundings to which it rejects heat is -
(A) 0.5 A (B) 0.25 A
(A) 31°C (B) 41°C
(C) 2 A (D) 1 A
(C) 11°C (D) 21°C
Page # 114 PREVIOUS YEAR PAPER [NEET - 2015]
29. The value of coefficient of volume expansion a
of glycerin is 5 × 10–4 K–1. The fractional
change in the density of glycerin for a rise of
a d
40°C in its temperature is -
(A) 0.015 (B) 0.020
(C) 0.025 (D) 0.010 c
X y
electron
30. The heart of a man pumps 5 litres of blood (A) abcd (B) adcb
through the arteries per minute at a pressure (C) The current will reverse its direction as
of 150 mm of mercury. If the density of mercury the electron goes past the coil
be 13.6 × 103 kg/m3 and g = 10 m/s2 then (D) No current induced
the power of heart in watt is -
(A) 1.70 (B) 2.25 34. A nucleus of uranium decays at rest into
(C) 3.0 (D) 1.50 nuclei of thorium and helium. Then :
(A) The helium nucleus has more kinetic
31. A beam of light consisting of red, green and energy
blue colours is incident on a right angled than the thorium nucles.
prism. The refractive index of the material of (B) The helium nucleus has less momentum
the prism for the above red, green and blue then the thorium nucleus.
wavelengths are 1.39, 1.444 and 1.47, (C) The helium nucleus has more momentum
respectively. than the thorium nucleus.
A (D) The helium nucleus has less kinetic energy
than the thorium nucleus.
Blue
 
35. A for F = F  aˆi  3j ˆ is acting at a
ˆ  6k
Green
Red  ˆ The value of  for
point r = 2iˆ  6j
ˆ  12k.
which an gular momentum about origin is
45° conserved is :
(A) –1 (B) 2
B C
The prism will: (C) zero (D) 1
(A) separate the blue colour part from the red
and green colours 36. Water rises to a height ‘h’ in capillary tube.
(B) separate all the three colours from one If
another the length of capillary tube above the surface
(C) not-separate the three colours at all of water is made less than ‘h’, then :
(D) separate the red colour part from the (A) water rises upto the tip of capillary tube
green and blue colours and then starts overflowing like a fountain.
(B) water rises upto the top of capillary tube
and stays there without overflowing.
32. A rectangular coil of length 0.12 m and width
(C) water rises upto a point a little below
0.1 m having 50 turns of wire is suspended
the top and stays there.
vertically in a uniform magnetic field of
(D) water does not rise at all.
strength 0.2 Weber/m2. The coil carries a
current of 2A. If the plane of the coil is inclined
37. A particle is executing a simple harmonic
at an angle of 30° with the direction of the
motion. Its maximum acceleration is  and
filed, the torque required to keep the coil in
maximum velocity is . Then, its time period
stable equilibrium will be :
of vibration will be :
(A) 0.15 Nm (B) 0.20 Nm
(C) 0.24 Nm (D) .12 Nm 2 
(A) (B)
a2 
33. An electron moves on a straight line path XY
as shown. The abcd is a coil adjacent to the 2 2
(C) (D)
path of electron. What will be the direction  
of current, if any, induced in the coil?
PREVIOUS YEAR PAPER [NEET - 2015] Page # 115
38. The energy of the em waves is of the order 43. The Young’s modulus of steel is twice that
of 15 keV. To which part of the spectrum of brass. Two wires of same length and of
does it belong? same
(A) X-rays (B) Infra-red rays area of cross section, one of steel and
(C) Ultraviolet rays (D)  - rays another of brass are suspended from the same
roof. If we want the lower ends of the wires
39. Light of wavelength 500 nm is incident on a to be at the same level,. then the weights
metal with work function 2.28 eV. The de added to the steel and brass wires must be
Broglie wavelength of the emitted electron in the ratio of:
is (A) 1 : 2 (B) 2 : 1
: (C) 4 : 1 (D) 1 : 1
(A) < 2.8 × 10–10m (B) < 2.8 × 10–9m
(C)  2.8  109 m (D)  2.8  1012 m 44. The input signal given to a CE amplifier having

 
40. At the first minimum adjacent to the central a voltage gain of 150 is Vi = 2 cos 15   .
 3
maximum of a single-slit diffraction pattern,
The corresponding output signal will be :
the phase difference between the Huygen’s
wavelet from the edge of the slit and the    2 
(A) 300 cos  15t   (B) 75 cos  15t 
wavelet from the midpoint of the slit is :  3  3 
 
(A) radian (B) radian  5 
4 2 (C) 2 cos  15t  6 
 

(C)  radian (D) radian
8  4 
(D) 300 cos  15t 
 3 
41. On a frictionless surface, a block of mass M.
moving at speed collides elastically with
45. An automobile moves on a road with a speed
another block of same mass M which is initially
of 5 km h-1. The radius of its wheels is 0.45
at rest. After collision the first block moves
m and the moment of inertia of the wheel
at an angle  to its initial direction and has a
about its axis of rotation is 3 kg m2. If the
 vehicle is brought to rest in 15s, the
speed . The second block's speed after magnitude of average torque transmitted by
3
the collision is : its brakes to the wheel is :
(A) 6.66 kg m2 s–2 (B) 8.58 kg m2 s–2
2 2 3 (C) 10.86 kg m2 s–2 (D) 2.86 kg m2 s–2
(A)  (B) 
3 4

3 3
(C)  (D) 
2 2

42. A potentiometer wire of length L and a


resistancer are connected in series with a
battery of e.m.f. E0 and a resistance r1. An
unknown e.m.f. E is balanced at a length l of
the potentiometer wire. The e.m.f. E will be
given by :
LE0r E0r l
(A) l r (B) r  r  L
1 1

E0l LE0r
(C) (D) r  r  l
L 1
Page # 116 PREVIOUS YEAR PAPER [NEET - 2015]

CHEMISTRY
1. In which of the following pairs, both the
species are not isostructural
(A) XeF4, XeO4 (B) SiCl4, PCl4+ (A) (B)
(C) diamond, silicon carbide
(D) NH3, PH3

2. Which one of the following esters gets (C) (D)


hydrolysed most easily under alkaline conditions?
OCOCH3
7. On heating which of the following releases
(A)
CO2 most easily ?
Cl (A) CaCO3 (B) K2CO3
OCOCH3 (C) Na2CO3 (D) MgCO3

(B) 8. In the reaction with HCl, an alkene reacts in


O2N accordance with the Markovnikov’s rule, to
give a product 1-chloro-1-
OCOCH3 methylcyclohexane. The possible alkene is:
(C) CH2| CH3
H3CO
OCOCH3 (A) I (B) II
(D)
CH3

3. Reaction of phenol with chloroform in


(C) I and II (D)
presence of dilute sodium hydroxide finally
introduces which one of the following
functional group?
(A) – CHO (B) – CH2Cl 9. Number of possible isomers for the complex
(C) –COOH (D) –CHCl2 Coen2Cl2Cl will be : en = ethylenediamine
(A) 4 (B) 2
4. Which of the following reactions can be used (C) 1 (D) 3
for the preparation of alkyl halides ?
anh.ZnCl2
I CH3CH2OH + HCl   10. A gas such as carbon monoxide would be
II CH3CH2OH + HCl  most likely to obey the ideal gas law at :
III (CH3)2COH + HC  (A) Low temperatures and low pressures.
(B) High temperatures and low pressures
anh.ZnCl2
IV (CH3)2CHOH + HCl   (C) Low temperatures and high pressures
(A) III and IV only (D) High temperatures and high pressures.
(B) I, III and IV only
(C) I and II only 11. If Avogadro number NA, is changed form 6.022
(D) IV only ×1023 mol–1 to 6.022 × 1020 mol–1, this would
change :
5. In an SN1 reaction on chiral centres, there is : (A) The ratio of elements to each other in a
(A) 100 % inversion compound .
(B) 100 % recemization (B) The definition of mass in units of grams.
(C) inversion more than retention leading to (C) The mass of one mole of carbon.
partial recemization (D) The ratio of chemical species of each
(D) 100 % retention other in a balanced equation.

6. Which of the following is not the product of 12. Gadolinium belongs to 4f series. It’s atomic
number is 64. Which of the following is the
correct electronic configuration of gadolinium?
dehydration of OH ? (A) Xe 4f65d26s2 (B) Xe4f86d2
(C) Xe4f95s1 (D) Xe4f75d16s2
PREVIOUS YEAR PAPER [NEET - 2015] Page # 117
13. What is the pH of the resulting solution when 21. In the extraction of copper from its sulphide
equal volumes of 0.1M NaOH and 0.01 M HCl ore, the metal is finally obtained by the
are mixed ? reduction of cuprous oxide with :
(A) 1.04 (B) 12.65 (A) sulphur dioxide (B) iron II sulphide
(C) 2.0 (D) 7.0 (C) carbon monoxide (D) copperI sulphide
14. Decreasing order of stability of O2, O2–, O+2
and O22– is : 22. Which one of the following pairs of solution
is not an acidic buffer ?
(A) O2  O22   O2  O2 (A) H3PO4 and Na3PO4
(B) HClO4 and NaClO4
(B) O2  O2  O2  O22 
(C) CH3COOH and CH3COONa
(C) O22  O2  O2  O2 (D) H2CO3 and Na2CO3

(D) O2  O2  O22   O2 23. Assuming complete ionization, same moles of
which of the following compounds will require
15. The correct statement regarding defects in the least amount of acidified KMnO 4 for
crystalline solids is : complete oxidation ?
(A) Frenkel defect is found in halides of (A) FeNO22 (B) FeSO4
alkaline metals. (C) FeSO3 (D) FeC2O4
(B) Schottky defects have no effect the
density of crystalline solids.
24. The number of structural isomers possible
(C) Frenkel defects decreases the density
from the molecular formula C3H9N is :
of crystalline solids.
(A) 3 (B) 4
(D) Frenkel defect is a dislocation defect.
(C) 5 (D) 2
16. Which of the following statements is not
correct for a nucleophile ? 25. 20.0 g of a magnesium carbonate sample
(A) Nucleophiles are not electron seeking decomposes on heating to give carbon
(B) Nucleophile is a Lewis acid dioxide and 8.0 g magnesium oxide. What
(C) Ammonia is a nucleophile will be the percentage purity of magnesium
(D) Nucleophiles attack low e– density sites carbonate in the sample ?
(A) 84 (B) 75
17. The hybridization involved in complex (C) 96 (D) 60
NiCN4 2– is : At. No. Ni = 28
(A) d2sp3 (B) dsp2 26. Two possible stereo–structuers of
(C) sp3 (D) d2sp2 CH3CHOH. COOH, which are optically active,
are called :
18. The stability of +1 oxidation state among Al, (A) Mesomers (B) Diastereomers
Ga, ln and Tl increase in the sequence : (C) Atropisomers (D) Enantiomers
(A) In < Tl < Ga < Al (B) Ga < In < Al < Tl
(C) Al < Ga < In < Tl (D) Tl < In < Ga < Al
27. The heat of combustion of carbon to CO2 is
–393.5 kJ/mol. The heat released upon
19. The sum of coordination number and oxidation
formation of 35.2 g of CO2 from carbon and
number of the metal M in the complex
[M(en)2 (C2O4)]Cl where en is ethylenediamine oxygen gas is :
is : (A) –3.15 kJ (B) –315 kJ
(A) 8 (B) 9 (C) +315 kJ (D)–630 kJ
(C) 6 (D) 7
28. The rate constant of the reaction A  B is
20. Which of the statements given below is incorrect ? 0.6×10 – 1 3 mole per second. If the
(A) OF2 is an oxide of fluorine concentration of A is 5 M, then concentration
(B) Cl2O7 is an anhydride of perchloric acid of B after 20 minutes is :
(C) O3 molecule is bent (A) 0.72 M (B) 1.08 M
(D) ONF is isoelectronic with O2N– (C) 3.60 M (D) 0.36 M
Page # 118 PREVIOUS YEAR PAPER [NEET - 2015]
29. 2–
The formation of the oxide ion, O (g), from 34. The name of complex ion, FeCN63– is :
oxygen atom requires first an exothermic and (A) Hexacyanidoferrate III ion
then an endothermic step as shown below : (B) Hexacyanoiron III ion
Og + e–  O–g ; fH– = –141 kJ mol–1 (C) Hexacyanitoferrate III ion
O–g + e–  O2–g ; fH– = +780 kJ mol–1 (D) Tricyanoferrate III ion
Thus process of formation of O2– in gas phase
is unfavourable even though O 2– is 35. Method by which Aniline cannot be prepared is :
isoelectronic with neon. It is due to the fact (A) potassium salt of phthalimide treated with
that, chlorobenzene followed by hydrolysis with
(A) addition of electron in oxygen results in aqueous NaOH solution.
(B) hydrolysis of phenylisocyanide with acidic
larger size of the ion
solution.
(B) electron repulsion outweighs the stability
(C) reduction of nitrobenzene with bromine
gained by achieving noble gas configuration.
in alkaline solution.
(C) O– ion has comparatively smaller size than
(D) reduction of nitrobenzene with H2/Pd in
oxygen atom.
ethanol.
(D) oxygen is more electronegative.
36. If the equilibrium constant for N2(g) + O2(g)
30. What is the mass of the precipitate formed 2NO(g) is K, the equilibrium constant for
when 50 mL of 16.9% solution of AgNO3 is
mixed with 50 mL of 5.8% NaCl solution ? 1 1
N (g) + O2 (g) NO(g) will be :
Ag = 107.8, N = 14, O = 16, Na = 23, Cl = 35.5 2 2 2
(A) 14g (B) 28 g (A) K2 (B) K1/2
(C) 3.5 g (D) 7 g 1
(C) K (D) K
2
31. Which is the correct order of increasing
energy of the listed orbitals in the atom of
37. Strong reducing behaviour of H3PO2 is due to:
titanium At.no. Z = 22
(A) Presence of two –OH groups and one P–
(A) 3s 3p 4s 3d (B) 3s 4s 3p 3d
H bond
(C) 4s 3s 3p 3d (D) 3s 3p 3d 4s (B) Presence of one –OH group and two P–H
bonds
32. Reaction of a carbonyl compound with one (C) High electron gain enthalpy of phosphorus
of the following reagents involves nucleophilic (D) High oxidation state of phosphorus
addition followed by elimination of water. The
reagent is : 38. 2,3–Dimethyl–2–butene can be prepared by
(A) sodium hydrogen sulphite heating which of the following compounds
(B) a Girgnard reagent with a strong acid ?
(C) hydrazine in presence of feebly acidic (A) (CH3)2 CH–CH2–CH=CH2
solution
(B) (CH3)2CH—CH—CH=CH2
(D) hydrocyanic acid
CH 3
33. The variation of the boiling points of the
(C) (CH3)3C—CH=CH2
hydrogen halides is in the order HF > Hi >
(D) (CH3)2C=CH–CH2–CH3
HBr > HCl.
What explains the higher boiling point of
39. Aqueous solution of which of the following
hydrogen fluoride ? compounds is the best conductor of electric
(A) The effect of nuclear shielding is much current ?
reduced in fluorine which polarises the HF (A) Fructose, C6H12O6
molecules. (B) Acetic acid, C2H4O2
(B) The electronegativity of fluorine is much (C) hydrochloric acid, HCl
higher than for other elements in the group (D) Ammonia, NH3
(C) There is strong hydrogen bonding
between HF molecules. 40. The vacant space in bcc lattice unit cell is :
(D) The bond energy of HF molecules is (A) 32% (B) 26%
greater than in other hydrogen halides. (C) 48% (D) 23%
PREVIOUS YEAR PAPER [NEET - 2015] Page # 119
41. What is the mole fraction of the solute in a 44. The following reaction
1.00 m aqueous solution ?
NH2 H
(A) 0.0177 (B) 0.177 N
+ Cl NaOH
(C) 1.770 (D) 0.0354
O
O
42. The oxidaiton of benzene by V2O5 in the
presence of air produces : is known by the name :
(A) benzaldehyde (A) Schotten–Baumen reaction
(B) benzoic anhydride (B) Friedel–Craft's reacton
(C) maleic anhydride (C) Perkin's reaction
(D) benzoic acid (D) Acetylation reaction

43. Caprolactam is used for the manufacture of 45. The number of water molecules is maximum
(A) Nylon–6,6 (B) Nylon–6 in :
(C) Teflon (D) Terylene (A) 18 moles of water
(B) 18 molecules of water
(C) 1.8 gram of water
(D) 18 gram of water
Page # 120 PREVIOUS YEAR PAPER [NEET - 2015]

ANSWERKEY
BIOLOGY
1. C 2. A 3. A 4. C 5. D 6. A 7. A
8. A 9. B 10. A 11. A 12. C 13. B 14. C
15. D 16. B 17. B 18. B 19. D 20. C 21.  A
22.  B 23.  A 24.  C 25.  D 26. B 27. C 28. C
29. D 30. C 31. B 32. C 33. B 34. A 35. C
36. C 37. B 38. B 39. D 40. C 41. B 42. C
43. B 44. B 45. B 46. B 47. A 48. C 49. D

50. B 51. B 52. A 53. C 54. A 55. D 56. D

57. B 58. A 59. B 60. A 61. B 62. B 63. D


64. A 65. A 66. D 67. D 68. A 69. B 70. D
71. B 72. B 73. B 74. A 75. B 76. A 77. A
78. D 79. D 80. A 81. B 82. D 83. C 84. B
85. A 86. C 87. A 88. B 89. D 90. D

PHYSICS

1. B 2. D 3. D 4. D 5. A 6. B 7. D
8. B 9. A 10. A 11. C 12. A 13. B 14. A
15. A 16. D 17. A 18. A 19. B 20. C 21. B
22. D 23. A 24. A 25. D 26. A 27. B 28. A
29. B 30. A 31. D 32. B 33. C 34. A 35. A
36. B 37. D 38. A 39. C 40. C 41. A 42. D
43. B 44. D 45. A

CHEMISTRY

1. A 2. A 3. A 4. B 5. C 6. A 7. D

8. B 9. D 10. B 11. B 12. D 13. B 14. B

15. D 16. B 17. B 18. C 19. B 20. A 21. D

22. A 23. B 24. B 25. A 26. D 27. C 28. A

29. B 30. D 31. A 32. C 33. C 34. A 35. A

36. B 37. B 38. C 39. C 40. B 41. A 42. C

43. B 44. A 45. A


PREVIOUS YEAR PAPER [NEET - 2015] Page # 121

SOLUTION
PHYSICS
1. B
4. D

Incompressible liquid
A1 V1 = A2V2
R2V = nr2V

R2 v R 2v
v  2
r2 nr
mg sin 30° = S mg cos 30°

2. D sin30 12 1
S   
 cos 30 2  3 3

m1 r m2 1
S   0.6
1.732
x (L–x) mg cos 
fnet
1 1 2
K=
2 1
 2

m x2 02  m2 L  x  02

dk 30
0
dx fnet = mg sin 30 –  mg cos 
m1x = m2 (L –x)
10   10  3
m1x = m2K – m2x a= 
2 2
m2L
x= m m a=5– 5 3 
1 2


a = 5 1 - 3μ 
3. D
1 2
2mE s= at
r 2
Bq
1
r = same 4=
2

 5 1  3 
B = same
1
 11  3
mPEP mE 10

qP q
1 9
3  1  
m = 4mP 10 10
q= 2qP
3 3
  0.3  1.932  0.5
2 E 10
EP 
2
EP = E 5. A
Adiabatic
Page # 122 PREVIOUS YEAR PAPER [NEET - 2015]
6. B 8. B
1
mv '2  mgh ...(1) V0  V 
2 n' = n  V  V cos 60 
for return motion  S 
1
mv '2  20510  m
2 9. A
V nxyrz
1 put values
mv' 2  200m ...(2)
2 KE v' x+y=0
KE when it collides –x – 3y + z = 1 –x = –1
1 x = 1, y = –1, z = –1
=2× mv'2  400 m
2
10. A
energy conservation for downfall
1 rRT
mv20  200 mg  400 m VSS =
2 M
put values we get
v20  400
r = 1.6
v0  400  20 m / s CP
r
CV
7. D
CP = rCV
1 1 1 = 1.6 × 5 = 5
 Rz2  2  2
 n
 1 n2
11. C
For lymen  
n1 = 1 A.B  0
n2 = 2 t 

1  4  1 2 2
 Rz2  
  4  
t

1 3 2
 Rz
 4
12. A
1 1 1 Potential at R = 3.5 – 0.5 = 3V
 Rz2  2  2 
  n1 n2  3
i  .03  3  102
For barmer = n1 2 n2 = 3 100
= 30 × 10–3
1 1 1
 Rz2  2  2 
 2 3 
13. B
1 1 1  dv
 Rz2    E     6y  a  0   6y  0
 4 9 dx

9  4  dv
2  5  Ey      6x  1  2z 
 Rz2    Rz  36 
 36   
dv
5 Ez    2y 
Rz2
36 5 4 5
    Point (1, 1, 0)
3 2 36 3 27
Rz 
4    ˆ
PREVIOUS YEAR PAPER [NEET - 2015] Page # 123
14. A 18. A
h = 0.25 × 106 Re = 6.385 × 106
r = Re + h = 6.63 × 106 m m1v12 m2v22

r1 r2
GM gR 2
V0   n2 = 4 n=2
r r

19. B

Q2
h F
2A0

+ –
Re
+ –
F
+ –

g + F –
=R
r –
+

gR2 gR + –
V0 = 
R h h –
1 +
R
+ –

9.8  6.38  106 62.54  106 d


v0  
0.25 1.04 2
1
F
CV 
6.38
2A0
= 7.9 × 103 m/s
C2 V 2 CV 2
15. A 
F = 2 0 A  d 2d
R = R1 + R2 d
l l   l l
 1 2   11  21 l1 = l2 = l
 A  A A
20. C

1  2 r  48 mt ˆi  4 cos t ˆj

2 
v  4  cos t ˆi  4 sin t ˆj
2 1 1
  
 1 2 a  42 sin t ˆi  42 cos t ˆj

2 1  2 212  w2 4 sin t ˆi  4 cos t ˆj


   
 12 1  2
 
a  w2r
16. D |a| = 4 2
Acceleration always towards the center
|v| = 16 2  4
17. A
162
Their relative velocity should remain in |r| = 4 a  42
4
direction of relative position
Page # 124 PREVIOUS YEAR PAPER [NEET - 2015]
21. B 23. A
R C
Refrigerator

T1
Q1 

~
V
T2
V
i Q2
Z
Performance factor
1 T2 = 253
Z R2 
2c2
Q2
G = cain C2 = Cmed n= 5
C2 > C1 

z2 < z1 VC2  v2  VR2


Q2 T2
i2 > i1   5
Q1  Q2 T1  T2
voltage across C = VC = i xC
VC21  V 2  VR21
6 6
T1  T   253 T2  5T1  5T2
V 2
V V2 2 5 2 5
C2 R2

= 303.6 = 30.6 = 31° C


VC22  VC1 VA > V b

22. D 24. A

 75  /2
2
  1.5m

hc 2hc
k1    k2   
 
k2 = 3k1

2 
2 2hc 3hc
   3
2  
2
25. D
Magnification (M) = L/I

26. A

2
2  I1  I2   1  25 
2
 Imax
 
3
Imin 2  25  1 
PV
 I1  I2   
n1 
2
2 2
P 6 3 9
n  v     
2 4
  2
  4
105, 210, 315, 420
PREVIOUS YEAR PAPER [NEET - 2015] Page # 125
27. B 31. D
RT critical angle for red light
P
M
1 
A B  C Re d  sin1  
 nR 
T = same
PA = 2PB
 1 
 sin1  
PA  M  1.39 
 A  B
PB MA B

MA 1 3  1 
 
 A B  1.5    sin1  
MB B  A 2 4  1.39 
Similarly criticle angle of green light
28. A
A  1 
 C Green  sin1  
 1.44 

 1 
 C Blue  sin1  
 1.47 
E
angle of incidence for all colour is
 m Ametr
20 1 1
45° = sin–1  sin1
2 1.414
This means that red light will emerge but
green and blue will totaly reflect.
480
480  20 32. B
RA = T = NIAB sin 
500
T = 50 × 2 × (0.12 × 0.1) 0.2 × sin 60
96
  19.2
5 3
Req. = R + RA T = 50 × 2 × 12 × 1 × 2 × 10–4 ×
2
= 40.8 + 19.2
= 60  T = 1200 × 10–4 × 3
E = 30 v
T = 0.12 × 1.732
30 1 T = 0.20
i   0.5
60 2
33. C
29. B
 = 0 (1 –rt)
a

 rt = 5 × 10–4 × 40
0
= 2 × 10–2 = .02 b b

30. A
c
energy work Pv
Power =   X e y
time time t
P = hdg = 150 × 10–3 × 13.6 × 103 × 10
P = 2.04 × 104 As electron comes near magnetic flux will
4
2.04  10  5  10 3 increase and then decrease hence induced
Power   1.7 current will change its direction.
60
Page # 126 PREVIOUS YEAR PAPER [NEET - 2015]
34. A 39. C
momentum = same Kmax = E – 
P2 12400
KE   ev  2.28ev
2m 5000

1 = 2.48 – 2.28 ev
K.E. 
mass = 0.20 ev
mHe < mth
h
 KEHE > KEth 
2m.Kma

35. A
 h
ˆ
F  ˆi  3j  6k min 
2m Kma

ˆ
F  2iˆ  6j  12k
min  2.8  109 m
angulr momentum = conserved
 = 0
  40. C
FF  0
  The path difference will be /2 hence phase
 r and F are parallel
differencce between wares reaching the
 = –1
 
F  ˆi  3j ˆ
ˆ  6k central maximum is .
2


F  2 ˆi  3j ˆ
ˆ  6k 
 
r  2F

36. B
h1R1 = h2R2
water stays at upper height

37. D
Vmax = a = 
max = 2a = a
 = 2 = (aw) (w) 41. A
 = () 2
1 1 v 1
mv2  m    mv' 2
 2 2 2 3 2
 
 T

q2
T  2
 q'2 = q2 
g

8q2
38. A q'2 =
g
0
12400
  0.82 A
15  1000
8v2 2 2
wavelength Range of x-ray is approximate q'   q v
g 3
0 0
0.03 A to 3 A
PREVIOUS YEAR PAPER [NEET - 2015] Page # 127
42. D 44. D
E0
r1 v0
Av = v
c

L vi = 2 cos (15t + /3)


v0 = (Av) vi
r
= 300 cos (15t + +/3)
E r
x 0 
r  r1 L 45. A
= 0 – t
 Er  1
x   0 . 0
 r1  r  L t


E = xl 

E0rl
= r  r  L 54 km/hr–1
1 0

43. B

mg l wl R = 45
y  I = 3 kg m2
Al Al
q
yAl 
w= rt
l
l = same 5
54 
l = same 18 100 20
  
A = same 0.45  15 45 9
w×y T = I
ws y 2 20 20
 s  T=3×   6.66 Kg.
wB yB 1 9 3
Page # 128 PREVIOUS YEAR PAPER [NEET - 2015]

CHEMISTRY
1. A 7. D
xe xe
F O 8. B
FF F O OO
9. D
O
F F 10. B
xe xe At high temperature & low pressure.
F F
O O O 11. B
Squar plane Tetrahedral As 6.022 × 1023 × massnuclear = 1.
These are having different structures. But 6.022 × 1020 × massnuclear 1

2. A 12. D
–NO2 group having – M effect
13. B
3. A pH = –logH+
OH 0.1  y  0.01  y 0.09
OH– = = m
2y 2
Dil. NaOH
+ CHCl3 pOH = –logOH–

OH  0.09 
OH = –log   = –1.346
 2 
CHO  pH = 14 – pOH
+
= 14 – 1.345
= 12.65
CHO
Salicyladehyde 14. B
4. B O2 kk 2s2 *2s2 2Pz2 2p2x = 2p2y ,
I, III and IV Lucas reagent
'
x 2p x ' =  * 2py
5. C
Some degree of racemisation will take place.  B.O. = 2
6. A O2+  B.O.= 2.5
O2–  B.O. = 1.5
O22–  B.O. = 1
 Stability order O2+ > O2 > O2– > O22–

OH 15. [D]
Frenkel defect is a dislocation defect

Hydride shift 16. B


Nucleophiles is a lewis base.

17. B

18. C

19. B

20. A

21. D
CH3
C—CH2—CH3 22. A
Must be a conjugate acid base pair

CORPORATE OFFICE : Motion Education Pvt. Ltd., 394 - Rajeev Gandhi Nagar, Kota
PREVIOUS YEAR PAPER [NEET - 2015] Page # 129
23. [B] Ct = 4.28, ie the conc. of A left
7  B formed = 0.72 M
 Mn2 
KMnO4 
It is an oxidising agent 29. [B]
 for FeSO4 least amount of FeSO4 is required This is due to the inner e– repulsion of O– to
because sulphur is present in its highest O.S., form O2-
so it will only oxidise Fe2+ to Fe3+
30. D
24. B AgNo3 + NaCl  AgCl + NaNO3
C3H9N  16.9% 5.8%
H7C3—NH2  2 50 ml 50ml
H3C 31. [A]
H 3C—N—H3C 1 using (n + )mole
H3C—NH—C2H5 1
32. [C]
25. [A] C=O+H2N—NH2

pH = 4.5 –7
8 gm > C = N —NH2
8 Hydrazone
n=
40
33. C
1 moles 1
n= n=
5 5 34. A

35. A
1
W= × 84 = 16.8 gm
5 O Cl
16.8 C
 present purity =  100 = 84 %
20 NK + [Resonance]
C
26. D O
Enantiomers

COOH COOH Hydrolysis


aq. NaOH

H OH HO H
Aniline does not form

36. [B]
CH3 CH3
K1/2
d-lactic acid l-lactic acid

27. [C] 37. B


C + O2  CO2 ; x = –393.5 kJ/mole
 for 2 mole of CO2 heat released –393.5 kJ 38. C
CH3
 35.2 
for 0.8 mole  44  of CO2 CH3—C—CH=CH2
 
heat released = + 393.5 × 0.8 CH3
= +314.8 kJ

28. [A]
CO – Ct = kT
5 – Ct = 0.6 × 10–3 × 20 × 60
Ct = 5 –0.72
Page # 130 PREVIOUS YEAR PAPER [NEET - 2015]
CH3 42. C

CH 3—C—CH–CH3 O

V2O5 H—C—C
CH3 O
Methyl Shift H—C—C

O
Maleic anhydride
CH3 CH3
43. B
CH3—C—CH—CH3 Nylon – 6
44. A
Schotten - Baumen reaction

O
CH3 CH3 Ph—NH 2+Ph—C—Cl
CH3—C=CH—CH3

2, 3 - Dimethyl-2- butene NaOH

39. C O
40. [B] Ph—NH—C—Ph
voccupied 45. [A]
packing efficiency=
vtotal nH2O = 18 moles
or  no. of H2O molecules = 18 × NA
Packing fraction

 4 / 3r   2
3

=  4r 
3

 
 3
= 0.74
 vacant space = 26%

41. [A]
xsolute 1000
M = 1x 
 solute  MMsolvent

xsolute 1000
1 = 1x 
solute 18
1–xsolute = 55.55 xsolute
1
xsolute = = 0.01768
56.55
PREVIOUS YEAR PAPER [NEET - 2014] Page # 131

PREVIOUS YEAR 2014 PAPER


BIOLOGY
1. Five kingdom system of classification suggested (A) Meghalaya
by R.H. Whittaker is not based on: (B) Corbett National Park
(A) Mode of reproduction. (C) Keolado National Park
(B) Mode of nutrition. (D) Western Ghat
(C) Complexity of body organisation
7. You are given a fairly old piece of dicot stem
(D) Presence or absence of a well-defined
and a dicot root. Which of the following
nucleus.
anatomical structures will you use to distinguish
between the two ?
2. The main function of mammalian corpus luteum
is to produce : (A) Secondary phloem (B) protoxylem

(A) progesterone (C) Cortical cells (D) Secondary xylem

(B) human chorionic gonadotropin 8. In ‘S’ phase of the cell cycle :


(C) relaxin only (D) estrogen only (A) amount of DNA remains same in each cell.
(B) chromosome number is increased
3. In which one of the following processes CO2 is
not released? (C) amount of DNA is reduced to half in each cell

(A) Aerobic respiration in animals (D) amount of DNA doubles in each cell.

(B) Alcoholic fermentation 9. A species facing extremely high risk of


(C) Lactate fermentation extinction in the immediate future is called :
(D) Aerobic respiration in plants (A) Endemic (B) Critically Endangered
(C) Extinct (D) Vulnerable
4. Choose the correctly matched pair :
(A) Moist surface of buccal cavity - Glandular 10. Fruit colour in squash is an example of :
epithelium (A) Dominant epistasis
(B) Tubular parts of nephrons – Cuboidal (B) Complementary genes
epithelium (C) Inhibitory genes
(C) Inner surface of bronchioles – squamous (D) Recessive epistasis
epithelium
(D) Inner lining of salivary ducts – Ciliated 11. Identity the hormone with its correct matching
epithelium of source and function
(A) Melatonin – Pineal gland, regulates the
5. Which of the following shows coiled RNA strand normal rhythm of sleepwake cycle
and capsomeres? (B) Progesterone – corpus – luteum, secondary
(A) Tobacco mosaic virus sex organs.
(B) Measles Virus (C) Atrial natriuretic factor –ventricular wall
(C) Retrovirus (D) Polio virus increases the blood pressure
(D) Oxytocin – posterior pituitary growth and
6. Just as a person moving from Delhi to Shimla maintenance of mammary glands
to escape the heat for the duration of hot
summer, thousands of migratory birds from 12. An example of edible underground stem is
Siberia and other extremely cold northern (A) Groundnut (B) Sweet potato
regions move to : (C) Potato (D) Carrot
Page # 132 PREVIOUS YEAR PAPER [NEET - 2014]
13. Which of the following causes an increase in 18. The solid linear cytoskeletal elements having
sodium reabsorption in the distal convoluted a diameter of 6 nm a of a single type of
tubule ? monomer are known as;
(A) Increase in antidiuretic hormone levels (A) Microfilaments

(B) Decrease in aldosterone levels (B) Intermediate filaments

(C) Decrease in antidiuretic hormone levels (C) Lamins (D) Microtubules

(D) Increase in aldosterone levels 19. Which one of the following living organisms
completely lacks a cell wall?
14. Which structures perform the function of
(A) Sea – fan (Gorgonia)
mitochondria in bacteria ?
(B) Sacharomyces
(A) Ribosomes (B) Cell wall
(C) Blue – green algae
(C) Mesosomes (D) Nucleoid
(D) Cyanobacteria
15. Select the option which is not correct with
20. Tracheids differ from other tracheary elements
respect to enzyme action :
in:
(A) Addition of lot of succinate does not reverse
(A) being imperforate (B) lacking nucleus
the inhibition of succinic dehydrogenase by
(C) being lignified
malonate.
(D) having casparian strips
(B) A non – competitive inhibitor binds the
enzyme at a site distinct from that which binds 21. Select the correct matching of the type of
the substrate. the joint with the example in human skeletal
(C) Malonate is a competitive inhibitor of system:
succinic dehydrogenase. Type of joint Example
(D) Substrate binds with enzyme at its active (A) Pivot joint - Between third and
site. fourth cervical
vertebrae
16. Which the particular type of drug that is
obtained from the plant whose one flowering (B) Hinge joint - Between humerus
branch is shown below? and pectoral girdle
(C) Gliding joint - Between carpals
(D) Cartilaginous joint - Between frontal and
pariental

22. A man whose father was colour blind marries a


woman who had a colour blind mother and
normal father. What percentage of male children
of this couple will be colour blind?
(A) Depressant (B) Stimulant (A) 0% (B) 50%
(C) Pain – killer (D) Hallucinogen (C) 75% (D) 25%

17. Fructose is absorbed into the blood through 23. A few normal seedlings of tomato were kept in
mucosa cells of intestine by the process called: a dark room After a few days they were found
(A) facilitated transport to have become white-coloured lime albinos,
Which of the following terms will you use to
(B) simple diffusion
describe them?
(C) co-transport mechanism
(A) Embolised (B) Etiolated
(D) active transport
(C) Defoliated (D) Mutated
PREVIOUS YEAR PAPER [NEET - 2014] Page # 133
24. Function of filiform apparatus is to: 31. Match the following and select the correct option:
(A) Stimulate division of generative (a) Earthworm (i) Pioneer species
(B) Produce nectar (b) Succession (ii) Detritivore
(C) Guide entry of pollen tube (c) Ecosystem service (iii) Natality
(D) Recognize the suitable pollen at stigma (d) Population growth (iv) Pollination
(a) (b) (c) (d)
25. Choose the correctly matched pair:
(A) (i) (ii) (iii) (iv)
(A) Adipose tissue – Dense connective tissue
(B) (iv) (i) (iii) (ii)
(B) Areolar tissue – Loose connective tissue
(C) (iii) (ii) (iv) (i)
(C) Cartilage – Loose connective tissue
(D) (ii) (i) (iv) (iii)
(D) Tendon – Specialized connective tissue
32. A location with luxuriant growth of lichens on
26. Forelimbs of cat, lizard used in walking forelimbs
the trees indicates that the:
of whale used in swimming and forelimbs of
(A) trees heavily infested
bats used in flying are an example of:
(B) location is highly polluted
(A) Adaptive radiation
(C) location is not polluted
(B) Homologous organs
(D) trees are very healty
(C) Convergent evolution
(D) Analogous organs 33. In vitro clonal propagation in plants is
characterized by:
27. Which one of the following is a non – reducing
(A) Northern blotting
carbohydrate?
(B) Electrophoresis and HPLC
(A) Sucrose (B) Lactose
(C) Microscopy (D) PCR and RAPD
(C) Ribose 5 – phosphate
(D) Maltose 34. An alga which can be employed as food for
human being is:
28. At which stage of HIV infection does one
(A) chlorella (B) Spirogyra
usually show symptoms of AIDS?
(C) Polysiphonia (D) Ulothrix
(A) When the infected retro virus enters host
cells. 35. Which one of the following growth regulators
(B) When HIV damage large number of helper is known as ‘stress hormone’?
T – Lymphocytes. (A) Ethylene (B) GA3
(C) When the viral DNA is produced by reverse (C) Indole acetic acid (D) Abscissic and
transcriptase.
36. The enzyme recombinase is required at which
(D) Within 15 days of sexual contact with an
stage of meiosis:
infected person.
(A) Zygontene (B) Diplotene
29. What gases are produced in anaerobic sludge (C) Diakinesis (D) Pachytene
digesters?
37. Assisted reproductive technology, IVF involves
(A) Methane, Hydrogen Sulphide and CO2
transfer of:
(B) Methane, Hydrogen Sulphide and O2
(A) Zygote into the fallopian tube.
(C) Hydrogen Sulphide and CO2
(B) Zygote into the uterus.
(D) Mehane and CO2 only
(C) Embryo with 16 blastomeres into the
fallopian tube.
30. Anoxygenic photosynthesis is characteristic of:
(D) Ovum into the fallopian tube.
(A) Spirogyra (B) Chlamydomonas
(C) Ulva (D) Rhodospirillum
Page # 134 PREVIOUS YEAR PAPER [NEET - 2014]
38. An example of ex situ conservation is: 45. Select the correct option:
(A) Seed Bank (B) Wildlife Sanctuary Direction Direction of reading
(C) Sacred Grove (D) National Park of RNA of the template
synthesis DNA strand
39. The osmotic of a cell kept in water is chiefly
regulated by: (A) 3’ – 5’ 5 ’ – 3’

(A) Vacuoles (B) Plastids


(C) Ribosomes (D) Mitohondria (B) 5’ – 3’ 5 ’ – 3’

40. Which one of the following is wrong about Chara? (C) 3’ – 5’ 3 ’ – 5’


(A) Globule and nucule present on the same plant.
(B) Upper antheridium and lower oogonium (D) 5’ – 3’ 3 ’ – 5’

(C) Globule is male reproductive structure


46. The organization which publishes the Red List
(D) Upper oogonium and lower round
of species is:
antheridium.
(A) IUCN (B) UNEP
41. The first human hormone produced by (C) WWF (D) ICFRE
recombinant DNA technology is:
(A) Estrogen (B) Thyroxin 47. A human female with Turner’ s syndrome:

(C) Progesterone (D) Insulin (A) has one additional X chromosome.


(B) exhibits male characters.
42. Which one of the following statements is not (C) is able to produce children with normal
correct? husban(D)
(A) In retina the rods have the photo pigment (D) has 45 chromosomes with XO.
rhodopsin while cones have three different
photo pigments. 48. Match the following and select the correct
(B) Retinal is a derivative of Vitamin (C) answer:
(C) Rhodopsin is the purplish red protein (a) Centriole (i) Infoldings in mitochondria
present in rods only. (b) Chlorophyll (ii) Thylakoids
(D) Retinal is the light absorbing portion of (c) Cristae (iii) Nucleic acids
visual photo pigment. (d) Ribozymes (iv) Basal body cilia or
flagella
43. Which one of the following statements is
correct? (a) (b) (c) (d)

(A) Mango is a parthenocarpic fruit. (A) (i) (ii) (iv) (iii)

(B) A proteinaceous aleurone layer is present (B) (i) (iii) (ii) (iv)
in maize grain. (C) (iv) (iii) (i) (ii)
(C) A sterile pistile is called a staminode. (D) (iv) (ii) (i) (iii)
(D) The speed in grasses is not endospemi(C)
49. Approximately seventy percent of carbon-
44. Pollen tablet are available in the market for: dioxide absorbed by the blood will be
(A) Breeding programmes transported to the lungs:
(B) Supplementing food (A) in the form of dissolved gas molecules
(C) Ex situ conservation (B) by binding to R.(B)C

(D) In vitro fertilization (C) as carbamino – haemoglobin


(D) as bicarbonate ions
PREVIOUS YEAR PAPER [NEET - 2014] Page # 135
50. Which vector can clone only a small fragment 58. To obtain virus – free healthy plants from a
of DNA? diseased one by tissue culture technique, which
(A) Yeast artificial chromosome part/parts of the diseased plant will be taken?
(B) Plasmid (C) Cosmind (A) Palisade parenchyma
(D) Bacterial artificial chromosome (B) Both apical and axillary meristems
(C) Epidermis only
51. The zone of atmosphere in which the ozone
(D) Apical meristem only
layer is present is called:
(A) Mesoshpere (B) Stratosphere 59. Fight-or-flight reactions cause activation of:
(C) Troposphere (D) Ionosphere (A) the kidney, leading to suppression of
reninangiotensin-aldosterone pathway.
52. Which one of the following fungi contains
(B) the adrenal medulla, leading to increased
hallucinogens?
secretion of epinephrine and norepinephrine.
(A) Amanita muscaria (B) Neurospora sp.
(C) the pancreas leading to a reduction in the
(C) Ustilago sp. (D) Morchella esculenta
blood sugar levels.

53. A scrubber in the exhaust of a chemical industrial (D) the parathyroid glands, leading to increased
plant removes: metabolic rate.
(A) particulate matter of the size 5 micrometer
60. Stimulation of a muscle fiber by o motor neuron
or above
occurs at:
(B) gases like ozone and methane
(A) the transverse tubules
(C) particulate matter of the size 2.5 micrometer
(B) the myofibril
or less
(C) the sarcoplasmic reticulum
(D) gases like sulphur dioxide
(D) the neuromuscular junction
54. Select the Taxon mentioned that represents
both marine and fresh water species: 61. Planaria posses high capacity of:

(A) Ctenophora (B) Cephalochordata (A) regeneration

(C) Cnidaria (D) Echinoderms (B) alternation of generation


(C) bioluminescence (D) metamorphosis
55. When the margins of sepals or petals overlap
one another without any particular direction, 62. Placenta and pericarp are both edible portions
the condition is termed as: in:
(A) Imbricate (B) Twisted (A) Banana (B) Tomato
(C) Valvate (D) Vexillary (C) Potato (D) Apple

56. An aggregate fruit is one which develops from: 63. Deficiency symptoms of nitrogen and potassium
(A) Multicarpellary apocarpus gynoecium are visible first in:
(B) Complete inflorescence (A) Young leaves (B) Roots
(C) Multicarpellary superior ovary (C) Buds (D) Senescent leaves
(D) Multicarpellary syncarpous gynoecium
64. Geitonogamy involves:
57. Commonly used vector for human genome (A) fertilization of a flower by the pollen from
sequencing are: the same flower.
(A) BAC and YAC (B) Expression Vectors (B) fertilization of a flower by the pollen from a
(C) T/A Cloning Vectors (D) T – DNA flower of another plant in the same population.
Page # 136 PREVIOUS YEAR PAPER [NEET - 2014]
(C) fertilization of a flower by the pollen from 71. Which one of the following are analogous
another flower of plant belonging to a distant structures?
population. (A) Gills of Prawn and Lungs of Man.
(D) fertilization of a flower by the pollen from (B) Thorns of Bougainvillea and Tendrils of
another flower of the same plant. Cucurbita
(C) Flippers of Dolphin and Legs of Horse.
65. Viruses have:
(D) Wings of Bat and Wings of Pigeon.
(A) Prokaryotic nucleus
(B) Single chromosome 72. Dr.F. Went noted that if coleoptile tips were
(C) Both DNA and RNA removed and placed on agar for one hour, the
(D) DNA enclosed in a protein coat agar would produce a bending when placed on
one side of freshlycut coleoptile stumps. Of
66. How do parasympathetic neural signals affect what significance is this experiment?
the working of the heart? (A) It is the basis for quantitative determination
(A) Heart rate is increased without affecting of small amounts of growthpromoting substances.
the cardiac output. (B) It supports the hypothesis that IAA is auxin.
(B) Both heart rate and cardiac output increase. (C) It demonstrated polar movement of auxins.
(C) Heart rate decrease but cardiac output (D) It made possible the isolation and exact
increases. identification of auxin.
(D) Reduce both heart rate and cardiac output
73. Non-albuminous seed is produced in:
67. A marine cartilaginous fish that can produce (A) Castor (B) Wheat
electric current is
(C) Pea (D) Maize
(A) Torpedo (B) Trygon
(C) Scoliodon (D) Pristis 74. During which phase(s) of cell cycle, amount
of DNA in a cell remains at 4C level if the initial
68. An analysis of chromosomal DNA using the amount is denoted as 2C?
Southern hybridization technique does not use: (A) G1 and S (B) Only G2
(A) Blotting (B) Autoradiography (C) G2 and M (D) G0 and G1
(C) PCR (D) Electrophoresis
75. Transformation was discovered by:
69. Archaebacteria differ from eubacteria in: (A) Hershey and Chase (B) Griffith
(A) Mode of nutrition (B) Cell shape (C) Watson and Crick (D) Meselson and Stahl
(C) Mode of reproduction
76. Given below s a simplified model of phosphorus
(D) Cell membrane structure
cycling in a terrestrial ecosystem with four
70. If 20 J of energy is trapped at producer level, blanks (A-D). Identify the blanks
then how much energy will be available to
peacock as food in the following chain?
Plant  mice  snake  peacock
(A) 0.002 J (B) 0.2 J
(C) 0.0002 J (D) 0.02 J
PREVIOUS YEAR PAPER [NEET - 2014] Page # 137
82. Which of the following is a hormone releasing
a b c d
intra Uterine Device (IUD) ?
Rock
(A) Litter fall Producers Detritus (A) LNG – 20 (B) Cervical cap
minerals
(C) Vault (D) Multiload 375
Rock
(B) Detritu s Produ cers Litter fall
minerals
83. Given below is the representation of the extent
Rock
(C) Produ cers Litter fall Detritus of global diversity of invertebrates. What
minerals
groups the four portions (A-D) represent
Rock
(D) Detritus Litter fall Producers respectively ?
minerals

77. In a popular of 1000 individuals 360 belong to


genotype AA, 480 to Aa and the remaining
160 to aa Based on this data, the frequency
of allele A in the population is :
(A) 0.5 (B) 0.6
a b c d
(C) 0.7 (D) 0.4
Other
(A) Crustaceans Crustaceans Mol luscs animal
78. Tubectomy is a method of sterilization in which:
g roup
(A) ovaries are removed surgically. Other
Crustacea
(B) small part of vas deferens is removed or (B) Molluscs animal Insects
ns
group
tied up.
Other
Crustacea
(C) uterus is removed surgically (C) insects Moll uscs
ns
animal
g roup
(D) small part of the fallopian tube is removed
Other
or tied up. (D) insects Crustaceans animal Mol luscs
g roup
79. Which of the following is responsible for peat
formation? 84. Male gametophyte with least number of cells
is present in :
(A) Riccia (B) Funaria
(C) sphagnum (D) Marchantia (A) Funaria (B) Lilium
(C) Pinus (D) Pteris
80. Which one of the following shows isogamy with
non-flagellated gametes ? 85. The shared terminal duct of the reproductive
(A) Ectocarpus (B) Ulothrix and urinary system in the human male is :

(C) Spirogyra (D) Sargassum (A) Ureter (B) Vas deferens


(C) Vasa efferentia (D) Urethra
81. Which one of the following is wrongly matched?
86. Injury localized to the hypothalamus would most
(A) Translation – Using information in m-RNA
likely disrupt :
to make protein.
(A) co-ordination during locomotion.
(B) Repressor protein-Binds to operator to stop
(B) executive functions, such as decision
enzyme synthesis.
making.
(C) Operon – Structural genes, operator and
promoter. (C) regulation of body temperature.
(D) Transcription – Writing information from (D) short – term memory.
DNA to t-RNA
Page # 138 PREVIOUS YEAR PAPER [NEET - 2014]

87. Select the correct option describing 89. The motile bacteria are able to move by :
gonadotropin activity in a normal pregnant (A) flagella (B) cilia
female :
(C) pili (D) fimbriae
(A) High level of FSH and LH facilitate
implantation of the embryo. 90. Person with blood group AB is considered as
universal recipient because he has :
(B) High level of hCG stimulates the synthesis
of estrogen and progesterone. (A) both A and B antibodies in the plasm(A)

(C) High level of hCG stimulates the thickening (B) no antigen on RBC and no antibody in the

of endometrium. plasm(A)

(D) High level of FSH and LH stimulates the (C) both A and B antigens in the plasma but

thickening of endometrium. no antibodies.

(D) both A and B antigens on RBC but no


88. The initial step in the digestion of milk humans
antibodies in the plasm(A)
is carried out by ?

(A) Trypsin (B) Rennin

(C) Pepsin (D) Lipase


PREVIOUS YEAR PAPER [NEET - 2014] Page # 139

PHYSICS
1. If force (F), velocity (V) and time (T) are 5. The force 'F acting on a particle of mass 'm' is
indicated by the force-time graph shown below.
taken as fundamental units, then the
The change in momentum of the particle over
dimensions of mass are :-
the time Interval from zero to 8 s Is :
(A) [F V T–1] (B) [F V T–2]
–1
(C) [F V T ] –1 (D) [F V–1 T] 6
3
2. A projectile is fired from the surface of the earth

F(N)
with a velocity of 5 ms–1 and angle  with the 0
2 4 6 8
horizontal. Another projectile fired from another –3
planet with a velocity of 3 ms–1 at the same t(s)
angle follows a trajectory which is identical with
the trajectory of the projectile fired from th earth. (A) 24 N s (B) 20 N s
The value of the acceleration due to gravity on (C) 12 N s (D) 6 N s
the planet is (in ms–2) is: (given g = 9.8 m/s2)
(A) 3.5 (B) 5.9 6. A balloon with mass 'm ' is descending down
(C) 16.8 (D) 110.8 with an acceleration 'a' (where a < g) . How
much mass should be removed from it so that
it starts moving up with an acceleration 'a' ?
3. A particle is moving such that its position
coordinate (x, y) are 2ma 2ma
(2m, 3m) at time t = 0 (A) (B)
g a ga
(6m, 7m) at time t = 2 s and
(13m, 14m) at time t = 5s ma ma
 (C) (D)
g a g a
 
Average velocity vector Vav from t = 0 to
t = 5 s is 7. A body of mass (4m) is lying in x-y plane at
rest. It suddenly explodes into three pieces.
(A)
1
5

13 î  14 ĵ  (B)
7
3
 
î  ĵ Two pieces, each of mass (m) move
perpendicular to each other with equal
speeds (v). The total kinetic energy
 
(C) 2 î  ĵ (D)
11
5
 
î  ĵ generated due to explosion is
3
(A) mv2 (B) mv2
4. A system consists of three masses m1, m2 2
and m3 connected by a string passing over a (C) 2 mv2 (D) 4 mv2
pulley P. The mass m1 hangs freely and m2
8. The oscillation of a body on a smooth horizontal
and m3 are on a rough horizontal table (the surface is represented by the equation,
coefficient of friction = ). The pulley is X = A cos(t)
frictionless and of negligible mass. The where X = displacement at time t
downward acceleration of mass m1 is -  = frequency of oscillation
(Assume m1 = m2 = m3 = m) Which one of the following graphs shows
correctly the variation 'a' with 't' ?
m2 m3
P
a a
(A) O T t (B) O T t

m1
a a
g(1  2) 2g (C) O T t (D) O T t
(A) (B)
9 3
Here a = acceleration at time t
g(1  2) g(1  2)
(C) (D) T = Time period
3 2
Page # 140 PREVIOUS YEAR PAPER [NEET - 2014]
9. A solid cylinder of mass 50 kg and radius 0 .5 m 14. A certain number of spherical drops of a liquid
is free to rotate about the horizontal axis. A of radius 'r' coalesce to form a single drop of
massless string is wound round the cylinder with radius 'R' and volume 'V' . If 'T' is the surface
one end attached to it and other hanging freely. tension of the liquid, then :
Tension in the string required to produce an
1 1 
angular acceleration of 2 revolutions S–2 is :- (A) energy = 4VT  r  R  is released
 
(A) 25 N (B) 50 N
(C) 78.5 N (D) 157 N 1 1 
(B) energy = 3VT    is absorbed
r R
10. The ratio of the accelerations for a solid sphere
1 1 
(mass 'm' and radius 'R') rolling down an incline (C) energy = 3VT  r  R  is released
of angle '' without slipping and slipping down  
the incline without rolling is : (D) Energy is neither released nor absorbed
(A) 5 : 7 (B) 2 : 3
(C) 2 : 5 (D) 7 : 5 15. Steam at 100°C is passed into 20 g of water
at 10°C. When water acquires a temperature
11. A black hole is an object whose gravitationai of 80°C, the mass of water present will be :
field is so strong that even light cannot escape [Take specific heat of water = 1 cal g–1 °C–1
from it. To what approximate radius would and latent heat of steam = 540 cal g–1]
earth (mass = 5.98 x 1024 kg) have to be (A) 24 g (B) 31.5 g
compressed to be a black hole ? (C) 42.5 g (D) 22.5 g
(A) 10–9 m (B) 10–6 m
–2 16. Certain quantity of water cools from 70° C
(C) 10 m (D) 100 m
to 60° C in the first 5 minutes and to 54°C in
the next 5 minutes. The temperature of the
12. Dependence of intensity of gravitational field
surroundings is :-
(E) of earth with distance (r) from centre of
(A) 45°C (B) 20°C
earth is correctly represented by :-
(C) 42°C (D) 10°C

E 17. A monoatomic gas at a pressure P, having a


E
R volume V expands isothermally to a volume 2V
O and then adiabatically to a volume 16V. The
O
(A) r (B) R r 5
final pressure of the gas is : (take  = )
3
(A) 64P (B) 32P
P
(C) (D) 16P
64
E E
R 18. A thermodynamic system undergoes cyclic
O O
(C) r (D) R r process ABCDA as shown in fig. The work
done by the system in the cycle is :

P
13. Copper of fixed volume V; is drawn into wire C B
3P 0
of length 'l. When this wire is subjected to a
constant force 'F', the extension produced in 2P 0
the wire is l . Which of the following graphs
is a straight line ? P0 D
A
1
(A) l versus (B) l versus l2 V0 2V0 V
l
(A) P0V0 (B) 2P0V0
1 P0V0
(C) l versus (D) l versus l (C) (D) zero
l2 2
PREVIOUS YEAR PAPER [NEET - 2014] Page # 141
19. The mean free path of molecules of a gas,
(radius 'r') is inversely proportional to :-
(A) r3 (B) r2 E E
(C) r (D) r
(A) (B)

20. If n 1 , n 2 and n 3 are the fundamental 0 0


d d
frequencies of three segments into which a
string is divided, then the original fundamental
frequency n of the string is given by -

1 1 1 1 E E
(A) n  n  n  n
1 2 3
(C) (D)
1 1 1 1 0 0
   d d
(B) n n1 n2 n3

(C) n  n1  n2  n3 24. A conducting sphere of radius R is given a


(D) n = n1 + n2 + n3 charge Q. The electric potential and the
electric field at the centre of the sphere
21. The number of possible nautral oscillations respectively are -
of air column in a pipe closed at one end of
length 85 cm whose frequencies lie below Q
1250 Hz are : (Velocity of sound = 340 ms– (A) zero and
4 0 R 2
1)

(A) 4 (B) 5
Q
(C) 7 (D) 6 (B) 4  R and Zero
0

22. A speedlng motorcyclist sees traffic jam


ahead of him. He slows down to 36 km/hour. Q Q
(C) 4  R and 2
He finds that traffic has eased and a car 0 4 0 R

moving ahead of him at 18 km/ hour is honking


(D) Both and zero
at a frequency of 1392 Hz. If the speeds of
sound is 343 m/s, the frequency of the honk
as heard by him will be 25. In a region, the potential is represented by
(A) 1332 Hz (B) 1372 Hz V(x,y,z) = 6x – 8xy – 8y + 6yz, where V is in
(C) 1412 Hz (D) 1454 Hz volts and x,y,z are in metres. The electric
force experienced by a charge of 2 coulomb
23. Two thin dielectric slabs of dielectric constants situated at point (1,1,1) is -
K2 and K2 (K1 < K2) are inserted between plates
of a parallel plate capacitor, as shown in the (A) 6 5 N (B) 30 N
figure . The variation of electric field 'E' between
the plates with distance 'd' as measured from (C) 24 N (D) 4 35
plate P is correctly shown by :
26. Two cities are 150 km apart. Electric power
P+ – Q is sent from one city to another city through
+ – copper wires. The fall of potential per km is
+ –
+ – 8 volt and the average resistance per km is
+ –
+ 0.5 . The power loss in the wires is -
+ –
(A) 19.2 W (B) 19.2 kW
+ –
(C) 19.2 J (D) 12.2 kW
K1 K2
Page # 142 PREVIOUS YEAR PAPER [NEET - 2014]
27. The resistance in the two arms of the meter 30. In an ammeter 0.2% of main current passes
bridge are 5 and R, respectively. When through the galvanometer. If resistance of
the resistance R is shunted with an equal galvanometer is G, the resistance of ammeter
resistance, the new balance point is at 1.6 will be -
1 . The resistance 'R' is -
1 499
(A) G (B) G
499 500

1 500
(C) G (D) G
500 499

G 31. Two identical long conducting wires AOB and


COD are placed at right angle to each other,
A B
with one above other such that 'O' is their
(A) 10 (B) 15 common point for the two. The wires carry I1
(C) 20 (D) 25 and I2 currents respectively. Point 'P' is lying
at distance 'd' from 'O' along a direction
28. A potentiometer circuit has been set up for finding perpendicular to the plane containing the wires.
the internal resistance of a given cell. The main The magnetic field at the point 'P' will be -
battery, used across the potentiometer wire,
has an emf of 2.0 V and a negligible internal 0  I1  0
(A)   (B) (I + I2)
resistance. The potentiometer wire itself is 4m 2d  I2  2d 1
long, when the resistance R, connected across
the given cell, has values of. 0 0
(i) infinity (ii) 9.5  (C) (I12 – I22) (D) (I12 + I22)1/
2d 2d
The balancing lengths', on the potentiometer 2
wire are found to be 3m and 2.85 m,
respectively. The value of internal resistance
of the cell is. 32. A thin semicircular conducting ring (PQR) of
(A) 0.25  (B) 0.95  radius 'r' is falling with its plane vertical in a
(C) 0.5  (D) 0.75  horizontal magnetic field B, as shown in figure.
The potential difference developed across the
29. Following figures show the arrangement of ring when its speed is v, is -
bar magnetis in different configurations. Each + + ++ ++ + + +
 + + + + +Q + + B + +
magnet has magnetic dipole moment m . + + ++ ++ + + +
Which configuration has highest net magnetic + + ++ ++ + + +
+ + + + + +r + + +
dipole moment ? + + ++ ++ + + +
+ + + + + + + + +R
P+ + + + + + + + +
N (A) zero
(B) Bvr2/2 and P is at higher potential
N S
(a) (b) (C) rBv and R is at higher potential
S N
S S N (D) 2rBv and R is at higher potential

33. A transformer having efficiency of 90% is


N working on 200V and 3kW power supply. If
N the current in the secondary coil is 6A, the
(c) 30° (d) voltage across the secondary coil and the
S N 60°
S N current in the primary coil respectively are -
(A) 300 V, 15 A (B) 450 V, 15 A
(A) (a) (B) (b) (C) 450V, 13.5 A (D) 600 V, 15 A
(C) (c) (D) (d)
PREVIOUS YEAR PAPER [NEET - 2014] Page # 143
34. Light with an energy flux of 25 × 104 Wm–2 41. Hydrogene atom is ground state is excited
falls on a perfectly reflecting surface at by a monochromatic radiation of  = 975 Å.
normal incidence. if the surface area is 15 Number of spectral lines in the resultant
cm2, the average force exerted on the spectrum emitted will be
surface is - (A) 3 (B) 2
(A) 1.25 x 10–6 N (B) 2.50 x 10–6 N (C) 6 (D) 10
–6
(C) 1.20 x 10 N (D) 3.0 x 10–6 N
42. The binding energy per nucleuon of 73Li and
35. A beam of light of  = 600 nm from a distance
4 nuclei are 5.60 MeV and 7.06 MeV,,
source falls on a single slit 1 mm wide and 2 He
the resulting diffraction pattern is observed 7
on a screen 2m away. The distance between respectively. In the nuclear reaction 3 Li +
first dark fringes on either side of the central 1  4 + Q, the value of energy Q
1H 2 He
bright fringe is -
(A) 1.2 cm (B) 1.2 mm released is :-
(A) 19.6 MeV (B) –2.4 MeV
(C) 2.4 cm (D) 2.4 mm
(C) 8.4 MeV (D) 17.3 MeV
36. In the Young's double-slit experiment, the
43. A radio isotope 'X' with a half life 1.4 × 109
intensity of light at a point on the screen where
years decays to 'Y' which is stable. A sample
the path difference is  is K, ( being the wave of the rock from a cave was found to contain
length of light used). The intensity at a point 'X' and 'Y' in the ratio 1 : 7. The age of the
where the path difference is /4, will be - rock is -
(A) K (B) K/4 (A) 1.96 × 109 years (B) 3.92 × 109 years
(C) K/2 (D) zero (C) 4.20 × 109 years (D) 8.40 × 109 years

37. If the focal length of objective lens is 44. The given graph represents V-I characteristic
increased then magnifying power of :- for a semiconductor device.
(A) microscope will increase but that of telescope
(B) microscope and telescope both will increase
(C) microscope and telescope both will decrease I
(D) microscope will decrease but that of
A
telescope
V
d
38. The angle of a prism is 'A'. One of its refracting
surface is silvered. Light rays falling at an
angle fo incident 2A on the first surface
Which of the following statement is correct ?
returns back through the same path after
(1) It is V-I characterstic for solar cell where,
suffering reflection at the silvered surface. point A represents open circuit voltage and
The refractive index , of the prism is - point B short circuit current.
(A) 2 sin A (B) 2 cos A (B) It is for a solar cell and point A and B
1 represent open circuit voltage and current,
(C) cos A (D) tan A
2 respectively.
(C) It is for photodiode and points A and B
39. When the energy of the incident radiation is represent open circuit voltage and current
increased by 20%, the kinetic energy of the respectively.
photoelectrons emitted from a metal surface (D) It is for a LED and points A and B represent
increased from 0.5 eV to 0.8 eV. The work open circuit voltage and short circuit current,
function of the metal is - respectively.
(A) 0.65 eV (B) 1.0 eV
(C) 1.3 eV (D) 1.5 eV 45. The barrier potential of a p-n junction
depends on :
40. If the kinetic energy of the particle is increased (a) type of semi conductor material
to 16 times its previous value, the percentage (b) amount of doping
change in the de-Broglie wavelength of the (c) temperature
particle is - Which one of the following is correct ?
(A) (a) and (b) only (B) (b) only
(A) 25 (B) 75
(C) (b) and (c) only (D) (a), (b) and (c)
(C) 60 (D) 50
Page # 144 PREVIOUS YEAR PAPER [NEET - 2014]

CHEMISTRY
1. What is the maximum number of orbitals that 10. Using the Gibbs energy change,
can be identified with the following quantum Go = + 63.3kJ, for the following reaction,
numbers? 
n = 3,  == 1, m = 0 Ag2CO3  2A g (aq) + CO32– (aq)
(A) 1 (B) 2 the Ksp of Ag2CO3(s) in water at 25°C is : –
(C) 3 (D) 4 (R = 8.314 J K–1 mol–1)
(A) 3 .2 x 10–26 (B) 8 .0 x 10–12
2. Calculate the energy in joule corresponding (C) 2.9 x 10–3 (D) 7.9 x 10–2
to light of wavelength 45 nm :
(planck's constant h = 6.63 x 10–34 Js; speed 11. The weight of silver (at wt. = 108) displaced
of light c = 3 x 108 ms–1) by a quantity of electricity which displaces
(A) 6.67 x 1015 (B) 6.67 x 1011 5600 mL of O2 at STP will be :–
–15
(C) 4 .42 x 10 (D) 4.42 x l 0–18 (A) 5.4 g (B) 10.8 g
(C) 54.0 g (D) 108.0 g
3. Equal masses of H2,O2 and methane have
been taken in a container of volume V at 12. Which of the following statements is correct
temperature 27°C in identical conditions. The for the spontaneous adsorption of a gas ?
ratio of the volumes of gases H 2 : O 2 : (A) S is negative and, therefore, H should
methane would be : be highly positive
(A) 8: 16 : 1 (B) 16 : 8 : 1 (B) S is negative and therefore, H should
(C) 16: 1 : 2 (D) 8 : 1 : 2 be highly negative
(C) S is positive and, therefore, H should
4. If a is the length of the side of a cube , the be negative
distance between the body centered atom (D) S is positive and, therefore, H should
and one corner atom in the cube will be : also be highly positive
2 4
(A) a (B) a
3 3 13. For the reversible reaction :
3 3 N2(g) + 3H2(g) 2NH3(g) + Heat
(C) a (D) a The equilibrium shifts in forward direction :
4 2
(A) By Increasing the concentration of NH3(g)
5. Which property of colloids is not dependent (B) By decreasing the pressure
on the charge on colloidal particles ? (C) By decreasing the concentrations of N2(g)
(A) Coagulation (B) Electrophoresis and H2(g)
(C) Electro–osmosis (D) Tyndall effect (D) By increasing pressure and decreasing
temperature
6. Which of the following salts will give highest
pH in water ? 14. For the reaction:
(A) KCl (B) NaCl X2O4()  2XO2(g)
(C) Na2CO3 (D) CuSO4 U = 2.1 k cal, S = 20 cal K–l at 300 K
Hence G is :–
7. Of the following 0.10m aqueous solutions, (A) 2.7 k cal (B) –2.7 k cal
which one will exhibit the largest freezing (C) 9.3 k cal (D) –9.3 k cal
point depression?
(A) KCI (B) C6H12O6 15. For a given exothermic reaction, Kp and K'P
(C) Al2(SO4)3 (D) K2SO4 are the equilibrium constants at temperatures
T1 and T2, respectively. Assuming that heat of
8. When 22.4 litres of H2(g) is mixed with 11.2 reaction is constant in temperature range
litres of Cl2(g), each at S.T. P. , the moles of between T1 and T2, it is readily observed that :–
HCl (g) formed is equal to : (A) KP > KP' (B) KP < KP'
(A) 1 mol of HCl (g) (B) 2 mol of HCl (g) 1
(C) 0 .5 mol of HCl (g)(D) 1.5 mol of HCl (g) (C) KP = KP' (D) KP = K'
P

9. When 0.1 mol MnO42– is oxidised the quantity


of electricity required to completely oxidise 16. Which of the following orders of ionic radii is
MnO42– to MnO4– is : – correctly represented ?
(A) 96500 C (B) 2 x 96500 C (A) H– > H+ > H (B) Na+ > F– > O2–
(C) 9650 C (D) 96.50 C (C) F– > O2– > Na+ (D) Al3+ > Mg2+ > N3–
PREVIOUS YEAR PAPER [NEET - 2014] Page # 145
17. 1.0 g of magnesium is burnt with 0.56 g O2 in 27. Among the following complexes the one
a closed vessel. Which reactant is left in which shows zero crystal field stabilization
excess and how much? energy (CFSE) is :–
(At. wt. Mg = 24 ; O = 16) (A) [Mn(H2O)6]3+ (B) [Fe(H2O)6]3+
(A) Mg, 0.16 g (B) O2 , 0.16 g (C) [Co(H2O)6]2+ (D) [Co(H2O)6]3+
(C) Mg, 0.44 g (D) O2, 0.28 g
28. Magnetic moment 2.83 BM is given by which
18. The pair of compounds that can exist
of the following ions?
together is:
(A) FeCl3, SnCl2 (B) HgCl2, SnCl2 (At. nos. Ti = 22, Cr = 24, Mn = 25, Ni = 28):
(C) FeCl2, SnCl2 (D) FeCl3. KI (A) Ti3+ (B) Ni2+
(C) Cr3+ (D) Mn2+
19. Be 2+ is isoelectronic with which of the
following ions? 29. Which of the following complexes is used to
(A) H+ (B) Li+ be as an anticancer agent?
(C) Na+ (D) Mg2+ (A) mer-[Co(NH3)3Cl3]
(B) cis-[PtCl2(NH3)2]
20. Which of the following molecules has the (C) cis-K2[PtCl2Br2]
maximum dipole moment ? (D) Na2CoCl4
(A) CO2 (B) CH4
(C) NH3 (D) NF3
30. Reason of lanthanoid contraction is :–
(A) Negligible screening effect of 'f' orbitals
21. Which one of the following species has plane
triangular shape ? (B) Increasing nuclear charge
(C) Decreasing nuclear charge
(B) N3 (C) NO3
(D) Decreasing screening effect
(C) NO2 (D) CO2
31. In the following reaction, the product (A)
22. Acidity of diprotic acids in aqueous solutions
increases in the order :– +
NH 2
(A) H2S < H2Se < H2 Te
(B) H2Se < H2S < H2Te +
H+
(C) H2Te < H2S < H2Se (A) is
(D) H2Se < H2Te < H2S Yellow dye

23. (a) H2O2 + O3  H2O + 2O2


(b) H2O2 + Ag2O  2Ag + H2O + O2 (A) N=N—NH
Role of hydrogen peroxide in the above
reactions is respectively –
NH2
(A) Oxidizing in (a) and reducing in (b)
(B) Reducing in (a) and oxidizing in (b) (B) N=N
(C) Reducing in (a) and (b)
(D) Oxidizing in (a) and (b)
NH 2
24. Artificial sweetner which is stable under cold
conditions only is :– (C) N=N
(A) Saccharine (B) Sucralose
(C) Aspartame (D) Alitame

25. In acidic medium, H2O2 changes Cr2O7–2 to (D) N=N NH 2


CrO5 which has two (–O–O) bonds. Oxidation
state of Cr in CrO5 is :–
(A) +5 (B) +3 32. Which of the following will be most stable
(C) +6 (D) –10
diazonium salt RN2 X ?
26. The reaction of aqueous KMnO4 with H2O2 in
(A) CH3N2 X  (B) C6H5N2 X 
acidic conditions gives :–
(A) Mn4+ and O2 (B) Mn2+ and O2
2+ (C) CH3CH2N2 X  (D) C6H5CH2N2 X 
(C) Mn and O3 (D) Mn4+ and MnO2
Page # 146 PREVIOUS YEAR PAPER [NEET - 2014]
33. D(+) glucose reacts with hydroxylamine and 37. Which one of the following is not a common
yields an oxime. The structure of the oxime component of Photochemical Smog?
would be : (A) Ozone (B) Acrolein
(C) Peroxyacetyl nitrate
(D) Chlorofluorocarbons
CH=NOH CH=NOH
H—C—OH HO—C—H 38. In the Kjeldahl's method for estimation of
nitrogen present in a soil sample, ammonia
HO—C—H HO—C—H evolved from 0.75 gm of sample neutralized
HO—C—H H—C—OH 10 mL of 1 M H2SO 4 , The percentage of
(A) (B) nitrogen in the soil is ;
H—C—OH H—C—OH (A) 37.33 (B) 45.33
(C) 35.33 (D) 43.33
CH2OH CH2OH
39. What products are formed when the following
compound is treated with Br2 in the presence
CH=NOH CH=NOH of FeBr3 ?
CH 3
HO—C—H H—C—OH
H—C—OH HO—C—H
HO—C—H H—C—OH CH 3
(C) (D)
H—C—OH H—C—OH CH3 CH 3
Br
CH2OH CH2OH
and
(A)
CH 3 CH 3
34. Which of the following hormones is produced Br
under the condition of stress which stimulates CH3 CH3
glycogenolysis in the liver of human beings? Br Br
(A) Thyroxin (B) Insulin
(B) and
(C) Adrenaline (D) Estradiol
CH3 CH 3
35. Which one of the following is an example of CH3 CH3
a thermosetting polymer? Br
(CH2—C=CH—CH2)n and
(A) (C)
Cl CH3 CH 3
Br
(CH2—CH)n
(B) CH3 CH 3
Cl
N H O O and
(C) (D)
(N—(CH2)6—N—C—(CH2)4—C)n CH 3 Br CH 3
Br
OH OH
CH2 CH2 40. Which of the following compounds will
undergo racemisation when solution of KOH
(D)
hydrolyses?
n CH 2Cl
(i) (ii) CH3CH2CH2Cl
36. Which of the following organic compounds CH3
polymerizes to form the polyester Dacron? CH3
(A) Propylene and para HO – (C6H4) – OH C
(iii) H3C—CH—CH2Cl (iv) H Cl
(B) Benzoic acid and ethanol C2H5
(C) Terephthalic acid and ethylene glycol
(A) (i) and (ii) (B) (ii) and (iv)
(D) Benzoic acid and para HO – (C6H4) – OH (C) (iii) and (iv) (D) (i) and (iv)
PREVIOUS YEAR PAPER [NEET - 2014] Page # 147

41. Among the following sets of reactants which 44. Identify Z in the sequence of reactions:
one produces anisole ? HBr / H2O2 C H ONa
2 5
(A) CH3CHO ; RMgX CH3CH2CH = CH2   Y  Z
(A) CH3 – (CH2)3 – O – CH2CH3
(B) C6H5OH ; NaOH ; CH3l
(C) C6H5OH ; neutral FeCl3 (B) (CH3)2CH2 – O – CH2CH3
(D) C6H5 – CH3 ; CH3COCl; AlCl3 (C) CH3(CH2)4 – O – CH3
(D) CH3CH2 – CH(CH3)–O–CH2CH3
42. Which of the following will not be soluble in
sodium hydrogen carbonate? 45. Which of the following organic compounds
(A) 2, 4, 6-trinitrophenol has same hybridization as its combustion
product CO2 ?
(B) Benzoic acid
(C) o-Nitrophenol (A) Ethane (B) Ethyne
(C) Ethene (D) Ethanol
(D) Benzenesulphonic acid

43. Which one is most reactive towards


Nucleophilic addition reaction?

CHO COCH3
(A) (B)

CHO CHO

(C) (D)

CH3 NO2
Page # 148 PREVIOUS YEAR PAPER [NEET - 2014]

ANSWERKEY
BIOLOGY
1. Bonus 2. A 3. C 4. B 5. A 6. C 7. B
8. D 9. B 10. A 11. A 12. C 13. D 14. C
15. A 16. D 17. A 18. A 19. A 20. A 21. C
22. B 23. B 24. C 25. B 26. B 27. A 28. B
29. A 30. D 31. D 32. C 33. D 34. A 35. D
36. D 37. A 38. A 39. A 40. B 41. D 42. B
43. B 44. B 45. D 46. A 47. D 48. D 49. D
50. B 51. B 52. A 53. D 54. C 55. A 56. A
57. A 58. B 59. B 60. D 61. A 62. B 63. D
64. D 65. D 66. D 67. A 68. C 69. D 70. D
71. A 72. A 73. C 74. C 75. B 76. B 77. B
78. D 79. C 80. C 81. D 82. A 83. C 84. B
85. D 86. C 87. B 88. B 89. A 90. D

PHYSICS

1. D 2. A 3. D 4. C 5. C 6. A 7. B
8. C 9. d 10. A 11. C 12. A 13. B 14. C
15. D 16. A 17. C 18. D 19. B 20. A 21. D
22. C 23. C 24. B 25. D 26. B 27. B 28. C
29. C 30. C 31. D 32. D 33. B 34. B 35. D
36. C 37. D 38. B 39. B 40. B 41. C 42. D
43. C 44. A 45. D

CHEMISTRY

1. A 2. D 3. C 4. D 5. D 6. C 7. C

8. A 9. C 10. B 11. D 12. B 13. D 14. B

15. A 16. B 17. A 18. C 19. B 20. C 21. B

22. A 23. C 24. C 25. C 26. B 27. B 28. B

29. B 30. A 31. D 32. B 33. D 34. C 35. D

36. C 37. D 38. A 39. C 40. B 41. B 42. C

43. D 44. A 45. B


PREVIOUS YEAR PAPER [NEET - 2014] Page # 149

SOLUTION
PHYSICS
By consevation of linear momentum
 Force   Force 
1. (mass) =  Accelerati on  =  Velocity / time 
    v
2mv1 = 2 mv  v1 =
= [F V–1 T] 2

1 1 1
u2 sin 2 Total KE generated = v2 +
mv v2 +
mv (2m)v12
2 2 2
2. As range = so g  u2
g
mv 2 3
2 = mv2 + = mv2
3 2 2
Therfore, gplanet =   (9.8 m/s2) = 3.5 m/s2
5
8. Displacement, x = A cos (t)
 dx
 r (13  2)î  (14  3)ĵ 11 Velocity, v = = –A
Asin (t)
3. v av = = = (i + j) dt
t 50 5

dv
4. Acceleration Acceleration, a = = –A2 cos (t)
dt
Net force in the direction of motion
=
Total mass of system 9. Here  = 2 revolutions/s2 = 4 rad/s2

m1g  (m2  m3 )g g 1 1 25
= = (1 – 2) I= MR2 = (50) (0.5)2 = Kg-m2
m1  m2  m3 3 2 2 4
As  = I so TR = I
5. Change in momentum,
 25 
I  (4)
p =  Fdt T=
R
=  4  N
= Area of F-t graph (0.5)

1 = 50 N = 157 N
= ×2×6–3×2+4×3
2
= 12 N-s 10. For rolling motion without slipping on inclined
plane
6. Let upthrust of air be Fa then
g sin 
for downward motion a1 =
mg – Fa = ma K2
for upward motion and for slipping motion on inclined plane
Fa – (m – m) = (m – m) a a2 = g sin 
2ma 1
Therefore m = a1 5
g a
Required ratio = a = 1  2 =
2
5 7
7.

v
2GM
m 11. Escape velocity = = c = seed of light
R
m
v 2GM 2  6.6  10 11  5.98  1024
R= =
c 2
(3  108 )2
2m
v1 = 10–2 m
Page # 150 PREVIOUS YEAR PAPER [NEET - 2014]
17. For isothermal process P1V1 = P2V2
F
P
A F  PV = P2(2V)  P2 =
13. Y =    = 2
AY
 For adiabatic process P2V2 = P3V3

P 
V   (2v) = P3 (16v)
But V = A so A = 2

5 /3
F 2 F 2 3 1 P
Therefore  =  2  P3 =   =
VY VY 2 8 64

14. As surface area decreases so energy is 18. Work done by the system in the cycle
released. = Area under P-V curve & V-axis
Released energy
1
= 4R2T [n1/3 – 1] where R = n1/3 r = (2P0 – P0) (2V0 – V0) +
2
1 1 
= 4R3T  r  R   1 
    (3P0  2P0 )(2V0  V0 )
  2  
1 1 
= 3VT  r  R  P0V0 P0V0
  =  =0
2 2

15.
Heat lost = Heat gained 1
19. Mean free path m =
mLv + msWw = mWsW 2d2n
 m × 540 + m × 1 × (100 – 80) where d = diameter of molecule
= 20 × 1 × (80 – 10)
1
 m = 2.5 g  m 
r2
Total mass of water = (20 + 2.5) g = 22.5 g

16. By Newton's law of colling 20. Total length of string   1   2   3

1  2  1  2  1
=k   0  But frequency 
t  t  length

70  60  70  60  1 1 1 1
 =k   0  so n  n  n  n
5  2  1 2 3

 2 = k [65 – 0] ....(i)


v
60  54  60  54  21. Frequency COP, fn = (2n + 1)
and =k   0  4R
5  2 
for
6 n = 0, f0 = 100 Hz
 = k [57 – q0] .....(ii) n = 1, f1 = 300 Hz
5
n = 2, f2 = 500 Hz
By dividing (i) by (ii) we have
n = 3, f3 = 700 Hz
10 65  0 n = 4, f4 = 900 Hz
= 37   n = 5, f5 = 1100 Hz
5 0
Which are less than 1250 Hz
  = 45°
PREVIOUS YEAR PAPER [NEET - 2014] Page # 151
22. Apparent frequency

29. Net magnetic moment = 2Mcos
 v  v0   343  10  2
n' = n  v  v  = 1392  343  5  which is maximumfor option (3)
 s   
30.
= 1412 HZ
9981
1 1000 S
23. Electric field, E 
K
As K1 < K2 so E1 > E2
I 21
Q 1000
24. At centre, E = 0 & V = 4  R
0

 21   9981 
 G=   S
 1000   1000 
 V V V
25. î  ĵ  k̂
E = – x y z G
S=
499
= [(6  8y)î  (8x  8  6z) ĵ  (6y)k̂] Total resistance of ammeter

At (1,1,1), E  2 î  10 ĵ  6k̂
 G 
 F
  499 

= E = 22  102  62 = 140 = 2 35 R=
SG
=  G  =
G
SG  G 500
Force = qE = 2 × 2 35 = 4 35 N  499 

26. Resistance = (0.5/km) (150 km) = 75


Total voltage drop = (8V/km) (150 km) = 1200 V 31. Net magnetic field, B = B12  B22

(V)2 (1200)2 2 2
Power loss = = W  0I1   I 
R 75 =    0 1
 2d   2 d 
= 19200 W - 19.2 kW
0
= I12  I22
2d
5 1 5 1.61
27. = 100   and = 100  1.6
R 1 R / 2 1

 R = 15  x x x x x
Q
x x x x x
r
E  V  32.
28. Internal resistance, r =  R = x x x x
 V  P R
2r

 2  2  x x x x x
  R
 2 
x x x x x
= – +
P R
 3  2 .85  x x x x
=   (9.5) = 0.5 = 0.5
 2.85  Induced emf = Bv (2r) = 2rBv
Page # 152 PREVIOUS YEAR PAPER [NEET - 2014]
39. By using hv = 0 + Kmax
VSIS VS (6)
33.  = V I  0.9 =  VS = 450 V We have
P P 3  103 hv = 0 + 0.5 ....(i)
and 1.2hv = 0 + 0.8 ....(ii)
3000 Therefore 0 = 1.0 eV
As VPIP = 3000 so IP = A = 15A
200

p 2IA h
34. Average force Fav = = 40. =
t c 2mK

2  25  104  15  104 1 K2 16K 4


=
3  108 2 = K1 = K
=
1
= 2.50 × 10–6 N
14
Percentage change = × 100 = –75%
35. Width of central bright fringe 4

2D 2  600  106  2


= = m 42. BE of 2He4 = 4 × 7.06 = 28.24 MeV
 1  103
= 2.4 × 10–3 m Be of 7 Li = 7 × 5.60 = 39.20 MeV
3
= 2.4 mm 7
3 Li + 11H  2He4 + 2He4 + Q
36. For path difference , phase difference = 2 rad 39.20 28.24 × 2
Q = 56.48 – 39.20 = 17.28 MeV

For path difference , phase difference =
4 2
Nx 1 Nx 1 1
 43. As N =  N N = =  
rad As K = 4I0 so intensity at given point y 7 x y 8 2
2
So t = 3T1/2 = 3 × 1.4 × 109 yrs.
 = 4.2 × 109 yrs.
where path difference is
4
44.
 K
K' = 4I0 cos2  4  = 2I0 =
  2 45.

LD 1
37. Magnifying powe rof Microscope = f f  f
0 e 0

f0
Magnifying power of telescope = f  f0
e

38.

A
A
°–
90
2A A

By Snell's law
(1) sin 2A = () sin A   = 2 cos A
PREVIOUS YEAR PAPER [NEET - 2014] Page # 153

CHEMISTRY

1. n = 3,  = 1, m = 0
6 7
Orbital is 3pz. 9. Mn O42  MnO4  e

0.1 mole 0.1 mole


34 8
hc 6.63  10  3  10 charge required = 0.1 F = 0.1 × 96500
2. E= =
 45  109 = 9650 C
–18
E = 4.42 × 10 J
10. G° = –2.303 RT log Ksp
3. According to Avogadro's hypothesis 63.3 × 1000 = –2.303 × 8.314 × 298 log Ksp
volume  moles log Ksp = –11.09
Ksp = 10–11.09 = 8 × 10–12
w
nH2 =
2
11. According to faraday's 2nd law
w
nO2 =
32 WAg WO2
EAg = EO2
w
nCH4 =
16
 5600 
w w w WAg  22400   32
So, ratio is : :  
=
2 32 16 108 8
= 16 : 1 : 2
 WAg = 108 g

4. The distance between the body centred atom


12. During adsorption entropy decreases,
3a so S < 0
and one corner atom is
2 G = H – TS
For spontaneous adsorption G < 0 so H
5. Tyndall effect is optical property. should be highly negative.

6. Na2CO3 will give highest pH in water because 13. According to Le-Chatelier's Principle
it is salt of strong base and weak acid  In exothermic reactions low temperature
favours the forward reaction
7. Depression in freezing point  vant Hoff's
 On increasing pressure equilibrium shifts
factor (i) for Al2(SO4)3  i = 5
towards less number of moles.

V L  22.4 14. X2O4()  2 XO2(g); ng = 2 – 0 = 2


8. nH2 = = =1
22.4L 22.4 H = U + ngRT

11.2 2
nCl2 = = 0.5 mole = 2.1 + 2 × × 300
22.4 1000

H2(g) + Cl2(g)  2HCl(g) H = 3.3 kcal


initially – 1 mole 0.5 mole 0 G =H – T.S
after reaction (1–0.5) 0.5 × 2
= 0.5 mole 0 = 1 mole 20
= 3.3 – 300 × ; G = –2.7 kcal
1000
Page # 154 PREVIOUS YEAR PAPER [NEET - 2014]
15. In exothermic reactions on increasing 27. Due to d5 configuration and H2O is a weal
temperature value of Kp decreases. ligand.
So, Kp > KP'
28. Ni+2 has two unpaired electron.
16. B
(3d8 configuration)

1 0.56 29. Cis-platin is used as an anticancer agent.


17. nMg = mole, nO2 = moles
24 32

30. Due to poor shielding of f-orbitals, nucleus


1
Mg(s) + O (g)  MgO(s) will exert a strong attraction, Causes
2 2
lanthanoid contraction.
1 0.56
Initially mole mole
24 32 31. This is an example of electrophilic substitution
0.0416 mole 0.0175 mole 0 reaction [coupling reaction]
after (0.0416 – 2 × 0.0175) 0 2 × 0.0175 mole
reaction 0.0066 mole
 mass of Mg = 0.0066 × 24g = 0.16 g

18. Both are reducing agent +


NH 2

19. Li+, Be+2 & Li+ both have 2 electron.

N=N NH2

N > N
20.
H H H F F
F

 = 1.4 D 0.23 D

21. NO3 has Sp2 hybridisation i.e. has planar shape.


32. Primary aliphatic amines form highly unstable
22. On moving down the group bond length alkyldiazonium salts. Primary aromatic amines
increases so liberation tendency of H will be form arene diazonium salts which are stable
more. for a short time in solution at low temperature
(273 – 278 K). The stability of arendiazonium
23. C can be explained on the basis of resonance.

24. C
Salt

N=N N=N N=N


25. CrO5 has 2 peroxy linkage.

26. KMnO4 is a strong oxidlsing agent & will oxidise


H2O2 to O2.
PREVIOUS YEAR PAPER [NEET - 2014] Page # 155
33. Glucose reacts with hydroxyl amine to form 38.  M x V (ml) = m mol
an oxime. 10 m mol H2SO4 = 20 m mol of NH3
CHO CH=N—OH
[H2SO4 + 2NH3  (NH4)2SO4]
1 mol NH3 contains 14 g nitrogen
H OH H OH
20 X 10–3 mol NH3 contains 14 x 20 x 10–3
HO H NH2OH HO H
nitrogen 0.75 g of sample contains
H OH H OH
H OH H OH 14  20  103
% Nitrogen = × 100 = 37.33 %
CH2—OH CH2—OH 0.75

D(+)glucose
CH3 CH3 CH 3
Br
34. Adrenaline commonly known as fight or flight Br2
+
hormone, it is produced by the adrenal glands FeBr3
CH3 CH3 CH3
39.
after receiving a message from the brain that Br
a stressful situation has presented itself. (major) (minor)

35. Thermosetting polymers are cross linked or In the above compound 1 ,3-Dimethyl
heavily branched molecules, which on heating benzene, sites for the attacking electrophile
undergo extensive cross linking in moulds and are
again become infusible. Most common
example is bakelite.
CH3
a c
36. Dacron or terylene is the best known example
of polyesters. It is manufactured by heating
a mixture of ethylene glycol and terephthalic
acid at 420 to 460 K in the presence of zinc b
acetate-antimony trioxide catalyst.

attack of electrophile on sites a & b


results in same compound as product.
nHO—CH2—CH2—OH + nHOOC COOH Although tendency of electrophile to attack
Ethylene glycol
on site c is very less due to high steric
[Ethane-1,2-diol] Terephthalic acid
hinderance so respective product is favoured
[Benzene-1,4-di
carboxylic acid] with very very less amount.

O O
—OCH2—CH2—O—C C—O— CH3
n C
Terylene or Dacron 40. Only compound(iv)
H Cl results in
C2H5

formation of racemic product due to chirality.


37. The common components of photochemical
smog are ozone, nitric oxide, acrolein,
formaldehyde and peroxyacetyl nitrate (PAN).
OH O Na O–CH 3
Hence chlorofluoro carbon is not common
component of photochemical smog. 41. NaOH CH3l
–H2O (SN 2)
Page # 156 PREVIOUS YEAR PAPER [NEET - 2014]

HBr /H2O2
44. CH3–CH2CH=CH2 
Peroxide effect 

OH O Na
NO2
42. + H 2 CO 3 CH3—CH2—CH2—CH2
(y)
Br
C2H5ONa
(carbonic acid) while 2.4.6-Trinitro phenol,
benzoic acid and benzene sulphonic acid are
CH3—(CH2)3—O—CH2—CH3
soluble in NaHCO3.
(z)

OR
5
45. C2H2 + O 2CO2 + H2O
Acid + NaHCO3  salt + H2CO3 2 2
Reaction is possible in forward direction if Both HC  CH & CO2 has same hybridisation
acid is more acidic then H2CO3. of carbon atom. (sp).
O-nitrophenol is less acidic than H 2 CO 3 ,
hence does not soluble in sodium hydrogen
carbonate.

43. Reactivity of carbonyl compounds towards


NAR depends on steric and electronic effects.
NAR reactivity :

CHO CHO CHO COCH3

> > >

NO2 CH3

–M of –NO2

increase (+)ve charge on Sp2 C of —C—


O

You might also like